Sie sind auf Seite 1von 123

~ SHORT NOTES ~

by T.I.D.E team

Together in Delivering Excellence (T.I.D.E)

MEDICAL BASED
SURGERY
BASED

INDEX
MEDICAL
PAEDIATRIC
PSYCHIATRIC
SURGERY
O&G
ORTHOPAEDIC

2
30
53
59
80
108

Ya Allah, Kuatkan Ingatan Kami Terhadap Apa Yang Kami Baca


Ya Allah, Tutuplah Segala Kesalahan Jawapan Kami Semasa Dalam Peperiksaan
Ya Allah, Berikanlah Ilham kpd Kami Untuk Menjawab Sepertimana Yang Pensyarah Kami Mahukan
Ya Allah, Kurniakan Pensyarah Yang Baik Hati Dan Pemurah Sebagai Pemeriksa Kami
Ya Allah, Kurniakanlah Pesakit Yang Baik Hati Dan Dapat Memberi Kerjasama Kepada Kami Nanti
Ya Allah, Semoga Apa Yang Kami Baca Dan Paham Sahaja Yang Ditanya Dalam Peperiksaan Nanti
Aamiin.

Special appreciation to members of T.I.D.E Team

Copyright @ 2015
All Rights Reserved
Final Year Medical Student (USM)

Together in Delivering Excellence (T.I.D.E)

MEDICAL
Notes

Together in Delivering Excellence (T.I.D.E)


Approach to Chest Pain [IQbaL]
HISTORY
1.

2.

3.
4.
5.

Nature of chest pain ( SOCRATES )


Constricting (cardiac ischemic or oesophageal spasm)
Dull, central & crushing, last for 20min (MI)
Radiates to jaw & upper extremities (cardiac cause)
Sharp pleuritic pain that catch on inspiration (pleura
or pericardium) & suggest pneumonia, pul. embolism
or pericarditis
Sudden substernal tearing & radiate to back ( aortic
dissection )

PHYSICAL EXAMINATION
1.
2.
3.
4.
5.

Abnormality of pulse rate and heart sound (cardiac)


Crepitation pneumonia or HF
Reduced breath sound in one side pneumothorax or
lung collapsed
Tenderness on chest may causes from MSS however MI
can present with chest tenderness
GIT origin in normal cardiac and respi system

Pain brought on by food, lying down, hot drinks, or


alcohol, and relieved by antacids GIT causes (eg GERD,
PUD, oesophaeal spasm)
PMHx known cardiac dz, HPT, HPL, smoking, FHx can
support diagnosis
Acute cholecyctitis & pancreatitis can cause pain referred
to chest
Associated Sx dyspnoea cardiac ischemia, PE,
pneumothorax or pneumonia

INVESTIGATION
Basic Ix
1. 12 lead ECG unless non-cardiac causes is confidently being
diagnosed eg pneumothorax
ST, QRS, arrhymias, tachy/brady
Pericarditis widespread concave ST, PR depression
2.

CXR
can confirm respi disorder eg pneumothorax,
pneumonia
can provide clues in cardiac dz (widened mediastinum
in aortic dissection or a large globular heart in cardiac
tamponade)

3.

Echocardiograhy

Laboratory
a) Cardiac biomarker CK-MB, Troponin I & T
b) FBC infection & screen for anaemia
c) RFT baseline
d) TFT
# some DDx can be excluded/confirmed after basic Hx, PE and
these Ix STEMI, pneumothorax, pneumonia, pericarditis

MANAGEMENT
1)
2)

Management will depends on diagnosis


Psychological tx may be helpful in some pt

Together in Delivering Excellence (T.I.D.E)


Diagnostic Approach for Dyspnoea [IQbaL]
Diagnostic
Hypothesis
COPD

Asthma

Pulmonary
Etiologies
Pulmonary
embolism

Pneumonia
(CAP, TB,
Pneumocystic
jiroveci
pneumonia)

Intertitial lung
disease (ILD)

Clinical Clues
History
>20 pack years tobacco

Chronic cough +sputum

Progressive/persistent dyspnea
Exposure to occupational
dust/chemical

Test

Treatment

Spirometry (FEV1/FVC
<70%)
Bronchodilator response
largely irreversible
CXR hyperinflation,
bullous changes, pul HPT
ECG cor pulmonale ( peak
P wave @ L2,L3 and AVF )
ABG

Nebulizer
bronchodilator, O2
Antibiotic (H. influenza,
Strep pneumonia)
Steroid ( beneficial in
acute exacerbation of
COAD)

Wheezing
Assessing for severe
asthma and lifethreatening Sx

Sudden onset of dyspnea,


peuritic chest pain
Hx of cancer
Hx of surgery, immobilization
Estrogen therapy

Tachypnea, cyanosis
JVP, loud P2, gallop
rhythm
Unilateral leg
swelling

D-dimer exclude PE if
normal
CT angiopraphy
Leg Duplex

Fever, productive cough


Drug injection
High-risk sexual exposures

CXR
FBC, Blood culture
HIV, CD4 (when
appropriate)

Cold, exercise, allergen, pets


symptoms worsening
Fmly Hx (atopic)

Cardiac
Etiology

O2 100%
Morphine + antiemetic
Immediate thrombolysis
in massive PE ( bolus
alteplase or surgery)
IV Heparin
O2, treat shock
Empirical antibiotic
IV fluid
PRN analgesic

CXR - Honeycombing
PFT
High resolution chest
CT
Lung biopsy

Steroid /
cyclophosphamide
Tx of underlying Dz

Chest pain
CAD risk factor

JVP
S3
crackles

ECG
Biomakers
Stress test
Angiography

MONA

Rheumatic heart disease

Significant murmurs

Arrhythmias

Anaemia

ABG
O2, nebulizer (B-agonist)
High dose steroid
For severe attack IV
aminophylline, consider
ventilation

Fine basal Crackles


Clubbing
Sx of pul HPT, cor
pulmonale & CTD

Palpitation

Heart failure

Crackles, fever thrush


Kaposi sarcoma
Skin pop marks
Severity CURB-65

PFT
Bronchodilator response
Methacholine induced
CXR TRO pneumothorax

Known connective tissue


disease (CTD)
Raynaud phenomenon
Occupational exposure
asbestos, silica
Sarcoidosis

ACS

Valvular heart
disease

Physical
Dec breath sounds,
wheezing
Clubbing
superimposed
bronchogenic CA,
chronic infection

CAD or risk factor


Poorly controlled HPT
PND
Alcohol abuse

Melaena
Menorrhagia
Rectal bleeding

Irregular pulse
Variable intensity of
S1

JVP
S3
Crackles
Peripheral oedema

Pallor, cachexia
Pale conjunctiva
Thalassemic facies
Gum hypertrophy

Echo
ECG absent P wave
Holter paroxysmal AF
Echo valvular defect, Lt
atrial thrombus
TFT

Identify underlying cause


Cardioversion (electrical
or pharmacological)
Control ventricular rate
(digoxin)
Antocoagulation to
prevent
thromboembolism

CXR
Echo
BNP

HCT level (low)

Sit pt upright
O2 100%, IV access
Treat any arrhythmias
Monitor ECG
Furosemide, dimorphine

Together in Delivering Excellence (T.I.D.E)


Careful Hx-taking is very important
1) Time course
Acute (within min) life-threatening eg. Acute pul Embolism, MI, pneumothorax, anaphylaxis, FB aspiration, pul.
oedema or cardiac temponade
Subacute (hours-days) AEBA, exacerbation of COPD, pul. oedema.
Chronic (weeks-months) CCF, COPD, cardiomyopathy, pul. HPT, valvular heart dz, anaemia
2) Severity
3) Associated Sx
Fever - pneumonia, bronchitis, laryngitis, viral causes, sepsis
Fever + cough community-acquired pneumonia or opportunistic infection in immunocompromised host (need CXR to
exclude pneumonia)
Central chest pain suggest CAD, pul embolism, pneumothorax, FB aspiration
Pleuritic chest pain suggest pleuritis, pneumonia, pneumothorax
Palpitation paroxysmal tachyarrhythmias, pul. embolism, valvular heart dz or anxiety attack
Wheezing Asthma, COPD, pul. oedema, bronchiolitis, FB aspiration
Haemoptysis exacerbation of bronchiectasis, bronchitis, chest malignancies, tuberculosis, cocaine toxicity
Dysphagia FB aspiration, tetanus, epiglottitis (+drooling saliva), GERD (+heart burn)
Bone pain sickle cell anaemia or fat embolism (a/w long bone #)
4) Position
Orthopnea CCF, COPD
Dyspnea on standing, relieve by supine pattern foramen ovale
5) Pattern
Appear during working, resolve during period off work occupational exposure
Seasonal asthma or reactive airway disease
6) Smoking Hx COPD, lung Ca, interstitial lung disease

Questions
1.

Common causes of chest pain?


ACS, Stable angina, pul embolism, pneumonia, viral pleuritis, GERD, anxiety & panic disorder

2.

Life threatening chest pain? [PETAAA]


PE, Esophageal rupture, Tension pneumothorax, AMI, Angina/ACS, Aortic disection

Together in Delivering Excellence (T.I.D.E)


Acute Coronary Syndrome [Julea]
HISTORY
NSTEMI/UA : incomplete occlusion, occur at rest
STEMI: complete occlusion, not relieved by rest/GTN
Chest pain (LORDSANFARO)
Retrosternal/central/left chest
Crushing/pressing/burning in nature
Radiate to jaw/left upper limb
A/w profuse sweating, n&v, sob, palpitation

PHYSICAL EXAMINATION
CVS examination
Diagnosis of ACS:
I.
Hx of ischemic type chest pain
II.
Ecg changes
III.
Cardiac biomarkers

Atypical sx: fatigue, SOB, epigastric discomfort, n&v


Risk fx:
Prev hx of IHD, CVA
DM,HPT,HPL
Smoking, physical inactivity, obese
Family hx of heart dz, stroke
DDX chest pain + SOB:
MI
Aortic dissection (sudden severe pain, tearing
sensation, radiate to neck,back,abd,leg)
Pulm embolism (period of immobility?)
Pneumothorax (hx of trauma to chest?)
Gerd (burning sensation in chest, sour taste, obesity)
Esophageal rupture (chest pain after vomiting)
Pneumonia (pleuritic chest pain, fever, cough)
INVESTIGATION
Diagnostic ix:
1. ECG
NSTEMI: ST depression, T inversion
STEMI: ST elevation, T inversion, Q wave
2. Cardiac enzymes: trop T, Trop I, CKMB
NSTEMI:increase enzymes
UA: normal
3. Echocardiography
4. Angiography
Supportive ix:
FBC (low hb can precipitate MI)
LFT/BUSE/CREAT
Glucose lvl
Lipid profile
PT/APTT
CXR: cardiomegaly, p.effusion, sign of LV failure
ABG

MANAGEMENT
1.
2.
3.
4.
5.
6.
7.
8.

Assess ABC
Bed rest, v/s, continuous ECG monitoring
O2 by nasal prong/facemask
Analgesic: S/L GTN, IV morphine +antiemetic
Pharmacological rx:
Antithrombotic(antiplatelet, anticoagulants),
B-blockers, nitrates, acei/ARB, statin
Non pharmaco: stop smoking, exercise, diet, control
HPT, DM, HPL

Specific mx for STEMI: reperfusion therapy


thrombolytics ( iv streptokinase) @ PCI if C/I to thrombolytic

MONA
Morphine,O2,nitrates(isoket),aspirin

Together in Delivering Excellence (T.I.D.E)


Common/Possible Question in Exam + Answer
ECG interpretation !!
Advice before discharge:
-diet lifestyle, exercise, stop smoking, compliance to meds and f/up
Cx of MI:
Cardiac arrhythmia
Heart failure & cardiogenic shock
Thromboembolism (d/t bed rest,cardiac failure)
Cardiac rupture
Pericarditis
Post infarction angina
LV aneurysm
Complex regional pain syndrome (pain at l arm after weeks,months following mi)
Dresslers syndrome (pericarditis, fever, pericardial effusion)

Indicators of successful reperfusion:


Reduce chest pain
ST segment isoelectric
Restoration of hemodynamic/electrical stability
Contraindications for thrombolytics in STEMI:

Together in Delivering Excellence (T.I.D.E)


Congestive Cardiac Failure [Dalilah]
HISTORY
1.

Risk factors :
CAD, HPT, renal failure, valvular/congenital heart disease,
pericardial disease (TB, effusion, tamponade), arrhythmia
and anemia

2.

Symptoms of pulmonary edema :


Acute breathlessness, orthopnea, PND, dry cough/
coughing with sputum, frothy sputum

3.

Symptoms of right sided heart failure :


Lower limb edema, abdominal distension due to ascites

4.

Symptoms of cardiogenic shock :


Cool, diaphoretic skin, cyanosis, dyspnea, altered
sensorium, reduced urine output

5.

Assess further based on NYHA to get the class of heart


failure

PHYSICAL EXAMINATION
GENERAL
Respiratory distress
Decreased alertness ( cardiogenic shock)
Tachypneic
Tachycardia
Cold peripheries, delayed CRT
Hypotension
Sign of valvular heart lesion
Raised JVP
Pitting edema
SPECIFIC
Apex beat shifted to left side ( cardiomegaly )
Gallop rhythm ( pulmonary edema )
Loud P2 ( pulmonary edema)
Third and forth heart sound
Lung crepitation ( pulmonary edema)
Sign of pleural effusion
Ascites
Tender hepatomegaly

INVESTIGATION
1.

FBP,LFT, BUSE & creatinine, cardiac enzyme, ABG

2.

ECG Ischaemic changes

3.

Echocardiography cardiac chamber dimension, systolic n


diastolic function, valvular heart disease,
cardiomyopathies

4.

Regular BP monitoring or intra-arterial BP monitoring

5.

Urine output (renal perfusion), alertness and conscious


level (cerebral perfusion) and general wellbeing

6.

Assessment of venous pressure


CVP (only reflect right ventricular filling pressure)
Pulmonary capillary wedge pressure (PCWP) with
Swan-Ganz catheter ; useful in suspected ARDS,
exclusion of VSD, associated hypotension requiring
treatment with inotrope to guide therapy

MANAGEMENT
1.

Priorities
a) Sit pt upright
b) Oxygen (35 to 100%) via facemask to maintain PaO2
more than 60 mmHg and SPO2 more than 90%
c) Treat underlying arrthymia
d) IV canula large bore :
IV morphine (2.5-5 mg) + IV /IM 10 mg
metoclopromide
frusemide 40-80mg IV
e) Sublingual nitrate if systolic BP > 100 mmHg

2.

Oxygenation

3.

Fluid challenge ( Hartmanns solution )

4.

Diuretics

5.

Venodilators

6.

Inotropic agent

7.

Noradrenaline/adrenaline

Together in Delivering Excellence (T.I.D.E)


Details on Management
1. Oxygenation
- Increase inspired oxygen to keep SPO2 more than 90%
- Mechanical ventilation : if hypercapnia persist despite high flow oxygen (eg NIPPV)
- Correct severe metabolic acidosis (pH less than 7.2 ) as it has negative inotropic and proarrthymogenic effect
2.

Fluid challenge ( Hartmanns solution )


- If invasive hemodynamic monitoring is not available , fluid should be administered in small volumes (100ml ) over 510 min interval with reassessment of BP, heart rate, peripheral perfusion n breath sound. If BP does not responds to
fluid (after 500-1000 ml), start vasopressor
- If invasive hemodynamic monitoring is available, volume should be administered until a PCWP of 18 mmHg is
attained

3.

Diuretics
- IV frusemide 40 mg or bumetanide 1 mg at 20 min interval if initial therapeutic response is inadequate

4.

Venodilators
- Sublingual nitroglycerin 0.3-0.5 mg up to 3 tabs every 5 min interval
- IV nitroglycerin 5-10 microgram/min increased by 5-10 microgram/min every 5 -10 min
- IV isoket ( isosorbide dinitrite) 2-10 mg/hr

5.

Inotropic agent
- Dopamine 5-10 microgram/kg/min. low dose stimulates systemic vasodilation; high dose stimulates heart rate and
contractility
- Dobutamine 15-20 microgram/kg/min. acts at beta adrenergic receptor , no alpha adrenergic receptor activity

6.

Noradrenaline/adrenaline
- Beta and alpha adrenergic agonist. Increase heart contractility and peripheral vasoconstriction
- Noradrenaline : 8 -12 microgram/kg/min
- Adrenaline : 0.05 0.1 microgram /kg/min

QUESTIONS
1. Medications ( MOA, dose and side effect )
2.

Chest xray finding


Cardiogenic pulmonary edema; cardiomegaly, widened mediastinum, bat wings, upper lobe diversion, kerley A,B,C,
blunted costophrenic angle.

3.

PE finding for pulmonary edema

4.

Sign of right heat failure vs left heart failure

ECG findings ( ischaemic changes ST elevation, T inversion, Q wave ), duration and onset. Localization of infarction area. ( sbb
associated dengan CAD )

Together in Delivering Excellence (T.I.D.E)


Rheumatic Heart Disease [Ain]
HISTORY

PHYSICAL EXAMINATION

- RHD : A chronic heart condition caused by rheumatic fever that


can be prevented and controlled. Rheumatic fever is caused by a
preceding group A streptococcal (strep) infection.
- Multisystem disease affecting connective tissue particularly of the
heart, joints, brain, cutaneous and subcutaneous tissues
1.
2.
3.
4.
5.
6.
7.
8.
9.

Age ? - 5-15 yrs school-age children living in closed


community (high risk group)
Any history of fever or URTI preceeding to the complaint
Any joint pain or others ass. symptoms
eg.malaise,pallor,fatique ?
Which joint affected,nature of the pain,is it migratory or
localized?
Any skin lesions or rash noted in the body?
Any abnormal movement noted?
Any swelling anywhere or nodule especially over bony
prominence?
Assess risk factor overcrowding,poor sanitation,
poverty,poor housing.
Any complication symptoms eg.heart failure,atrial
fibrillation

#The knees, ankles, elbows, and wrists are the joints most likely to
become swollen from rheumatic fever. The pain often migrates from
one joint to another.

INVESTIGATION
1.
2.
3.
4.
5.

FBC anemia,leucocytosis
Inflammatory marker ESR/CRP positive
Throats swab for group A streptococcus
Anti-streptolysin O titre (ASOT) - elevated
Investigations for evidence of carditis
Chest x-ray cardiomegaly, pulmonary venous
congestion
ECG- First degree A-V block, T wave changes, low
voltage QRS
Echocardiogram cardiac dilatation, valve
involvement, pericardial effusion

Carditis in RHD :
Mitral valve (90 %) : MR children,adolescent
MS adult,later can get AF as cx.
Aortic valve : AR,AS
Less common affected : pulmonary, tricuspid
MANAGEMENT
Principle of management :
-

Step I - primary prevention (eradication of streptococci)


Step II - anti inflammatory treatment (aspirin,steroids)
Step III- supportive management & management of
complications
Step IV- secondary prevention (prevention of recurrent
attacks)
Step v tertiary prevention

1. Bed rest until CRP normal for 2 weeks (maybe by 3 months)


2. Benzylpenicillin 0.6-1.2g IM or Penicillin V 250-500 mg 2-3 times
daily for 10 days.
(if allergic give erythromycin or azithromycin for 10 days)
3. Analgesia for carditis/arthritis :aspirin 100 mg/kg/d in divided
dose (max 8g/d) for 2 day then 70 mg/kg/d for 6 weeks.
if moderate to severe carditis : add prednisolone
4. Immobilize joint in severe arthritis rest and supportive splinting
5. Treatment of chorea - Haloperidol (0.5 mg/8h) or diazepam
6. Secondary Prevention of Rheumatic Fever- aims to prevent illness
or progression of disease once a problem has been identified
Benzathine penicillin G 1 200 000 U every 3 weeks*
Intramuscular
Penicillin V 250 mg twice daily Oral
# For individuals allergic to penicillin : Erythromycin 250 mg twice
daily
7. Tertiary preventionaims to prevent complications once a disease
is established. Reducing symptoms to minimise disability and
prevent premature death. Eg.heart valve surgery, medication to
manage heart failure eg.diuretics and preventing stroke.

10

Together in Delivering Excellence (T.I.D.E)


1. Diagnosis of RHD
JONES criteria : evidence of recent strep infection plus 2 major criteria or 1 major criteria + 2 minor

# Exceptions to Jones Criteria


-

Chorea alone, if other causes have been excluded


Insidious or late-onset carditis with no other explanation
Patients with documented RHD or prior rheumatic fever,one major criterion,or of fever,arthralgia or high CRP suggests recurrence

2. Duration of Secondary Rheumatic Fever Prophylaxis

Fever without carditis - At least 5 y or until age 18 y


Rheumatic fever with carditis and heart disease (persistent valval lesion) - At least 10 y since last residual episode and at least until
age 40 y,sometimes lifelong prophylaxis
Rheumatic fever with carditis & heart disease (no valvar lesion) -10 y or well into adulthood
More severe valvular disease,post-valve surgery cases - lifelong

3. Differential diagnosis of acute rheumatic fever based on symptoms

11

Together in Delivering Excellence (T.I.D.E)


COPD [Rozana]
HISTORY

PHYSICAL EXAMINATION

Epidemiology :
> 35 yo, 10-20% in over 40s

General :
Tachypnoea, use of accessory muscle, wheeze, cyanosis

Chronic bronchitis : defined clinically as cough, sputum


production, on most days for 3 months of 2 sucessive years, sx
improvise if pt stop smoking

Specific :
sign of airflow limitation and air trapping in
advanced stage (barrel chest, loss of cardiac and
liver dullness, prolonged expiration, reduced breath
soundhyperinflation, reduce expansion)

Emphysema : define histologically as enlarged air spaces distal


to terminal bronchioles with destruction of alveolar walls

Cx : cor pulmonale: edema, JVP, pneumothorax


Risk factor :
Genes (alpha-1 antitrypsin enzyme deficiency causes
panlobular emphysema), Exposure to particles, Tobacco
smoke, Organic and inorganic occupational dusts, Indoor air
pollution from heating and cooking with biomass in poorly
ventilated dwellings, Outdoor air pollution, Lung growth and
development, Oxidative stress, Respiratory infections,
Socioeconomic status
Sx :

chronic cough, sputum, dyspnoea(interfere daily


activities), wheeze, chest tightness
extrapulmonary : LOW, cor pulmonale sx

Complication : acute infection +/- infection, polychythaemia,


respi failure, cor pulmonale, pneumothorax, lung carcinoma,
osteoporosis
INVESTIGATION
Laboratory : FBC =anemia of chronic disease, PCV (chronic
hypoxemia)
Others :
Spirometry : post bronchodilator FEV1/FVC ratio < 0.7
= not fully reversible airflow limitation

MANAGEMENT
Mx of acute COPD
Controlled oxygen therapy
Nebulized bronchodilators (salbutamol and
ipratropium)
Steroids (IV hydrocortisone and oral prednisolone)
Antibiotics, if evidence of infection
Physiotherapy to aid sputum expectoration
If no response repeat nebulizers and consider iv
aminophlline
If no respone
1) Consider nasal intermittent positive pressure
ventilation. 2) Consider intubation & ventilation
Mx of stable COPD
Non pharmaco/ general : smoking cessation, encourage
exercise, treat poor nutrition or obesity, influenza and
pneumococcal vaccination
Pharmaco
Mild (FEV1 50-80% predicted) : antimuscarinic
eg. Ipratropium/ B2 agonist inhaled PRN

Peak expiratory flow rate : low

ECG :detect pulmonary HPT ( advanced disease)


right atrial and ventricular hypertrophy (cor
pulmonale)
ABG : PaO2 +/- hypercapnia

Moderate (FEV1 300-49% predicted) : regular


anticholinergic eg. Ipratropium or long acting inhaled
B2 agonist eg. salmeterol + inhaled corticosteroid eg.
beclamethasone

Severe (FEV1 ) : LABA + inhaled steroid,


anticholinergic.

Imaging :
Hyperinflation (flattened diaphragm and increased
lung volume), large central pulmonary arteries,
peripheral vascular marking, bullae, hyperlucency of
lung
Exclude other diagnosis eg. Lung cancer, heart failure,
bronchiectasis and TB

Pulmonary HPT : Assess the need of LTOT(long term 02


therapy), treat edema with diuretics

12

Together in Delivering Excellence (T.I.D.E)

Pink puffers & blue bloaters (end of a spectrum)

Pink puffers have alveolar ventilation, a near normal PAO2 and normal or low PCO2, breathless but not cyanosed,
may progressed to type 1 respi failure
Blue bloaters have alveolar ventilation, with low PAO2 and high PACO2, cyanosed but not breathless and may go on
to dev. Cor pulmonale

13

Together in Delivering Excellence (T.I.D.E)


Tuberculosis [Fatin]
HISTORY
o

Epidemiology
Mycobacterium tuberculosis
Transmit through microscopic droplet ( cough,
sneeze, speaking)
Risk factor
Immunocompromise (DM, chronic dz, HIV, steroid,
malnutrition)
Travelling to endemic area
Substance abuse (drug/alcohol)
Contact with TB pt (occupation, family member)
Living in overcrowded area
Prev TB infection
S&S
Chronic cough >2w
Blood stained cough
LOW, LOA
Fever, night sweat
SOB, chest pain, pleuritic chest pain
Extra-pulmonary : hematuria (renal), back pain
(spine), seizure (meninges)
Complication (lymphatohematogenous spread)
Extra-pulmonary TB
-bone, brain, liver&kidney, heart
ARDS
Lung failure
Relapse of disease

PHYSICAL EXAMINATION
o

General
Cachexic
Fever
Muscle wasting

Specific
Lung :
Consolidation =chest expansion, dull
percussion, bronchial BS, crepitation
Pleura effusion = trachea deviated if massive,
chest expansion, stony dull, absent BS, vocal
resonance
Lung collapse = trachea deviation ipsilateral
mediasternal shift, chest expansion, dull,
absent/reduce BS
Other : lymphadenopathy

1) pneumonia
2) lung carcinoma
3) lung abscess
4) fungal infection

INVESTIGATION
o

Laboratory
FBC leucocytosis as sign of infection or anaemia due
to chronic disease
Sputum direct smear for AFB
Mantoux test- result read after 72H
Sputum c+ sensitivity 3 morning specimen
Sputum cytology- to look for any abnormal cells to
suggest malignancy
Blood culture + sensitivity :to detect any
microorganism
Broncoscopy - tumour, foreign body, inflammation
Pleura fluid analysis (pleura tapping)
Imaging
X-ray :
Primary TB: perihilar and paratracheal
lymphadenopathy, patchy area of consolidation,
pleura effusion feature
Post 1 TB: consolidation at post segment of
upper lobe @ sup segment of lower lobe,
tuberculoma at Rt upper lobe, cavitation
Milliary TB : millet seed nodule (1-3mm) evenly
distributed

MANAGEMENT
Anti-TB therapy
Intensive phase (2M) -> 2EHRZ
Maintainence phase (4M) -> 4HR
ST

1 LINE : Rifampicin (R)-hepatitis


Isoniazid (H)- hepatitis
Pyrazinamide (Z) joint and ms pain
Ethambutol (E)- visual disturbance
Streptomycin (S) ototoxicity
DOTS directly observed therapy short case

14

Together in Delivering Excellence (T.I.D.E)


Common/Possible Question in Exam + Answer
1)

Screening of high risk group


HIV pt
Immigrant
Person in prison/drug rehab centre
Pt with dm, renal dz, steroid, immune sup drug
Hemato malignancy

2)

Classification of TB
PTB +ve smear : * 2 sputum smear positive AFB
* 1 sputum +ve AFB and +ve radiological finding
* 1 sputum +ve AFB and +ve culture

3)

PTB ve smear : * 3 sputum smear ve


* sputum smear ve but subsequent culture +ve

TB meningitis Tx

The duration of anti tb is longer which for 12 months duration


Intensive(2 months) maintenance (10 months)
Other drug to give is steroid (6 weeks or longer)

4) Preventive measures
Primary intervention
Identification + immediate isolation
Herd immunity-BCG vaccination
Contact tracing of individual who are in close contact with cases
Reduce risk of transmission by using ppe(personal protective equipment), cough etiquette
5)

Why hemoptysis occur?


Due to erosion of vessel located in the wall of cavity or rupture of dilated vessel in cavity

6)

Mantoux test

7) Follow up : every 2 months, take CXR and sputum smear AFB


8) Intensive therapy 2 months, then maintenance therapy 4 months but can be extended when :
Cavitating lesion in CXR
Extrapulmonary TB
Immunocompromised pt

15

Together in Delivering Excellence (T.I.D.E)


Lung Cancer ( Bronchogenic Carcinoma) [Zuraidah]
HISTORY

Def : malignancy of the lung arising from the epithelium of the


bronchial tree.
nd

Prevalence : 2 after prostate ca in men , breast ca in women.


Aetiology : smoking, asbestos exposure, radon gas exposure, familial
predisposition (genetic), HIV infection, air pollution (pesticide), lung
diseases.
History :
0
1. Sx related to 1 tumor: cough, dyspnea, hemoptysis, chest pain,
postobstructive pneumonia
2.

Sx related to mediastinal spread:


- Hoarness of voice with left sided lesion (caused by recurrent
laryngeal nerve palsy
- Obstructive of svc with right sided tumor or asso
lymphadenopathy
- Elevation of hemidiaphragm from phrenic nerve palsy
- Dysphagia from esophageal obs and pericardial temponade

PHYSICAL EXAMINATION

General
- Hoarseness of voice, Cachexic, alopecia (chemo), nicotine
staining, cyanosis, clubbing, flaps, cervical l/n, raised JVP + sign of
SVC obstruction, leg edema
- Vital sign tachypnea
* Horner syndrome (ptosis), paraneoplastic synd ( wasting of small
ms of hand)
Specific (Respiratory system)
Inspection
Barrel shaped, mvmnt of chest reduce on affected, use of accessory
ms
Palpation
Trachea deviation, apex beat, chest wall tenderness (mets), inc tactile
fremitus
Percussion - dullness, liver span enlarged (mets)
Auscultate - bronchial bs, rhonchi, rub
DDx

3.

Sx related to mets: sites liver, brain, pleural cavity, bone,


adrenal glands, contralateral lung & skin

4.

Paraneoplastic synd:
- Pain in arm/legs caused by hypertrophic osteoarthropathy
- Sx of hypercalcemia caused by scc

5.

Systemic effects: fever, anorexia, low/loa, weakness, profound


fatigue
INVESTIGATION

Laboratory
FBC: WBC raised in concomitant infection
ABG: hypoxia with respi acidosis in severe endobronchial obs
ESR: > 100 in 1hour
Serum sodium, calcium
Sputum examination malignant cell cytology, c+s for any u/l
lung infection
Lung fx test: FEV1 of 1000ml after planned resection
Invasive: pleural fluid cytology, percutaneous transthoracic
needle biopsy

Imaging
- CXR:
0
1 tumor - hilar mass or coin lesion, rib erosions, raised
hemidiaphragm (phrenic nerve palsy), lymphangitis
carcinomatosis, any lung collapse
0
if 2 tumor cannon ball appearance
- CT scan TAP: metastasis, staging
- Bronchoscopy (+washing & brushing): endobronchial tumor
- Bone scan: staging

1.
2.
3.
4.
5.

Pulmonary TB
Pneumonia
Lung abscess
Bronchiectasis
Sarcoidosis

MANAGEMENT

Depends on multiplicity factors:


0
It is a 1 or mets lesion
Hilar or mediastinal infiltration
Chest wall involvement
Asso with complication- massive pleural effusion, svc
obstruction, collapse-consolidation
Phrenic nerve involvement
Paraneoplastic syndromes
Treatment :
1) Surgery
Lobectomy- the most effective type of surgery, even when the
lung tumor is very small.
A wedge- remove the tumor, surrounded by a margin of normal
lung.
Segmentectomy- removes the portion of the lung where the
cancer developed.
Pneumonectomy. If the tumor is close to the center of the chest,
remove the entire lung.
Radiofrequency ablation- needle inserted into the tumor to
destroy the cancer with an electrical current
* SCLC is not recommended for surgery d/t aggressive &
micromets. Go for chemo
2) Medical chemotherapy or radiotherapy
3) Treatment for sx such as infection & breasthless
4) Pain management & quality of life
5) Adequate hydration & food intake

16

Together in Delivering Excellence (T.I.D.E)


Common/Possible Question in Exam + Answer

1. Cx of lung ca?
i.
Hemoptysis
ii.
Acute breathlessness d/t endobronchial narrowing
iii.
Massive, recurrent hemorrhagic pleural effusin
iv.
SVC obstruction
v.
Paraneoplastic syndrome

3. Contraindication for surgery?


i.
Metastatic carcinoma
ii.
FEV1 < 15000ml
iii.
Severe pulmonary hpt
iv.
Uncontrolled major cardiac arrhythmias
v.
Co2 retention
vi.
Myocardial infarction in the past 3 months

2. Aim of staging?
To identify candidates for surgical resection, since this
approach offers highest potential cure

4. Which tumor respond well to chemo?


Small cell lung ca (SCLC), combination of cisplatin & etoposide
is the best therapeutic index of ay regime
Role for chemo in non small cell ca (NSCLC) suggested that
bnefits are small

Notes
Types :
Small cell lung ca (SCLC) 20%, rapid growing, strong correlation with smoking, mets rapidly to various organ (liver, brain, bone,
git, adrenal glands ), histologically- keratinization
Non small cell lung ca (NSCLC) 80%
I.
Adenocarcinoma (50%), commonly seen in non smoker, arises from bronchial mucosal glands in the outer, or peripheral
area of lungs, histo-gland formation
II.
Squamous cell carcinomas (30%), aka epidermoid carcinomas, centrally located, cavitary lesion, histo- presence of
keratin pearls and has tendency to exfoliate.
III.
Large cell carcinomas (20%), undifferentiated ca, large peripheral mass on cxr, histo-highly atypical cell with focal
necrosis
Anatomical Staging -

17

Together in Delivering Excellence (T.I.D.E)


Bronchiectasis [Rafidah]
HISTORY
Definition: Abnormal and permanent dilated airways resulting
from Inflamed thickened and irreversibly damage bronchial
walls cause mucociliary transport mechanism become
impaired and frequent bacterial infection ensues.

Aetiology:

Congenital: cystic fibrosis,


Primary ciliary dyskinesia,
Kartageners syndrome
Post-infection: measles, pertussis,
Bronchiolitis, pneumonia, HIV, TB
Bronchial obstruction: tumour,
foreign body
Rheumatoid arthritis, IBD
Allergic bronchopulmonary aspergillosis

Cough with copious purulent sputum


Recurrent hemoptysis
SOB
Intermittent fever and night sweat
History of recurrent infection
Weight loss

PHYSICAL EXAMINATION
General:
Respiratory distress
Finger clubbing
Specific:
Coarse crackles over affected area, usually basal lung
Sign of collapse, fibrosis or pneumonia

S&S

Complication

Pneumonia
Pleural effusion
Pneumothorax
Hemoptysis
Cerebral abscess
amyloidosis
INVESTIGATION

Laboratory
Full blood count white cell count (infection)
Sputum culture
Imaging
Chest radiograph: cystic shadow, thickened bronchial
walls(tramline and ring shadow)
High resolution CT scan thickened, dilated bronchi
and cyst at the end bronchioles.
Bronchoscopy to locate site of hemoptysis or
exclude obstruction

MANAGEMENT
Non surgical:
Non pharmacological
1) Postural drainage at least 3 times daily for 10
20min
Pharmacological
1) Antibiotic: according bacterial sensitivities
2)

Bronchodilator: may be useful in asthma, copd,cf,


allergic bronchopulmonary aspergillosis

3)

Corticosteroid: eg. prednisolone

Surgical : resection of the affected lobe

18

Together in Delivering Excellence (T.I.D.E)


Common/Possible Question in Exam + Answer
1)

What are the major respiratory pathogens in bronchiectasis?


Staph. Aureus, pseudomonas aeruginosa, H. influenza, and anerobes

2)

What are the common site for localized disease?


Left lower lobe and lingula

3)

What is the indication of surgery in bronchiectasis?


Bronchiectasis localized to a single lobe or a segment without clinical, bronchographic, ct evidence of bronchiectasis of
bronchitis affecting other parts of the lungs.

19

Together in Delivering Excellence (T.I.D.E)


Pleural Effusion [Farhan]
HISTORY

PHYSICAL EXAMINATION

Accumulation of fluids in the pleural space


Transudates (<25g/L):
Due to increase venous pressure (CCF, constrictive
pericarditis, fluid overload)
Hyponatremia (cirrhosis, nephrotic syndrome,
malabsorption)
Hypothyroidism (right pleural effusion)
Meigs syndrome (benign ovarian tumor, ascites, pleural
effusion)

General examination (look for signs of associated disease):


Malignancy (cachexia, clubbing, lymphadenopathy,
mastectomy scar)
Stigmata of CLD (Dupuytren contracture, spider naevi)
Cardiac failure (displaced apex beat, BP, cyanosis,
tachycardia)
Hypothyroidism (dry skin, thin hair, cold hands, round
puffy face)
SLE (malar rash, discoid rash, oral ulcers, alopecia)
RA (symmetrical joint swelling)

Exudates (>35g/L):
Increased leakiness of pleural capillaries due to
Infection (pneumonia, tuberculosis)
Ischemia (pulmonary infarction, SLE, rheumatoid arthritis)
Malignancy (bronchogenic ca, malignant mets, lymphoma)

Chest examination (only apparent if effusion > 300ml):


Inspection: asymmetrical chest movement
Palpation: reduced chest expansion, mediastinal shift,
trachea deviation (>1000ml), decreased tactile fremitus,
Percussion: stony dullness
Auscultation: Reduced or absent breath sound, reduced
vocal resonance

Common complaints: dyspnea, cough, pleuritic chest pain.


Ask about associated symptoms e.g. dyspnea with bilateral
leg swelling, orthopnea, and PND CCF; or night sweats,
fever, and weight loss TB
Ask about occupation; might give a clue to illness

INVESTIGATION
CXR (PA):
Blunted costophrenic angle (small effusion)
Clear air fluid level with concave upper border
Air fluid level with flat upper border (presence of
pneumothorax)
Lateral decubitus film is useful to detect smaller effusion;
layering of an effusion indicates free flowing effusion
Pleural fluid analysis (send for):
Clinical chemistry (protein, glucose, pH, LDH, amylase)
Bacteriology (microscopy & culture, TB culture)
Cytology
Immunology (RF, ANA, complement) if indicated
Pleural biopsy
If pleural fluid analysis is inconclusive

MANAGEMENT
Transudative effusions are managed by treating underlying
causes
If effusion is symptomatic (exudative/transudative) drainage
can be done to provide relief
Drain fluid slowly (max 2L/24h)
If drain large amount quickly; it can cause re-expansion
pulmonary edema
Pleurodesis (pleural sclerosis)
Talc, tetracycline, bleomycin sulfate, zinc sulfate
Thoracoscopic talc pleurodesis most effective for
malignant effusions
S/E: fever, chest pain, nausea
Surgery
Persistent collections and increasing pleural thickness
(on ultrasound) requires surgery pleurectomy

20

Together in Delivering Excellence (T.I.D.E)

Lights criteria
To differentiate between transudate and exudate for proteins ranged in between 25 35g/L
According to Lights criteria, the fluid is exudate if:
Effusion protein : serum protein ratio > 0.5
Effusion LDH : serum LDH ratio >0.6
Effusion LDH level is greater than 2/3 of the upper limit of serum LDH

Pleural fluid analysis


Normal pleural fluid characteristics
Clear ultrafiltrate of plasma that originates from the parietal pleura
A pH of 7.60-7.64
Protein content of less than 2% (1-2 g/dL)
Fewer than 1000 white blood cells (WBCs) per cubic millimeter
Glucose content similar to that of plasma
Lactate dehydrogenase (LDH) less than 50% of plasma
Sample
Clear, straw-coloured
Turbid, yellow
Haemorrhagic
Clinical chemistry
Glucose <3.3mmol/L
pH <7.2
LDH (pleural:serum >0.6)
Amylase

Causes
Transudate, exudate
Empyema, parapneumonic effusion
Trauma, malignancy, pulmonary infarction

Empyema, malignancy, TB, RA, SLE


Pancreatitis, carcinoma, bacterial pneumonia, oesophageal
rupture

Chest drain
Safe triangle for chest drain insertion:
1. Lateral border of pectoralis major
2. Anterior border of latissimus dorsi
3. Horizontal line at nipple level

Indication:
Pneumothorax
Malignant pleural effusion, complicated parapneumonic effusion, empyema
Pleural effusion compromising ventilation
Traumatic haemopneumothorax
Complication:
Thoracic or abdominal organ injury
Lymphatic drainage chylothorax
Damage to long thoracic nerve of Bell winging scapula
Arrhythmia (rare)

21

Together in Delivering Excellence (T.I.D.E)


Chronic Kidney Disease (Kidney Damage >3 Months) [Khiru]
HISTORY
In history:
i)
Past UTI
ii) known BP, DM, FHx
iii) Drug hx (NSAIDS, gentamycin, sulphonamides, tetracyclines,
vancomycin,amphotericin, cisplatin, ACEi, ARB, methotrexate, heavy
metal poisoning)
iv) Fatigue, weakness
v) Anorexia, vomiting, metallic taste
vi) Pruritus
vii) Bone pain
viii) Dyspnea
ix) Ankle swelling
Causes

PHYSICAL EXAMINATION
Signs: pallor, yellow skin pigmentation, brown nails, purpura, bruising,
excoriation, increase BP,cardiomegaly, pericardial rub, peural effusion,
pulmonary or peripheral edema, proximal myopathy

Acute kidney injury


DM
HPT
Glomerulonephritis/ pyelonephritis
Polycystic kidney disease
Renal vascular disease
Analgesic nephropathy (antipyretics, caffeine, NSAIDs)
Med

Complication

Electrolyte: hyperk,hypoCa,hyperphosphatemia

Haematological: anemia, bleeding tendency (plt dysfunction)

CVS: cardiac failure, HPT, pericarditis, accelerated atherosclerosis

Neurological: drowsy, seizure, peripheral neuropathy

Metabolic/ endocrine: hyperlipid, renal osteodystrophy

GIT: anorexia, N&V, bleeding

Skin: pruritus, easy bruising

INVESTIGATION
1. Blood: Hb reduced (normochromic normocystic), ESR, Urea & electrolyte
(increase urea & creatinine), glucose (DM), reduced calcium, increase
phosphate, increase alkaline phosphate (renal osteodystrophy), increase
PTH
2. Urine: microscopic culture & sensitivity, dipstick, 24H urinary protein

3. Imaging: renal ultrasound-renal size small, <9cm, but normal or large with
CRF in DM, PKD, amyloidosis, myeloma, systemic sclerosis, asymmetric
renal vascular disease

MANAGEMENT
1.Refer nephrologist treat reversible causes: relieve obstruction, stop
nephrotoxic drugs, deal hypercalcemia & cardiovascular risk.
2.Lifestyle advice. Should exercise, healthy weight & stop smoking. Sodium
restriction, moderate protein diet. Potassium restriction only if
hyperkalemia; bicarbonate supplements to correct acidosis.

3.HPT. Target BP <140/85 (<130/80 if diabetic or >1g proteinuria/d). In


diabetic kidney disease, even with normal BP, treat with ACEi or ARA.
4.CVS: statins & aspirin

4. CXR-cardiomeegaly, pleural effusion, pulmonary edema


5. Bone xray- renal osteodystrophy
6. Renal biopsy- consider if cause is unclear & normal size kidneys

5.Renal bone disease (osteodystrophy): treat if increase hyperparathyroidism.


Phosphate rises in CRF, which increase PTH further, & also precipitates in
kidney & vasculature. Restrict dietary phosphate. Give binders (calcichew)
to bind phosphate in gut to reduce absorption. Vit D (alfacalcidol) & Ca2+
supplements reduce bone disease & hyperparathyroidism
6.Edema. High dose loop diuretics (frusemide) restrict on fluid & sodium
intake

7.Restless legs. Clonazepam or gabapentin


8.Prepare for dialysis/ transplantation

22

Together in Delivering Excellence (T.I.D.E)


Common/Possible Question in Exam + Answer
1.

Definitions

Azotemia: accumulation of nitrogenous product (chiefly urea) in blood as indicated by raised serum urea & creatinine

Uremia: manifestations of organ dysfunction a/w azotemia

ARF: significant deterioration in renal function occurring over hours or days, clinically manifestation as an abrupt &
sustained rise in serum urea & creatinine.

CRF: permanent reduction in GFR (5-25ml/min) sufficient to produce detectable alteration in well-being & organ function.
(>3 months)

ESRF: final stage of CRF (GFR<5ml/min) when pt cnt survive w/o transplantation or long term dialysis

Normal GFR:90-120ml/min

2.

Classification of CKD

3.

Indications for dialysis

Acid base imbalance (severe metabolic acidosis pH<7.2 or base excess <10)

Electrolyte imbalance (Resistant hyperK >7mmol/L)

Intoxication (ingestants/toxins-lithium)

Overload (Fluid overload not responsive to diuretics, refractory pulmonary edema, volume overload causing respi distress)

Uremic symptoms (uremic encephalopathy, uremic pericarditis)

4.

Indication for renal biopsy

Unexplained acute renal failure or chronic renal insufficiency

Acute nephritic syndrome

Unexplained proteinuria & hematuria

Previously identified & treated lesions to plan for future therapy

Systemic diseases a/w kidney dysfunction e.g SLE, Foodpasture syndrome, wegeners granulomatosis t0 confirm extent of
renal involvement & to guide management

Suspected transplant rejection, to differentiate it from other causes of acute renal failure

To guide treatment

5.

Renal Osteodystrophy

Skeletal complications of CRF characterized by:


i.
osteomalacia, osteitis fibrosa (caused by hypocalcemia, hyperphosphatemia & reduced synthesis of 1,25-DHCC)
ii.
osteoporosis
iii.
osteosclerosis (increased density at upper & lower ends of vertebra/ jugger jersey spine)

6.

Causes of Anemia in CRF

Reduced renal erythropoietin production

Hemolysis & depression of erythropoiesis d/t uremia

GIT bleeding

Marrow fibrosis (osteitis fibrosa of secondary hyperparathyroidism)

23

Together in Delivering Excellence (T.I.D.E)


Chronic Liver Disease [Sofia]
PHYSICAL EXAMINATION

HISTORY
A.

Hand:
Leukonychia, clubbing, palmar erythema, bruising, asterixis,
scratch mark

B.

Face:
Jaundice, fetor hepaticus

C.

Chest:
Gynecomastia, loss of axillary hair, spider naevi

3. Changes in color of urine and stools

D.

Abdomen:
Hepatosplenomegay, ascites, testicular atrophy

4. Social hx: sexual activity and alcohol history

E.

PR exmntn:
Stool colour

1. Duration of jaundice
2. Ass symptoms :
Dyspepsia , fat intolerance or biliary colic, arthralgia,
myalgias, rash, anorexia, abdominal pain, fever,
pruritus

5. Complication: bleeding tendency, easily bruising


6. Constitutional hx : LOW or LOA

INVESTIGATION
1.

FBC- low Hb (hemolysis), increased retic count, serum


bilirubin, LDH & urobiliogen, decreased plasma

7. Risk factors:
Recent travel history, Exposure of patients with
jaundice, Parenteral exposures-transfusions, IV
abuse, tattoos
8. Occupational history-contact with rats.
9. Exposure to contaminated foods or water. Recent
eat shellfish (HAV), any water sport activity, source
of water
10. Drug hx: Use or exposure to medication-OTC,
physician prescribed, Complementary or alternative
medicine-herbal or vitamin preparations or steroids.
11. Family history- hemolytic anemias, congenital
hyperbilurbinemias and hepatitis.

haptoglobulin (hemolysis)
2.

FBP- bite cells, blister cells , Heinz bodies

3.

Clotting factor- deranged in liver dz

4.

Liver function tests.

5.

Viral serology for HAV, HBV, HCV

6.

Tumor marker- alpha fetoprotein

7.

Liver biopsy.- in case of hepatocellular carcinoma

8.

ERCP-in case of post hepatic jaundice

9.

Ultrasound in case of post hepatic jaundice

MANAGEMENT
For Viral Hepatitis B:
1.
2.

*basically a lot can be asked, need to rule out other


causes
3.

4.

For acute hepatitis mainly supportive mx & monitor HBV


markers
For chronic
mostly observed unless had high viral load
antiviral: interferon, entecavir, tenofovir. Lamivudine.
Response is judged by HBV DNA level, or if HBeAg is
present by seroconversion to anti-HBe
Pt w chronic hep should had routine lifelong f/up. Fr every
6 month or 1 year check for LFT, alpha-fetoprotein, u/s of
liver
High risk group shud be screen for HepB and given
vaccination

24

Together in Delivering Excellence (T.I.D.E)


Causes of jaundice:
Prehepatic:
Malaria
Hemolytic anemia: sickle cell anaemia, G6PD deficiency,
Hereditary spherocytosis
Intrahepatic:
The viral hepatitis group of infections
Alcoholic liver dz
Leptospirosis
Paracetamol overdose
Primary biliary cirrhosis
Gilbert's syndrome
Liver cancer
Autoimmune hepatitis
Primary sclerosing cholangitis

CAUSES OF HEPATOSPLENOMEGALY
Infection:
Acute viral hepatitis
Infectious mononucleosis
Cytomegalovirus
Rubella
Malaria
Schistosomiasis or filariasis
Haematological disease:
Myeloproliferative disease
Leukaemia
Lymphoma
Pernicious anaemia
Sickle cell anaemia

Posthepatic:
Gallstones
Pancreatic Cancer
Gallbladder Cancer Or Bile Duct Cancer
Pancreatitis

Thalassaemia
Myelofibrosis

Metabolic disease:
Niemann-Pick disease
Gaucher's disease
Hurler's syndrome
Chronic liver disease and portal hypertension
Amyloidosis
Acromegaly
Systemic lupus erythematosus
Sarcoidosis

Child Pugh score (ABCDE)


Parameters
Albumin (g/L)
Bilirubin (micromol/L)
Coagulation (Prothrombin time)
Distention (ascites)
Encephalopathy

1 point
<34
>35
1-3
None
none

2 points
34-51
28-35
4-6
Slight
1-2

3 points
>51
<28
>6
Moderate
3-4

Grade A: 5-6
Grade B: 7-9
Grade C: >10
*risk of variceal bleeding is higher if score >8
*can also predict mortality:
1 year survival
5 years survival
Grade A
84 %
44%
Grade C
42%
21%

Hepatic encephalopathy
As liver fails, nitrogenous waste (eg: ammonia) builds up in circulation and passes to the brain, where astrocytes clear it (by
process involving the conversion of glutamate to glutamine).
This excess glutamine causes osmotic imbalances and shift fluid into the cell cerebral edema.
Grading:
I.
Altered mood/behavior, sleep disturbance (eg: reversed sleep pattern) , dyspraxia (pls copy this 5 pointed star), poor
arithmetic, no liver flap
II.
Increased drowsiness, confusion, slurred speech +/- liver flap, inappropriate behavior/personality change (ask family)
III.
Incoherent, restless, liver flap, stupor but not coma
IV.
Coma

25

Together in Delivering Excellence (T.I.D.E)


Approach To Anemia [Baisyatul]
HISTORY
1.

2.

3.

4.

5.
6.

Presenting symptoms:
Weakness, lethargy, shortness of breath, fatigue,
postural dizziness
Further history:
History of GIT bleed ( hematemesis, PR bleed)
Heavy menstrual blood loss
Bleeding after tooth extraction
Bruising, Heamathrosis
Bone pain,Recurrent infection
Jaundice
Lymph gland swelling (lymphoma)
Past medical/surgical history
History of gastric surgery/malabsorption
Underlying RA, Underlying OA (required NSAIDs),
chronic kideney disease
History of previous blood transfusion,
chemotherapy
Social history
Strict vegetarian diets (B12 def)
Exposure to toxins(benzene) risk of leukemia
Alcoholic
Family history- thalassemia, sickle cell dzs, colon ca,
haemophilia, G6PD
Drug history-NSAIDs, anticoagulant

PHYSICAL EXAMINATION
General
Specific
Abdomen
-

2.

Peripheral blood film: target cells, tear drop RBCs,


poikilocytosis, blast cells, hypersegmented
polymorph, pencil cells

3.

Serum iron, serum ferritin, TIBC

4.

Coagulation study: PT, aPTT, INR

5.

Factor assay for factor deficiency

6.

Hb electrophoresis for thalassemia

Surgical scar (gastrectomy, stoma)


Hepatosplenomegaly

Lymph node
Lymphadenopathy at neck and inguinal
Complete examination
PR examination
Stool occult blood
Hematuria

INVESTIGATION
Blood investigations:
1. FBC :
Hb (low)
Total white cell count and differential count
leukocytosis (basohilia): CML
leukocytosis (eosinophilia): lymphoma
lymphocytosis: lymphoma, CLL
Platelet count
Mean cell volume (MCV)
low MCV-microcytic
normal MCV-normochromic
high MCV-macrocytic
Reticulocyte count (reticulocytosis)

Pallor
Thalassemic facies
Cachexic (malignancy)
Pallor of palmar, koilonychia
Bruising, scratchmark (pruritus)
Conjunctiva pallor, jaundice
Angular stomatitis, gum hypertrophy, glossitis

MANAGEMENT
1.

Blood transfusion
For actively bleed patient
Severe and symptomatic anemia
Hb<8

2.

Ferrous sulphate therapy


IDA

3.

Nutritional therapy and dietary folate


B12 and folate deficiency

4.

Splenectomy
Hypersplenism

5.

Bone marrow transplant

6.

Chemotherapy : leukemia

26

Together in Delivering Excellence (T.I.D.E)


COMMON QUESTIONS
1.

Anemia classification
Morphology
Hypochromic microcytic
Thalassemia
IDA
Sideroblastic anemia
Causes
Blood loss
-Trauma
-Colon ca
-Bleeding oesophageal varices

2.

Lab result
Iron deficiency anemia
Ferritin: low
Iron: low
Microcytic hypochromic
TIBC: increased

Reference:
1.
2.
3.
4.

Xpress revision short case UM


Oxford handbook
Student notes medical
Medscape

Normochromic normocytic
Haemolytic anemia
Bone marrow failure
Anemia of chronic dzs

Macrocytic
B12 deficiency
Folate deficiency
Alcoholism

Inadequate production
-B12 and folate deficiency
-Thalassemia, IDA
-Leukemia, aplastic anemia
-Renal failure

Excessive destruction
-G6PD
-Haemolytic spherocytosis
-AIHA,malaria,sepsis

Thalassemia
Ferritin: normal
Iron: normal
Microcytic hypochromic
TIBC: normal

Anemia of chronic disease


Ferritin: normal
Iron: low
Normochromic normocytic
TIBC: normal

27

Together in Delivering Excellence (T.I.D.E)


Systemic Lupus Erythematosus [Aiman]
HISTORY
Age: 20-40 years old
Gender: Female predominat (10x)
+ve Family hx

Diagnostic features:
(at least 4 features, or 2 features in separate occasion)
1.
2.
3.
4.
5.
6.
7.
8.

Malar rash (spared nasolabial fold)


Discoid rash
Photosensitivity
Oral ulcer (initially painless unless infected)
Arthritis (2 or more, involve peripheral joint)
Serositis (pleuritic or pericarditis chest pain)
Renal disorder (persistant proteinuria frotty urine)
Neurological disorder (seizure or psychosis with absent of
offending drugs or metabolic derangement)
9. Haematological disorder (anemia, leucopenia,
thrombocytopenia)
10. Immunological (anti dsDNA antibodies, antibody to SM
antigen, antiphospholipid antibodies)
11. Antinuclear antibody (ANA)

PHYSICAL EXAMINATION
General Examination
Skin : pallor (anemia), petechae (thrombocytopenia),
discoid rash, subacute cutaneous erythematosus
Face : butterfly rash, oral ulcer, pale conjunctiva
(anemia)
Pitting oedema
generalised oedema
high BP
Others
Small joint arthritis (rarely with swelling)
chorea

Differential Diagnosis :
Depends on the presentation

Complication:

HPT due to renal disorder


Thromboembolism and recurrent miscarriage (in +ve of
antiphospholipid antibodies)
Bleeding tendency (easy bruises)
Anemic symptom (SOB, lethargy)
HOW and when diagnose + on what medication, how many
flare episode
INVESTIGATION

Bedside:
-urine dipstic : proteinuria
Lab:
- FBC : anemia, leucopenia, thrombocytopenia pancytopenia
(hemolytic anemia normocytic normochromic)
ESR raised (CRP normal unless there is serositis/arthritis/infection
present)
- BUSE : renal function (urea creatinine), electrolyte imbalance
- Urinalysis : RBC, proteinuria, cast on the urine microscopy)
Specific:
- Identification of autoantibody
1. Antinuclear antibody (ANA)
if negative unlikely to be SLE
unless Extractable nuclear antigen is positive (exp: Ro
antigen, Sm antigen, La aantigen)
2. Anti dsDNA antibodies (+ve in 20-30%)
3. Antiphphospholipid antibody
- C3 C4 level : low in active disease (during flare)
- Renal biopsy : in Lupus nephritis
- Skin biopsy : deposition of IgG and complement

Discoid skin lesions


Erythematous macules
Leukemia
Nephrotic syndrome

MANAGEMENT
1. Avoid the flare
Sunlight exposure (UV light) by using sunblock, clothing
2. Medical therapy
Depends on severity:
Mild : require analgesic and NSAIDs
+troublesome cutaneous and joint symptom : Hydrochloroquine (200400mg/day)
Mild-moderate disease : short course of oral corticosteroid
(rash, synovitis, serositis)
Acute/life-threatening disease : high dose corticosteroid (oral
prednisolone 40-60mg/day) or (methylprednisolone 500mg-1g/day)
+ pulse IV cyclophophomide
Then change to other immunosuppressive drug (azathioprine,
methotrexate, ciclosporin) for the step down after cyclophosphamide
or used with corticosteroid
(in renal and cerebral involvement)
+antiphospholipid antibody syndrome (thrombosis) : require lifelong
warfarin
(in recur thrombosis despite on warfarin => target INR 2.5-3.5)
Diuretic
Anti HPT

28

Together in Delivering Excellence (T.I.D.E)


1.

Definition : chronic, remitting and relapsing multisystem autoimmune disease in which organ and cell undergo damage
mediated by tissue-binding autoantibodies and immune complex

2.

Exacerbation or flares usually follows: (all cause apoptosis of cell)


o Sun exposure
o Viral or bacterial infection (esp: epstein-bar virus)
o Sex hormone status(on hormonal therapy, pregnancy-however there is fertility problem in sle patient and no
contraindication for pregnancy unless in severe cases)
o Oxidative stress
o Drugs [hydralazine(anti-HPT), isoniazid(anti-TB), procainamide(anti-arrhythmia)

3.

Immunosuppressive drugs
a. Cyclophosphamide(to achieve remission)
MOA:cross linking with cell DNA-not specific/SE:myelosuppressive, infertility in male who receive high dose as children
b. Ciclosporin
MOA:interfere with activity and growth of T celss/SE: nephrotoxic, neurotoxic
c. Methotrexate
MOA:inhibit Dihydrofolic acid reductase(antifolate), interfere with DNA synthesis specific for S phase/SE: cranial nerve
palsy, hepatotoxicity
d. Azathioprine(to maintain remission)
MOA:purine analogues(affect more to proliferative cells such as T cells and B celss/SE:bone marrow suppression

4.

Exacerbation, complete remission and chronic persistant condition. Early death associated with renal, cerebral or infection.
Later age stroke and coronary artery disease become more prominent. Deformity due to joint destruction is rare compare
with OA or RA. Increased long term risk to developed lymphoma.

5.

Prolong steroid used side effect. Refer corticosteroid drugs note.

29

Together in Delivering Excellence (T.I.D.E)

PAEDIATRIC

Notes

30

Together in Delivering Excellence (T.I.D.E)


Acute Exacerbation of Bronchial Asthma (Alliah)
HISTORY
Definition
Chronic airway inflammation leading to increase airway
responsiveness leads to recurrent episodes of wheezing,SOB,chest
tightness & coughing particularly at night and early morning.
Important history
Current sxpattern of sxpresent tx (response & compliancy)
Previous hosp.admission
Typical exacerbation
Precipitating factor
Infection (URTI)
Non-compliance to drug
Home/school environment
(allergen:cat,bedsheet,smoking,insects)
Predisposing fctr
Hx of atopy (eczema, allergic rhinitis, conjunctivitis)
Aggravating fctr
Rhinosinusitis
GERD
Sensitive to NSAIDs (esp.aspirin)
Family hx of allergy and asthma
Personal impact to lifestyle
School performance : attendance,academic,co-curiculum
Impact to family
Financial?parents quarrel?
Who take care pt/who send pt to hosp during attack?

PHYSICAL EXAMINATION

Tachypnea
Tachycardia
Cough
Drowsy
Wheezing, rhonchi
Prolonged expiratory phase

As attack progress

Cyanosis

Tight chest

Sternal retraction

Agitation

Inability to speak

Tripod sitting position

Diaphoresis

Pulsus paradosus
Signs of chronic illness

Barrel chest

Generalized polyphonic expiratory wheeze

Prolonged expiratory phase

Harrisons sulci

Crepitation (d/t segmental atelectasis)

Reduce liver and cardiac dullness

Eczema/dry skin

Hypertrophied turbinates
MANAGEMENT

Complication
Status asthmaticus
Respiratory failure
Pneumothorax
Lung collapse
INVESTIGATION

a) Lung function test : PEFR > 5y/o or Spirometry


b) Brochodilator reversibility test (improve 10-15% post
brochodilator)
c) Skin prick test (to aid dx of atopy)
d) CXR - particularly to exclude foreign body or chronic chest
infection or to exclude complications in severe acute
episode

Principle
To allow child to lead as normal life as possible by :

Controlling sx

Prevent exacerbation

Optimize pulmonary function

Minimize treatment and side effects


Drug
Reliever (Short-acting B-agonist )
Controller/preventer : Corticosoteroids, LABA,Combination (Inhaled
c/steroid+LABA) , anti-leukotrienes
Acute

Close monitor pulse,color,PEFR,ABG & SpO2 for at least 4H


High flow oxygen via face mask
Nebulised bronchodilator (salbutamol)
Prednisolone 1-2mg/kg body wt orally (max 40mg)
If life-threatening features present :
- give IV aminophylline 5mg/kg over 20min followed by
maintenance infusion then IV hydrocortisone 100mg 6hrly
- add Ipratropium bromide 0.25mg to nebulised B-agonist
Reassess patient with pulse oximetryresponse to tx

31

Together in Delivering Excellence (T.I.D.E)


Common/Possible Question in Exam + Answer
Assessment of severity
Intermittent

Persistent

Mild

Moderate

Severe

Monitoring (each f/up)


1)

Assessment of asthma control


Interval sx
Frequency & severity of acute exacbtn
Morbidity 2 asthma
Quality of life
PEFR/FEV1 monitoring

2)

Compliance to asthma therapy


Frequency
Technique

3)

Asthma education
Understanding asthma in childhood
Reemphasize compliance to therapy
Written asthma action plan

Daytime sx
Nocturnal sx
Exercise induced sx
Exacerbation affect
sleep & activity
PEFR/FEV1

< 1x a week
< 1x a month
NO
NO
Normal lung fx
> 1x a wk
> 2x a mth
YES
1x a month
>80%
Daily
> 1x a wk
YES
> 2x a mth
60-80%
Daily
Daily
Daily
> 2x a mth
<60%

32

Together in Delivering Excellence (T.I.D.E)


Rheumatic Fever [Sofia]
HISTORY
1. History of preceded sore throat
2. Risk factor: low socioeconomic status, overcrowding, poor housing, genetic
susceptibility
3. Carditis Pericarditis: audible friction rub
4. Arthritis: migratory polyarthritis, large joints usully affected (shoulder,
elbow, knee, ankle), a/w fever
5. Syndenham chorea (SC): are involuntary movements, hypotonia, and mild
muscular weakness. Chorea can be generalized or unilateral, predominantly
involving the face, hands, and arms. Movements are present at rest,
aggravated by stress, and usually cease during sleep. interfere with
volitional movements and result in a clumsy gait, dropping and spilling, and
explosive bursts of dysarthric speech.
6. Erythema marginatum: bright pink macule/ papule that spread outward in
circular serpiginous pattern, multiple, appear at trunk / proximal
extremities, rarely distal, never on face, non pruritic, painless, blanch on
pressure, brought out by warmth
7. Subcutaneous nodules: non pruritic,painless round, firm, freely movable,
elbows, wrist, knee, ankle, Achilles tendon, on extensor surface

PHYSICAL EXAMINATION
General:
Fever, rashes, nodules, involuntary jerky movement, protruding of tongue
revealed bag of worm
CVS:
Murmur, pericardial rub, cardiomegaly, sign of cardiac failure
Examination of joints involved: red, swollen, warm, synovial aspiration
revealed leukocytosis
To test for syndenham chorea:
pronator sign when raising hand above head, if + pt may pronate 1 or
both hand,spoon configuration occur when hand are extended, milking
grip elicit by putting ur finger at pt grip hand
DDX
-

JRA
Leukemia
SLE
INVESTIGATION

1)
2)
3)
4)
5)

FBC- leukocytosis, ESR/CRP raised, throat swab


Anti-streptolysin O titre- >240 Todd U
Anti-DNAase B
Imaging : ECG, CXR, Echo- to look for carditis
Histo: Aschoff bodies in heart (granulomatous structures consisting of
fibrinoid change, lymphocytic infiltration, occasional plasma cells, and
abnormal macrophages surrounding necrotic centres)

Anitschkow cells are enlarged macrophages found within granulomas (called


Aschoff bodies) associated with the disease. Ovoid nucleus and chromatin
that is condensed toward the center of the nucleus in a wavy rod-like pattern

MANAGEMENT
Aim: to suppress inflammatory response to minimize cardiac
damage, provide symptomatic relief, & eradicate pharyngeal
infection
1. Bed rest until acute phase protein return normal
2. Anti inflammatory:
if no/mild carditis- oral aspirin 80-100mg /kg/day in 4
doses for 4 weeks, taper over 4 weeks
If pericarditis/ moderate to severe carditis- oral
prednisolone 2mg/ kg/day in 2 divided doses for 4 weeks,
taper w addition of aspirin as above
3. Anti failure: diuretics & ACE inhibitors
4. Anti streptococcal therapy: IV C.Penicillin 50 000 U/kg/dose
6H, give oral erythromycin if allergic to penicillin
5. Chorea: control w haloperidol or valproic acid, diazepam
Secondary prophylaxis
IM benzathine penicillin every 3 4 weeks (<30 kg
0.6 mega unit, >30kg 1.2mega unit)
Oral penicillin 250 mg twice daily
If allergic to penicillin give oral erythromycin 250mg
twice daily
Duration of prophylaxis
Until age of 21 or 5 years after last attack of ARF
whichever was longer,
Lifelong if got carditis or valvular involvement
* The main symptoms of SC are believed to arise from an
imbalance among the dopaminergic system, intrastriatal
cholinergic system, and inhibitory gamma-aminobutyric acid
(GABA) system. Evidence of this imbalance has been
suggested by the successful control of chorea by dopaminergic
antagonists and valproic acid, a drug known to enhance GABA
levels in the striatum and substantia nigra.

33

Together in Delivering Excellence (T.I.D.E)


RHEUMATIC FEVER- d/t delayed immunologic rxtn of group A Beta hemolytic streptococcal infection of pharynx
HISTORY
1 Required Criteria and 2 Major Criteria and 0 Minor Criteria
1 Required Criteria and 1 Major Criteria and 2 Minor Criteria
Required criteria:
Evidence of antecedent Strep infection: ASO / Strep antibodies / Strep group A throat culture / Recent scarlet fever / antideoxyribonuclease B / anti-hyaluronidase
Major criteria (CASES)
Carditis
Arthritis
Subcutaneous nodules
Erythema marginatum
Syndenham chorea
Minor

Fever
Raised ESR/CRP
Arthralgia
Prolonged PR
Leukocytosis
Previous Rheumatic Fever

Evidence of carditis: cardiomegaly, heart failure, pericarditis, tachyC > fever (>10 per celcius), murmur /MR, MS, AR

34

Together in Delivering Excellence (T.I.D.E)


Infective Endocarditis [Baisyatul]
HISTORY
Presenting complain
Symptoms of systemic infection ( fever, malaise,
anorexia, loss of weight, headache, myalgia,
arthralgia)
Congestive heart failure ( SOB, orthopnoea, PND, leg
swelling)
Emboli symptoms ( neurological deficit, haematuria,
acute renal failure)
Predisposing factor:
50% occur in normal valve (acute IE)
Dental procedure
Dermatitis
IVDU (subacute IE)
Prosthetic valve
Chronic rheumatic valvular disease
Nosocomial infection (IV line)
Elderly

INVESTIGATION
For diagnosis: DUKE criteria
1.
-

Blood investigations
FBC (normocytic normochromic anemia, leucocytosis,
high ESR and CRP)
Blood culture n sensitivity ( 3 samples from different
sites)

2.
-

Urine FEME
Microscopic haematuria

3.

CXR (cardiomegaly)

4.

ECG (prolonged PR interval, regular rhythm)

5.

Echocardiography- transthoracic or transesophageal


(to look for vagetations)
*transesophageal more sensitive, better for visualizing mitral
lesion

PHYSICAL EXAMINATION
General examination
Thin, poor nutrional status (anorexia)
Septic sign ( feverish, tachycardia)
Finger clubbing
Splinter hemorrhage
Oslers node (tender nodules on finger pulp)
Janeway lesion (non tender macule on palms or sole)
Pale conjunctiva, subconjuctival petechiae
Dental hygiene
Specific examination
(chest)
Changing cardiac murmur
Cardiac failure ( gallop rhythm )
Pericardial rub
TR/MR
Lung crepitations
(abdomen)
splenomegaly (if emboli and cause abscess
formation/ heart failure)
Complete examination by
Fundoscopy to look for Roths spot (central pale area
with surrounding)
Urine dipstick to look for haematuria (microscopic)
MANAGEMENT
General management
Asses ABC
Heart failure management (oxygen, diuretics,monitor
input output chart)
Antibiotic therapy
Duration 2-4 weeks
Empirical therapy: benzylpenicillin + gentamicin
Adjust according to culture
Streptococci (as above)
Staphylococci (cloxacillin or vancomycin +
gentamicin)
Indication for surgery
Persistent bacteremia or fever despite optimal
antibiotic
Extensive valve ring infection
Prosthetic valve endocarditis esp staph aureus
Recurrent ep of systemic embolism

35

Together in Delivering Excellence (T.I.D.E)


COMMON QUESTION
1.

Causes of IE
Strep viridans (most common), staph aureus, enterococcus

2.

Diagnostic criteria for IE: DUKE criteria


Major criteria:

Positive blood culture


Typical organism in 2 separate culture OR
Persistently +ve blood culture (>12hour apart)

Evidence of endocardium involved


+ve echo ( vegetation, abscess, dehiscence of prosthetic valve) OR
New valvular regurgitation (change in murmur not sufficient)
Minor criteria:

Predisposing heart condition or IVDU

Fever >38

Vascular or immunological signs

Positive blood culture (that does not meet major criteria)

Positive echo (that does not meet major criteria)


How to diagnose:
2 major OR
1 major and 3 minor OR
All 5 minor criteria (if no major criteria)

3.

Prophylaxis antibiotic
According to AHA guideline, only high risk patient should be considered giving prophylaxis for dental procedure
Because the risk associated with adverse effect of antibiotics usually outweighs the benefit of prophylaxis
2008 Cochrane review found there was no evidence prophylaxis antibiotic treatment was either effective or ineffective at preventing
IE in at-risk population undergoing invasive dental procedure
According to OXFORD handbook~ also explain the same
High risk group that need prophylaxis:
Prosthetic cardiac valve
Congenital heart disease
Oral amoxicillin 2g single dose 30-60 minutes before procedure
st
nd
If allergic give 1 or 2 cephalosporin

4.

Classification

ACUTE ENDOCARDITIS
Sudden onset, progress
rapidly(days)
Affect normal valve
Common organism: Staph.
Aureus, GBS

Reference:
1.
2.
3.
4.

Oxford handbook of clinical medicine


Express revision short case UM
Sarawak handbook
Medscape

SUBACUTE ENDOCARDITIS
Insidious and progress
slowly( weeks to months)
Affect abnormal valve
Common organism: strep.
Viridans,
-

PROSTHETIC VALVE ENDOCARDITIS


After valve replacement
Early PVE occur within 3 months
Late PVE occur after 3 months
Common organism: strep.
Viridans, staph. Aureus,
enterococci

36

Together in Delivering Excellence (T.I.D.E)


Glomerulonephritis/ Nephrotic Syndrome [Aiman]
HISTORY
Age: 6-10years old
Acute nephritis syndrome:
- Oedema-facial puffiness
- Hematuria-tea coloured
- Oligouria-reduce urine
- HPT-headache, burring of vision
- Azotemia-fatigue, reduce
alertness, confusion(high urea
and creatinine)
- Post infection AGN: impetigo(24w), pharyngitis (1-2w), cellulitis
Rule out other causes:
- (Wegener granulomatosis)
Sinusitis, nephritis, pulmonary
infiltration
- (HSP) joint pain, swelling,
abdominal pain and purpura
- (SLE) arthralgia, malar rash, oral
ulcer
Complication:
HPT(encephalopathy-seizure)
APO(SOB)
ARF

PHYSICAL EXAMINATION
Nephrotic syndrome clinical
syndrome of massive
proteinuria
- Oedema-generalized
swelling (periorbital,
scrotal, vulva, leg, ankle
swelling and abdominal
distension) SOB
- Proteinuria-frotty urine
Rule out other causes:
- Post strep AGN
- SLE
*after rule out this two
causes then we can say it is
idiopathic (most common)

Urinalysis
-RBC cast-hematuria vs hemoglobinuria

*culture and sensitivity: to detect


any growth when UTI is suspected
as cause or aggravating factor in
both nephron and nephri

Gross proteinuria(>2+)
-ve

- Proteinuria:>1gm/m2/d
- Early morning(for more
concentrated urine)
protein creatinine
(>3.5mg/mg)
- Quantitative urinary
protein excretion
i)spot protein creat ration
ii)24 hr urine protein

Blood investigation
*both
- FBC-anemia, leukocytosis
- BUSE +creatinine renal f(x)
(high urea, high creat)

- Serum cholesterol
:hypercholesterolaemia
- LFT (se albumin):
hypolabuminaemia
(<25g/dL)

Post strep: bacteriological and


serological: ASOT- >200, increase
anti DNAse B, throat and skin
swab.
Complement:C3 reduce, C4 normal

SLE (for both to rule out)


-ANA/anti dsDNA:+ve in SLE
Complement: both C3 and C4
reduce

Imaging: ultrasound of kidney


Special: renal biopsy

Differential Diagnosis :
i) Henoch-schonlein purpura
ii) SLE
iii) Vasculitis (Wegener
granulomatosis)
iv) Rapidly progress
Glomerulonephritis

Complication:
Hypovolemia (fluid go to
rd
3 space)
Primary peritonitis
Thrombosis

INVESTIGATION
Urine dipstic
- Protein +ve(usually>2+)
- Hematuria +ve
- Nitrite (positive is evidence of
infection)

Nephritic
- Altered consciousness level
- Oedema but less severe than
nephritic
th
- BP-Hypertension(>90 centile)
- Scar at limb

Nephrotic
- Leuconychia-due to
hypoalbuminimia
- Generalized oedema-look at
face, limb, genitalia
- Distended abdomen with
ascites
- Patient with generalized
oedema with cold peripheries,
poor pulse volume,
hypotension hypovolemia
- With fever and abdominal
tenderness peritonitis
Differential Diagnosis :
SLE
Post strep AGN

MANAGEMENT
Nephrotic chart (fluid intake, Urine output, BP, ddaily weight, urine
protein)
1. Fluid restriction (to control
oedema + circulatory overload
during oliguric phase)
-no added salt diet
2. Diuretic : give frusemide
3. Antibiotic: penicillin V x 10/7
4. Look for complication
a) HPT
- encephalopathy(seizure)
*control the BP
- heart failure (APO)
*oxygen, diuretic, fluid
restriction
b) ARF (acidosis, uremia,
hyperkalemia[cardiac
toxicity]hyperphospahtemia,
hypocalcimia)
*monitor renal function
Follow up fpr 1 yr (3 monthly)
Monitor BP, urinalysis & renal
function
Repeat C3 if not normalized
during discharge (after 6 wk)
Short term outcome: excellent
(mortality <0.5%)
Long term outcome: 1.8%
progress to CKD

1. Confirm the diagnosis fulfill 4


criteria(gross proteinuria,
oedema, hypoalbuminimea,
hypercholesterolemia)
2. Exclude other causes (SLE,
PSAGN) then diagnose as
idiopathic
3. Normal protein diet, with
adequate calories, no added
while oedema
4. Careful assessment of
haemodynamic status(hypo vs
hypervolemic)
-fluid restriction only during
chronic oedematous state
5. Penicillin V:during diagnosis
and relapse
diuretic: no need in steroid
responsive
6. Steroid therapy
7. Tx complication
a) Hypovolemia: Infuse human
albumin(no frusemide)
b) Primay peritonitis:penicillin
rd
and 3 gen cephalosphorin
c) Thrombosis: anticoagulant

37

Together in Delivering Excellence (T.I.D.E)


COMMON QUESTION
1.

Indication for renal biopsy:


a. Steroid resistant nephrotic syndrome (not respond to 4 week treatment of 60mg/m2/day prednisolone)
b. Features suggestive non-minimal change NS
c. Persistant HPT, renal impairment with or without or gross hematuria
d. Severe acute renal failure require dialysis
e. Feature suggestive non PSAGN
f. Delayed resolution(PAGO):
i. Persistant Proteinuria:>6month
ii. Azotemia:>3w
iii. Gross hematuria:>3w
iv. Oliguria:>2w

2.

Steroid therapy:
a. To induce remission (rule of 4)
i. 4weeks: 60mg/m2/day
ii. 4weeks: 40mg/m2/EOD
iii. 4month: reduce 25% dose monthly
*remission: urine dipstick trace or nil for 3consequtive day
b.

In relapse (urine protein >40mg/m2/hr orurine dipsticx >or equal to 2+ for 3 consequtive days)
i. 60mg/m2/day until remission
ii. 40mg/m2/EOD for 4weeks
*if patient has UTI with no gross oedema, steroid therapy is not necessary, treat the infection

c.

Frequent relapse (2 or more relapse in 6 month or 4 or more in 1 year)


i. 60mg/m2/day until remission
ii. 40mg/m2/EOD for 4weeks
iii. Tapered down the dose 2 weekly and keep as low an alternate dose as possible for 6 months. (if
relapse during therapy, treat as relapse)

d.

Steroid dependent (2 or more relapse during steroid taper or within 14 days cessation of steroid)
i. If non-toxic: reinduce steroid
ii. If stroid toxic (short stature, striae, cataract, glaucoma, severe cushingoid features: consider
cyclophosphamide therapy
1. For steroid dependent NS with steroid toxicity
2. Counsel parent about effectiveness and SE(leucopenia, gonadal toxicity, hemorrhagic cystitis)
3. Need to monitor FBC and urinalysis
4. Dose:2-3mg/kg/day x 8-12weeks

3. Watch out for adrenal crisis (in long term corticosteroid therapy when they undergone situation of stress)

38

Together in Delivering Excellence (T.I.D.E)


Thalassemia [Amy]
HISTORY
**Autosomal recessive
o First presentation age, pallor/jaundice/FTT

B thal major : 3-6 months

B thal intermedia : later than 18m

Alpha thal intermedia : at birth


o Family history consanguinity, genetic screening done?
o Transfusion pre & post Hb level, condition, no of PC, frequent,
transfusion reaction, Hep status

Major : every 3-4 weeks

Intermedia :when symptomatic


o Iron chelation therapy (Desferal) when started
o Regular follow up growth chart, blood level, ferritin level
o Any admission d/t complication
o Splenectomy done? On Penicillin?
o Paediatrics history
o Diet history should avoid high iron diet
o Complications

Iron overload : hypothyroidism, DM, CLD(cirrhosis),


cardiomyopathy(cardiac haemosiderosis sterile
pericarditis, arrhythmia), Short stature decreased puberty
growth spurt, Delayed puberty

Premature bilirubin stone formation (cholelithiasis)

HypoPTH

Vision & Hearing Desferrioxamine toxicity


o

Social Hx : disease impact on children/family

Academic performance, school missed, able to take part in


PJK

Body image, pubertal anxiety

Financial consideration transport, other cost


Understanding of disease, perception, compliance
INVESTIGATION
1. FBC
MCHC anaemia
Leukopenia, Thrombocytopenia if hypersplenism
2.

Peripheral Blood Film / FBP


Microcytic hypochromic RBCs
Anisocytosis, poikilocytosis, target cells, fragements(IV
haemolysis)

3.

Haemolysis marker
Reticulocyte count high
LDH elevated

4.

LFT deranged liver enzyme (cirrhosis), unconjugated bilirubin


raise

5.

Hb Electrophoresis
Beta thal : HbA2, HbF elevated, decreased hbA
Alpha thal : HbA, HbA2, HbF decreased, HbH band in HbH
disease

6.

7.

Genotyping
Beta thal : point mutation, Alpha thal : deletion mutation

Serum Ferritin
Increase iron overload
**iron chelation tx when Se Ferritin >1000mcg/L

PHYSICAL EXAMINATION
General Examination
o Growth : height, weight-short stature, sexual development
o Skin : pallor/jaundice/bronze-iron overload
o Thalassemic facies : frontal bossing, prominent maxilla
Abdomen
o Desferal scar pigmented, round, no lipodystrophy
o Surgical scar open/lap
o Hepatosplenomegaly
Chest
o Heart failure displaced apex, murmur, bibasal crept
Others
o Sign of CLD
o Signs of hypothyroidism
o Screen visual acuity
o Tanner staging

Diagnosis :
(Beta thal major/HbE/HbH) , transfusion dependant/not, Clinically
thal major/intermedia, ongoing problem&issues

MANAGEMENT
Blood transfusion
o Thal intermedia or major
Chelation Therapy
o When Se Ferritin reach >1000mcg/L
o Deferoxamine (Desferal)
Via slow subcutaneous infusion pumps (6-8h), 6 days a week
S/E : ototoxicity, ophthalmotoxicity
o

Deferiprone
Orally TDS, S/E : agranulocytosis

Splenectomy
Indication : increase in red cell transfusion requirement by 50% or
more over one year
Vaccine :H. Influenzae, Strep Pneumoniae, Neisseria meningitides
Prophylaxis : Penicillin
Cholecystectomy if stone present
Screening for endocrinopathies
o Monitor pubertal development
o DM screening early diagnosis, good control
o Thyroid screening hypothyroidism
Ix : Ca/Mg/PO (hypoPTH), TFT, FBS, OGTT, LH/FSH

39

Together in Delivering Excellence (T.I.D.E)


1.

2. Cause of jaundice is thalassemia pt


o Aplastic crisis/haemolytic crisis
o Gallstone
o Infection Hep B, CMV, EBV
o Cirrhosis
o Other unrelated causes eg autoimmune, drugs etc
3. Transfusion reaction

40

Together in Delivering Excellence (T.I.D.E)


Leukemia [Hidayah]
HISTORY
Definition
Acute leukemia : Heterogenous group of malignant disorder which
is characterised by uncontrolled clonal & accumulation of blast cells
in the bone marrow & body tissues.
Epidemiology
Commonest within 2-10 years (Peak : 3-4 years)
Incidence decreases with age AND rise >40 years.
In children - most common malignant disease (85% )
History
1. Common complaint
Fever
Pallor
Lethargy, malaise
Petechiae or ecchymoses,nosebleed
bone pain, joint pain refuse to walk,limping gait
CNS disease (mets to CNS)
-Headache, vomiting, Abnormal sensorium, Hypothalamic
symptom(abn wtgain&hyperphagia)
Testicular swelling
2. Family hx of blood malignancy (leukemia,lymphoma, etc)
3. DDx
i. Infection
a. Dengue (retro-orbital pain,rashes,myalgia, from
dengue area)
b. TB (contact with TB pt,night sweats)
c. Epstein-barr virus (infectious mononucleosis)
hx of sore throat
d. OM (fever, bone pain, swelling, redness,
focal painful swelling,hx of trauma)
e. Rheumatic fever (C-carditis, A-arthritis
(polymigratory), S-subcutaneous nodule,
E-erythema marginatum, S-sydendham chorea)
ii. Non-infectious
a. Juvenile rheumatoid arthritis(fever, morning
stiffness,rash)
b. Haemophilia (bleeding tendency)
c. ITP(preceding URTI/viral infection)
d. Aplastic anemia (exposure to chloramphenicol, radiation)
iii.Malignant disease (Osteosarcoma, Ewing sarcoma)
(constitutional sx)
iv. Trauma

INVESTIGATION
1. FBC
Anemia : normochromic normocytic
Thrombocytopenia
WBC : normal,leucopenia or leukocytosis
2. Peripheral blood film/smear

3. Bone marrow aspiration (confirm dx)


i. Hypercellular
ii. Presence of blasts cells
iii. Normal haemopoiteic cells are suppressed
- FAB classification : presence >30% blasts in bone marrow
- WHO classification : presence >20% blasts in marrow
4. Cytochemistry staining
Myeloperoxidase (+AML)
Periodic acid Schiff (+ALL)
Acid phosphatase (+ALL)
1. Immunophenotyping
- differentiate T-ALL and B-ALL
-Monoclonal antibodies used :
AML : CD13, CD33
ALL : B-ALL CD10, CD 19, CD22
T-ALL CD3, CD7
2. Cytogenetics or karyotyping
-prognostic value
-PCR, FISH, DNA microarrays

MANAGEMENT
General
1. Blood & blood product support
2. Insertion of Central venous catheter-better access for
chemotherapy, blood products, AB
3. Prevention of vomiting
4. Prophylaxis & treatment of infection
5. Prevention of tumor lysis syndrome
6. Social & psychological support

PHYSICAL EXAMINATION
i.
ii.
iii.
iv.
v.
vi.
vii.

Pallor (palm,sole&conjunctiva), bruises


Lympadenopathy (cervical, inguinal)
Hepatosplenomegaly
Painless testicular enlargement
CNS involvement (ICP blurring vision,diplopia)
Rashes, hess test (dengue)
Examination of affected bone (JRA,OM,hemophilia)

Specific
1. Chemotherapy
a. Induction : 4-6w initial treatment phase for rapid reduction
of leukemic burden
b. Consolidation(intensification)
c. Maintainance
d. CNS prophylaxis : prevent leukemia relapse in CNS (IT
methotrexate)
2. Stem cell transplant
Peripheral/bone marrow
Allogenic/autologous

41

Together in Delivering Excellence (T.I.D.E)


1.

Principles of leukaemogenesis

Neoplastic cell is pluripotent cell or early


progenitor cell

1.
2.
3.

Dysregulation of cell growth


Proliferation of the leukemic clone
Blocked of differentiation at an early stage

2.

Causes of Leukemia
1. Hereditary Factors ( Fanconis anaemia, Downs syndrome, Ataxia telangiectasia)
2. Radiation, Chemicals exposure (Benzene, herbicides&pesticides, smoking) & Drugs (alkylating agents,chloramphenicol)
3. Virus related Leukemias (Retrovirus - HTLV 1 & EBV)
4. Pre-existing blood disorders (myelodysplastic disorders,Myelofibrosis, aplastic anaemia, paroxysmal nocturnal haemoglobinuria
,CML)

3.

Prognostic for ALL


Age
Sex
TWBC
Chromosomal abn

4.

Good
1-10
Female
<50 000
Trisomies (4,10)

Poor
<1; >10
Male
>50 000
t(9,22)

Complication
1. Tumor lysis syndrome (TLS): massive tumor cell deat with rapid release of intracellular metabolites exceeds excretory capacity of
kidney acute renal failure
2. Can occur b4 chemo is started
3. Characteristic
i.
Hyperuricemia d/t release of intracellular purines
ii.
Hyperkalaemia d/t tumor cell lysis or renal
iii.
Hyperphosphatemia lead to tissue damage (renal failure,pruritus,gangrene)
iv.
Hypocalcemia

5.

Prevention of TLS
Hydration -Double hydration (3000ml/m2/day)
Alkalization of urine (add NaHCO3 into IV fluid, keep urine pH7-7.5)
Allopurinol (10mg/kg/day-max300mg/day)
KIV delay chemotherapy until metabolic status stabilized
Close monitoring electrolyte BUSE,Ca, PO4, uric acid, creatinine, bicarb
Strict i/o chart

6.

Chemotherapy drugs &side effect

42

Together in Delivering Excellence (T.I.D.E)


Haemophilia (Bleeding Disorder) [Hamizah]
HISTORY
1.

Sign and symptoms (Bleeding, Bruises)


Site of bleeding
Frequency
Onset (Spontaneous?) or caused by trauma?
Does it follow a viral infection (ITP)
Is it acute, chronic or recurrent?
Associated with rash like petechiae or purpura?
-

2.
3.
4.
5.

6.
7.

8.

9.

Joint bleeding signs: tingling, cracking, warmth, psin, stiffness and


refusal to use joint.
How severe the joint problem.
Any symptoms of headache, stiff neck, vomiting, lethargy,
irritability suggesting CNS bleed?
Any symptoms of hematuria, renal colic?

Any bleeding after immunization, circumcision, epistaxis, hematuria,


dental extraction or operation?
Past history of requiring blood transfusion.
Hx of easy bruising, heavy menses
History of concomitant illness eg: chronic inflammatory disorder, liver
disease, autoimmune disease, hematologic malignancies and allergic
drug reactions.
Past hx : Hepatitis, HIV/AIDS, SLE
Family history :
Consanguineous marriage
Bleeding disorder in family
Severity of bleeding disease in family
Drug history
Alcohol
Warfarin
Steroid
NSAIDS
Anticonvulsant
Complications
Symptoms of anaemia & SHOCK

INVESTIGATION
Laboratory
FBC: Hb (anaemia?), WCC, platelet
-

Bleeding time
Raised in vWB disease, platelet disorder, on aspirin

vWB factor

Coagulation profile
aPTT prolonged in liver disease, haemophilia,
prothrombin time prolonged in liver disease or
anticoagulant

Assay for Factor 8, factor 9

PHYSICAL EXAMINATION
General
Pallor
Lymphadenopathy
Hepatosplenomegaly
testicular infiltration (enlargement)
Bone tenderness
Renal infiltration
Skin infiltration
(presence of one or more of above suggests leukaemia)
Type of bleeding:
Bruises: site, size, number, colour
Purpura: palpable in HSP
Petechiae: sites (buccal mucosa, palate, tongue, retina), no
of lesions indicates platelet or vascular disorder
Examine if bleeding occurs in:

Musculoskeletal system (joint) look for tenderness, pain


with movement, decreased range of motion, effusion and
warmth.

CNS look for altered mental state, meningism and


abnormal neurological findings.

Genitourinary system bladder spasm/ distension/ pain


and costovertebral angle pain

Retinal examination to dx intracranial haemorrhage


Signs of significant blood loss:

tachycardia

Tachypnea

Hypotension on standing

MANAGEMENT
If shock: Resuscitation
Lifestyle changes:
Avoid violent contact sport
Avoid aspirin and NSAIDS that affect platelet function
For toddler, safety measures include anticipatory guidance
including the use of car seats, seat belt and bike helmets
If minor bleeding: pressure and elevation of the part. Desmopressin
raises factor 8 levels.
If major bleeding: increase factor 8 levels to 50% of normal
If life threatening bleeding: need 100% eg: recombinant factor 8.
Early appropriate replacement therapy
Factor VIII prophylaxis program (aim is to convert severe
disease into moderate one. WHO recommendation is
Factor VIII 25-40 IU/kg on alternate days a week)

43

Together in Delivering Excellence (T.I.D.E)


Common/Possible Question in Exam + Answer
1.

What is hemophilia?
Hemophilia is X-linked recessive disorder. Hemophilia A is factor 8 deficiency while hemophilia B(Christmas disease) is factor 9
deficiency.

2.

The common site of bruising: anterior tibia and knee.

3.

Ddx
-

Coagulation disorder: Haemophilia A&B, von Williebrand disease


Platelet disorder: ITP
Vascular cause: Marfan syndrome, SLE
Laukaemia

4.

What are the characteristics of abnormal bruising?


a. Large bruises that raised and out of proportion to theinjury that caused it
b. Unexplained bruises that occur without history of trauma
c. Bruises that seems last more than few weeks
d. A family histry of bleeding or easy bruising in the family suggest genetic bleeding such as von Williebrand disease and
haemophilia.
e. Any bruises in an infants who has not started to crawl, cruise or walk
f. Bruises in unusual places eg: chest, back, hand, ear, face and buttock.
g. Bruises associated with burn, scald, fracture,
h. Excessive bleeding after dental procedure or surgery
i.
Epistaxis that last for longer than 15 minutes after proper tx with direct pressure
j.
Menstrual periods longer than 7 days or heavy for more than 3 days.

5.

What are primary causes of death in patients with severe haemophilia?


The main cause is HIV/AIDS acquired from treatment with contaminated blood product
Second is intracranial haemorrhage
Other cause: cirrhosis and obstruction of the airways

6.

What are coagulation pathways?

7.

How can synovitis be differentiated from an acute haemarthrosis?


The swelling of synovitis does not respond to a single infusion of factor.
The joint is less painful than that of acute hemarthrosis and the range of motion is frequently not limited

44

Together in Delivering Excellence (T.I.D.E)


Diabetes Mellitus Type 1 (Paeds) [Nur Diana]
HISTORY

PHYSICAL EXAMINATION

*Etiology:
i) Result from autoimmune destruction of insulinproducing Beta cells (islets) of the pancreas.
ii) Can also result from rare inherited defects in
mitochondrial genes

Assess hydrational status, nutritional well-being


Assess vital signs; tachycardia, tachypnea
Monitor growth & development; plot growth chart;
do developmental assessment
after 5 years of onset diabetes- evaluate for
complications

*Epidemiology:
i) Genetic determinants play a role in the susceptibility to
DM1
ii) Siblings/offspring of the patient have a risk of 3-6% to
get DM1
iii) identical twin 30-60% at risk to get DM1
*Sign & Symptom:
Early
Polyuria
Polydipsia
Polyphagia
Weight loss

Late
Vomiting
Dehydration
Abdominal pain
Drowsiness

*Cx:
i) DKA; may leads to cerebral edema.
ii) Dehydration
iii) Long term- retinopathy, nephropathy, neuropathy and
macrovascular disease.
INVESTIGATION

1.
2.
3.
4.
5.
6.

Random blood glucose > 11.1 mmol/L


Fasting blood glucose > 7.o mmol/L (no calori intake at
least 8 hours)
OGTT rarely needed in children
Urine dipstick; check for ketone, glucose level
Serum TSH; 1x/year- early detection of autoimmune
disease.
blood lipid profile hyperlipidemia

**Ix. to look for any complications is ordered accordingly to


the patient history and clinical presentation.

MANAGEMENT

A.
B.
C.
D.
E.
F.
G.
H.
I.

Insulin therapy
Home blood glucose monitoring
Diet
Exercise
Education
Monitor hba1c
Monitor growth and development
Screening for celiac disease
Refer to diabetes support group

45

Together in Delivering Excellence (T.I.D.E)


Frequently asked questions (FAQs)
1.

What is meant by somogyi phenomenon?

It occurs when patient increase evening dose of insulin. Cause nocturnal hypoglycemia. Lead to exaggerated counter-regulatory
response and rebound hyperglycemia in the next morning.
2.

How to diagnose DM1?


i)
Symptoms + random plasma glucose > 11.0 mmol/L ; OR
ii)
Fasting plasma glucose > 7.0 mmol/L; OR
iii)
2-hours post prandial > 11.0 mmol/L during an OGTT.

** (i) or (ii) or (iii)


3.

When should we test for ketone?


a)
Illness with fever and/or vomiting
b)
Persistent blood glucose level > 14 mmol/L; in an unwell child, young child, insulin pump users, or patient with
history of prior episodes of DKA.
c)
Persistent polyuria with elevated blood or glucose in urine
d)
Drowsiness
e)
Abdominal pain or rapid breathing

46

Together in Delivering Excellence (T.I.D.E)


Epilepsy (Recurrent, Unprovoked Seizure) [Khiru]
HISTORY
-

What pt doing before fits?


Duration of fits
st
1 episode?
Date of last seizure
A/w fever? (febrile seizure)
Up rolling of eyeball, stiffening of limbs, jerky movement
Postictal lethargy/ neurological deficits
Urinary incontinence, tongue biting, trauma while falling
down
Perioral cyanosis
How seizure stopped? (spontaneously or by med)
If multiple seizure regain consciousness aft each
seizure? Total duration? (tro status epilepticus)
Family history of epilepsy, dysmorphism, downs
syndrome, frequent abortion
Obstetric history (fetal distress, birth trauma, instrumental
delivery, neonatal jaundice)
Current AED
AED history (max dose, efficacy, reason to stop)
Previous eeg, mri findings
INVESTIGATION
nd

Recommend only if 2 afebrile seizure occur


1. FBC
2. RBS
3. BUSE+ calcium+ Mg
4. Lumbar puncture if suspect brain infection
5. Toxicology if suspect drug exposure
st
6. EEG recommended after all 1 unprovoked seizure
7. Neuroimaging indication for: persisting postictal deficit,
condition of child not returned to baseline within several
hours after seizure
8. Anticonvulsant not indicated unless:
Neurological deficit in child
Unequivocal epilectic activity on eeg
Risks of having further seizure unacceptable
Brain imaging show structural abnormality

PHYSICAL EXAMINATION
-

Look for dysmorphism


Neurocutaneous sign (cafe au lait, axillary freckles, Lisch
nodules, bony lesion, optic glioma, relative,
neurofibroma)
Complete CNS & developmental examination

MANAGEMENT
A) Acute seizure management
Rapid cardiopulmonary assessment
Put pt in lateral position with head lower than trunk
(recovery position) aid drainage of secretion
Clear airway & give oxygen via hig flow mask
DO NOT restrain seizure movement
DO NOT put anything in between teeth
Establish IV/ intraosseous assess
Diazepam (rectal=0.5mg/kg; iv=0.3mg/kg) max
10mg
If seizure persist phenytoin
If seizure cont aft >10 mins, consider IV midazolam or
phenobarbitone pr sodium valproate
(start inotropic support & arrange ICU, secure airway
& prepare use mechanical ventilation; titrate
phenobarbitone to achieve burst-suppression pattern
on EEG)

47

Together in Delivering Excellence (T.I.D.E)


Questions
1.

Classification of seizure types


Generalized

Tonic-clonic, Abscence (typical, atypical), Myoclonic, Tonic, Clonic, Atonic


Focal seizures
Epileptic spasms

2.

Principles of antiepileptic drug therapy for epilepsy

Treatment recommended if >2 episodes

Monotherapy as far as possible

Increase dose gradually until epilepsy in controlled OR max dose OR s/e occur
nd
st
nd
st

Add on 2 drug if 1 drug failed. Optimize 2 drug, try withdraw 1 drug

Rational combination therapy (combine drug with diff MOA & consider their spectrum of efficacy, drug interactions &
adverse effect)

Monitor drug level only to check compliance OR polypharmacy (drug interaction suspected)

3.

Advice for parents

Compliant to med

Treat for 2 years seizure-free period taper off over 3-6 months

Sudden withdrawal of drug breakthrough seizure

If epilepsy is photosensitive, watch TV in a brightly lit room

Avoid sleep deprivation & alcohol

DO NOT lock bathroom

No cycling in traffic, swimming alone

Inform school

4.

AED & side effects

48

Together in Delivering Excellence (T.I.D.E)


Celebral Palsy [Dodi]
HISTORY
Definition :
Disorder of movement and posture d/t non progressive lesion of
motor pathways in the developing brain
-must be no loss of milestones ACQUIRED previously
-mostly diagnosed after 2 y/o( prof salmi - 3 y/o) bcoz before this
neuron not complete myelination yet.
Risk factor: premature and twin birth
S&S:
asymmetric gross motor fx,increased ms tone/floppiness,
Feeding difficulties ( slow feeding,ganging,vomiting d/t oromotor
incoordination),abnormal gait once walking is achieved,generalized
windswept deformity d/t contracture and marked ms wasting
Hx :
-

Birth hx (antepartum,intrapartum,postpartum)
Intrapartum asphyxia : APGAR score- cry spontaneously or
not?
Dev milestones :
Gross N fine motor, hearing, speech, language,
Social, behavior N emotional
Any major events like seizures,aspiration -pneumonia(lung infection,mental retardation

Describe it, frequency and interval of free sx


Mobilization
INVESTIGATION

no definitive laboratory studies for diagnosing cerebral palsy, only


studies to rule out other symptom causes, such as metabolic or
genetic abnormalities,
*Thyroid function studies - Abnormal thyroid function may be
related to abnormalities in muscle tone or deep tendon reflexes or to
movement disorders.
*Lactate and pyruvate levels - Abnormalities may indicate an
abnormality of energy metabolism (ie, mitochondrial cytopathy).
*Ammonia levels - Elevated ammonia levels may indicate liver
dysfunction or urea cycle defect.
*Organic and amino acids - Serum quantitative amino acid and urine
quantitative organic acid values may reveal inherited metabolic
disorders.
*Chromosomal analysis - Chromosomal analysis, including karyotype
analysis and specific DNA testing may be indicated to rule out a
genetic syndrome, if dysmorphic features or abnormalities of various
organ systems are present.
*Cerebrospinal protein - levels may assist in determining asphyxia in
the neonatal period. Protein levels can be elevated, as can the
lactate-to-pyruvate ratio.
*Imaging :Confirm extent of damage by MRI/CT scan
Others:- (EEG) is useful in evaluating severe hypoxic-ischemic injury N
diagnosis of seizure disorders
- Electromyography (EMG) and nerve conduction studies are helpful
when a muscle or nerve disorder is suspected

PHYSICAL EXAMINATION
General
Inspection
Position of the patient
Pt is conscious N alert?? (usually pt sleeping)
If conscious- I can/cannot build rapport with him/maintain eye
contact or not/playing or not with toys
Hydrational N nutritional status adequate?
No respi distress if presence evidenced by tachypnea,nose
flaring,use of accessory muscle N intercostals recession
Head appears small but I ll confirm later by plotting head
circumference chart
Body size is appropariate/inappropriate with age but ill confirm
later by plotting growth chart
On nasogastric tube( if present)
There is minimal movement ? exmple : at left upper
limb,others ______
Abn : fisting of hands,scissoring of legs,flexion of extremities
Abn movement-choreoathethoid( not chorea; in Huntington
chorea,and syndenham chorea)
Wheelchair/stroller- walking difficulties
Note any bed sores
UMNL : Hypertonia,power at least 3 (cannot be elicited properly as pt
do not follow command), hyperreflexia, clonus,babinski positive,no
fasciculation

MANAGEMENT
Multidisciplinary approach :
Rehabilitation
a. Physiotherapy-fx in sitting,standing,walking,gait
training,mechanical aid (support,prevention and correction
of deformities,assisting wanted movement,controlling
involuntary mvmt)
b. Occupational therapy-increasing muscular coordination and
eye hand coordination (to improve adls)
c. Speech therapy-to stimulate language development
d. Behavioural therapy

Orthopaedic mx
Tenotomy to correct contractures
Tendon transfers to correct deformities from muscle
imbalances
- Arthrodesing operations
Nutritional needs
- Increasing needs d/t involuntary movements,contractures
- Poor intake d/t dysfunctional swallowing,inability to chew
- Require supplementation
Medication
- Control seizures-sodium valproate?
- Muscle relexantbeclofe,clonazepam N botox injection
- Controlling mvmt
- Management of dysf(x) bowel act
- Aspiration pneumonia anti reflux meds-ranitidine
-

49

Together in Delivering Excellence (T.I.D.E)


1.

CAUSES OF CP (commonest)
ANTENATAL Vascular occlusion,cong infection (TORCHES), congenital malf *
INTRAPARTUM-HIE,perinatal asphyxia
POSTPARTUM
o Prematurity (PERIVENTRICULAR LEUKOMALACIA) *
o Low BW infant*
o Kernicterus*
o Meningitis*
o Head injury- MVA,child abuse*
o Stroke*
o Symptomatic hypoglycemia

2.

Site of involvement?
*UMN( pyramidal)- spastic
*cerebellum- ataxic hypotonic
*basal ganglia/extrapyramidal tr dyskinetic

3.

Types of CP

*UMN( pyramidal)- spastic


hemiplegia(UL >LL)
o fisting of affected hand
o scissoring of LL
o pronated forearm
quadriplegia(LL>UL)
o a/w hx of HIE
o extensor posturing
diplegia(Minimal affected UL)
o Scissoring legs
o prone in premature infants
*cerebellum- ataxic hypotonic
hypotonia,truncal ataxia,dysmetria,gross incoordination
incoordinate mvmt,intention tremor,ataxic gait
*basal ganglia/extrapyramidal tr dyskinetic
frequent involuntary mvmt :chorea,dystonia(proximal lmb),athetosis(distal limb)
floppiness,poor trunk control,delayed motor dvlpmt
intellect relatively unimpaired
* both spastic N extrapyramidal types-mixed
4.

Co-morbidites? (Sign N sx)

a) Head
Microcephaly
Seizures
Mental retardation
Developmental delay
Cortical blindness
Hearing prob
Speech d/o
Feeding prob

b) Chest n abdomen
GERD
Hyperactive a/way
disease
Recurrent aspiration
Chest deformity
Bronchopneumonia
Constipation

c)others:
Contractures,
Spasticity,
Abnormal movement,
Wearing pampers,
Dislocated hips
Scoliosis

d)DDx :
a)with weakness
anterior horn dz (spina bifida, vp
shunt)
NMJ (mys.gravis)
peri.neuropathy (charcoat
marrie tooth/ HMSN)
duschene muscular atrophy
b)w/o weakness
down syndrome
prader willi synd,
ECP

50

Together in Delivering Excellence (T.I.D.E)


Downs syndrome [Amal]
HISTORY
- Note abt the chief complaint: health/hearing/vision status,
developmental delay
(Newborn)
GI tract : duodenal atresia TOF, p.stenosis-vomiting
Anorectal malformation, Hirschprung no stool
CVS : AVSD, VSD, ASD, TOF, PDA
CNS : hypotonic, laxity
Other: hypothyroid, hip dislocation, cataracts
(Childhood)
Delay dev milestone, IQ problems, seizures, recurrent URTI,
hearing loss, visual impairment, leukemia, hypothyroid
Short : myb d/t ht dse, nutritional def d/t feeding problem,
coeliac dse, thyroid
Under/overweight
Atlantoaxial instability mostly asymptomatic but cn have
neck pain, torticollis, limited ROM, gait problem, sensory
deficit, hyperreflex, spacticity
(Adult)
Female :delay menarche. Fertility ok
Male: infertile d/t low testosterone
- risk for Alzheimer/ dementia
Other hx: obstetric hx- maternal age
Family hx of downs

PHYSICAL EXAMINATION
SKULL :
Flat occiput, bracy&microcephaly, large fontanelle, hypoplasia of
maxillary
EYE :
Epichantal folds, upslanting palpebral fissure, brushfiled spots,
strabismus, nystagmus, cataracts
FACE:
Small nose,ear,&mouth. Protruding tongue, , flat nasal bridge, flat
facial appearance, hypertelorism, micro&hypodontia, smooth
philtrum, thin upper lip
NECK :
Broad, shot, generous nuchal skin
ABDOMEN :
Protuberant, umbilical hernia
HAND:
th
Single transverse palmar crease, short 5 finger w clinodactyly,
st
nd
wide space btwn 1 and 2 toes
LIMBS:
Short extremities, broad hands
CNS:
Joint hyperflex, hypotonia
BEHAVIOUR :
Natural spontaneity, cheerful but some are anxious

INVESTIGATION
Laboratory :
FBC BMA for leukemia
TSH for hypothyroid
Pap smear every 1-3 yr from first intercourse
Cytogenetic : karyotyping
Echo: for congenital ht dse
Radiograph :
Cervical xray to measure atlantodens distance TRO
instability at 3 y/o but not indicated in most cases
Prenatal screening:
Markers: Low maternal serum alpha fetoprotein, high
hCG, low unconjugated estriol
Ultrasound: hypoplastic nasal bone, thick nuchal fold,
echogenic bowel, short bone, pyelectasis (dilatation of
renal pelvis)
Invasive test: amniocentesis at 14-16 weeks gestation
Chronionic villus sampling 10-13 weeks gestation but
risk of contamination, transverse limb deficiency
Percutaneous umbilical blood sampling

MANAGEMENT
Medical :
-

intervention for specific issues : surgical/meds for


ht dse, abx for infections, infant stimulation
programs, special education,
physical/occupational/speech tx
goal is for the children to develop to their full
potential
parents support group
supportive care
balanced diet for appropriate weight
follow ups for cardiac/thyroid/hearing/visual
assessment
dental check-ups at 2 y/o and cont 6 monthly

Surgical :
for specific issue etc heart disease

51

Together in Delivering Excellence (T.I.D.E)


Common/Possible Question in Exam + Answer

Sx of congenital hypothyroidism : GOOD BABY- decreased activity, Large anterior fontanelle, Poor feeding and
weight gain, Small stature or poor growth, Jaundice, Decreased stooling or constipation, Hypotonia, Hoarse cry
Halls ten cardinal signs of Down Syndrome in neonates
Poor moro reflex
Hypotonia
Flat facial profile
Upward-slanting palpebral fissures
Simple, small round ears
Redundant loose neck skin
Single palmar crease
Hyperextensible large joints
Pelvis radiograph abnormal
Hypoplasia of fifth-finger

Fq
85
80
90
80
60
80
45
80
70
60

52

Together in Delivering Excellence (T.I.D.E)

PSYCHIATRIC

Notes

53

Together in Delivering Excellence (T.I.D.E)


MDD [Amalina]
HISTORY

F>M, average age onset is 40y/o


50% pt have diurnal variation: severe in morning, less in evening

PHYSICAL EXAMINATION
General :
Generalized psychomotor retardation.
Hand wringing, hair pulling agitation.
Stooped posture, no spontaneous movement, averted gaze.
Mood, affect, feeling :
May deny depression, but family may complain withdrawal
and reduce in xtvt
Speech:
Reduce rate and volume of speech, respond with single
words and delay response to Qs.
Perceptual disturbance:
May have psychotic features (delusion/hallucination)
If mood incongruent, i.e grandiose + depression, should
consider schizo disorder
Thought :
Negative views, guilt, suicidal

Conditions screening
Substance abuse causing depressed mood (eg. drugs, alcohol, medications)
Medical illness causing depressed mood
Other psychiatric d/o: mania, hypomania, bipolar, schizoaffective,
schizophrenia, etc.
Bereavement unless sx persist for >2 months or show marked functional
impairment, morbid preoccupation with worthlessness, suicidal ideation,
psychotic symptoms, or psychomotor retardation.
INVESTIGATION
Laboratory

FBC, LFT/RFT

Adrenal and thyroid function esp in over/underweight

HIV test if homosex, IVDU or high risk pt

Urine toxicology TRO substance abuse

ECG before ECT

DDx :

Medical d/o mononucleosis esp in adolescent, SLE,


adrenal/thyroid disorder

Neurological d/o Parkinson, dementing illness


(Alzheimer), epilepsy, CVA, tumour

MANAGEMENT

Pharmacotx
SSRI safer, mild
S/E : headache, GI disturbance, sex dysfx, rebound
anxiety
Ex: Citalopram, Fluoxetine
TCA lethal if overdose.
S/E: sedation, weight gain, otostatic hypotension
MAOI for refractory depression.
S/E: otostatic hypotension. hypertensive crisis if w
sympatomimetics, serotonin syndrome if with SSRI.
Hospitalization:
Risk of suicide, have reduce ability to get
food/shelter, need for diagnostic procedure, rapid
progressive symptom
Psychotx:
Behavior tx, cognitive tx, supportive psychotx,
family tx, dynamic psychotx
ECT:
If pt unresponsive or unable to tolerate
pharmacotx, want rapid reduction of sx
Premed atropine, then GA then muscle relaxant
then induce generalized seizure
S/E : Retrograde amnesia

54

Together in Delivering Excellence (T.I.D.E)


GOOD prognostic indicators:

Mild episodes

Shorter hospital stay

Absence of psychotic symptom

Stable family

Absence of comorbid psychiatric disorder

Advanced age of onset


POOR prognostic indicator :

Coexisting dysthymic disorder

Alcohol and other substance abuse

Anxiety disorder symptom

H/o more than one previous depressive episode

Men generally more chronically impaired than women


Treatment:
Dose of antidepressant should be raise to max recommended level and maintained for at least 4-5 weeks before consider in
unsuccessful. But, if pt improve on low dose, dont raise unless improvement stop. When pt does not respond to appropriate dose of drug
after 2-3 weeks, clinicians may obtain plasma concentration of the drug to check for compliance or pharmacokinetic disposition.
Serotonin syndrome:
A potentially life-threatening condition resulting from increased CNS serotonergic activity that is usually drug related. Symptoms may
include mental status changes, hyperthermia, and autonomic and neuromuscular hyperactivity. Diagnosis is clinical (HUNTER criteria).
Treatment is supportive, stop drug and serotonin antagonist cyproheptadine. In most cases, serotonin syndrome manifests within 24 h, and
most occur within 6h, of a change in dose or initiation of a drug. Manifestations can range widely in severity. They can be grouped into the
following categories:

Mental status alterations:


o Anxiety, agitation and restlessness, easy startling, delirium

Autonomic hyperactivity:
o Tachycardia, hypertension, hyperthermia, diaphoresis, shivering, vomiting, diarrhea

Neuromuscular hyperactivity:
o Tremor, muscle hypertonia or rigidity, myoclonus, hyperreflexia, clonus (including ocular clonus), extensor plantar
responses
o May be more pronounced in the lower than the upper extremities

Symptoms usually resolve in 24 hr, but symptoms may last longer after use of drugs that have a long half-life or active metabolites (eg,
monoamine oxidase inhibitors, SSRIs).
MDD Subtype:
Melancholic

Anhedonia, early morning awakening, psychomotor disturbance, excessive guilt, anorexia.


Antidepressant with dual action on serotonergic and noradrenergic receptor

Atypical

Hypersomnia, hyperphagia, reactive mood, leaden paralysis, hypersensitivity to interpersonal rejection


Effective for MAOI, SSRI, Bupropion

Catatonic

Catalepsy, purposeless motor activity, extreme negativism or mutism, bizarre posture, echolalia

Psychotic

Presence of hallucination or delusion


Antidepressant + antipsychotic + ECT

55

Together in Delivering Excellence (T.I.D.E)


Schizophrenia [Hamizah]
HISTORY
Epidemiology:

Man 20 y.o

Woman 30 y.o

High prevalence in lower socioeconomic grp

Strong association with genetic and substance use


Risk factor:
1. Predisposing factor: personality, genetic, environment
2. Precipitating factor: stress, drug
3. Perpuating factor: ongoing stress
Criteria of Schizophrenia (DSM V)
A.

B.
C.
D.
E.
F.

Two (or more) of the following, each present for a


significant portion of time during a 1-month period (or less
if successfully treated).
At least one of these should include 13
1. Delusion
2. Hallucination
3. Disorganized speech
4. Grossly disorganized or catatonic behavior
5. Negative symptoms (i.e., diminished emotional
expression or avolition)
Social/occupational dysfunction
Duration of 6 months
Schizoaffective and mood disorder exclusion
Substance/general mood condition exclusion
If there is a history of autism spectrum disorder or other
communication disorder of childhood onset, the additional
diagnosis of schizophrenia is made only if prominent
delusions or hallucinations are also present for at least 1
month (or less if successfully treated).(Global
Developmental Delay or Autism Spectrum Disorder)

PHYSICAL EXAMINATION
MSE
Appearance: Disorganized appearance
Speech: Disorganized speech
Mood/Affect: Flattened affect
Thought: Disorganized thought process
-

Loosening of association

Thought block

Thought content: abnormal, Delusion

2.

Blood: FBC, TFT, LFT/RFT (for baseline)

Urine toxicology - to exclude substance

Radiology: CXR

ECG (need to before doing ECT)

Assess support: family, friend, financial, spiritual, belief


system

Grandiose

Thought form: Illogical

Insight: Impaired
MANAGEMENT
1.
Typical

Atypical

pts consent)
Review old notes/medical report from prev hosp admission

judgement

Verify hx from family members and related people (with

Persecutory (Paranoid)

Judgement: Impaired personal and social judgement, normal test

Psychosocial + Spiritual

Cognitive: Intact memory and orientation, impaired attention

Biological

Reference

Perception: Distorted (Auditory hallucination)

INVESTIGATION

1.

2.

Pharmacological : Antipsychotic
Haloperidol, Flupentixol
MOA: block D2 receptors in limbic and neocortex
reduce dopamine overactivity
S/E:
(cortex, limbic) drowsiness, seizure, lethargy
(basal ganglia/ EPS) parkinsonism, dystonia,
akithisia
(pituitary gland) hyperprolactinaemia,
gynaecomastia, galacctorhea, weight gain
Risperidone, Olanzapine
MOA: block serotonin receptor & D2 receptor
reduce dopamine and serotonin overactivity
S/E:
no EPS
Anti histamine effect eg: allergic, sedation
Psychosocial + Spiritual
a. Psychoeducation
b. Behavioural therapy
c. Family therapy
d. Group therapy

56

Together in Delivering Excellence (T.I.D.E)


1.

What are the positive and negative symptoms of Schizophrenia


Positive symptoms
Negative symptoms (5A)
Delusion
Affective flattening
Hallucination
Alogia
Disorganized behavior
Avolition
Disorganized speech
Anhedonia
Attention lacking

2.

How to differentiate between hallucination and pseudo-hallucination ( COINS )


o Conscious
o Objective space
o Involuntary
o No stimulus
o Sense organ

3.

Schizophrenia spectrum
o Schizophrenia
o Schizophreniform
o Brief psychotic
o Delusional disorder
o Schizoaffective

4.

Course of Schizophrenia
Prodromal
Social withdrawn
Psychotic
Perceptual disturbance, delusion, disordered thought process/content
Residual
Flat affect, social withdrawal, odd thinking/behavior

5.

Differential Diagnosis
o
o
o
o
o
o
o
o
o

6.

7.

Medical and neurological disorder


Schizophreniform disorder
Brief psychotic disorder
Mood disorder
Schizoaffective disorder
Psychotic disorder not otherwise specified
Delutional disorder
Fastitious disorder and malingering
Mental retardation

Prognosis in Schizophrenia
Good prognosis
Late onset
Obvious precipitating factors
Acute onset
Good premorbid social sexual and work hx
Mood disorder symptoms
Married
Good support system
Positive symptoms
Female Sex
Family history of mood disorder and family
history of schizophrenia

Bad prognosis
Early onset
No precipitating factors
Insidious onset
Poor premorbid social sexual and work hx
Withdrawn, autistic behaviour
Single, divorced, widowed
Good support system
Negative symptoms
Female Sex
Neurological sign and symptoms
History of perinatal trauma
No remission in 3 years
Many relapse
Hx of assaultiveness

Hallucination vs Illusion
Hallucination:sensory perception without actual external stimulus eg: auditory, visual, olfactory and tactile

Illusion: Misinterpretation of an existing sensory stimulus

57

Together in Delivering Excellence (T.I.D.E)


Anxiety Disorder: Panic Disorder + OCD [Julea]
Epidemiology:

F>M & more frequently in higher socioeconomic groups


Risk factor/causes:

Genetic

Environmental

Biological

Psychosocial fx

Neurotransmitter imbalances(inc NE, dec GABA and


serotonin)
PANIC DISORDER
Panic D with agoraphobia- fear being alone in public place
Panic D w/out agoraphobia- specific phobia @ social phobia
Panic D:

Recurrent panic attack w/out precipitant with persistent


fear of having additional attacks

Intense fear + 4 :
-Palpitation
-Sweating
-Shaking
-SOB
-Chest pain
-Choking sensation
-Nausea
-Light headedness
-Depersonalization
-Fear of losing control
-Fear of dying
-Numbness or tingling
-Chills
-Hot flushes
st

1 attack may follow period of stress or physical exertion,


subsequent attack occur spontaneously

A/w major depression, substance dependence, social and


specific phobia, OCD?
OCD
Obsession: Recurrent & persistent intrusive thought causing marked
anxiety & pt attempt to suppress the thought
Compulsion: Repetitive behavior in response to obsession, when
performed is to relieve anxiety caused by obsession
eg Obsessive-Compulsive thoughts:

Fear of being contaminated by germs or dirt

Spending a lot of time washing or cleaning

Excessive focus on religious or moral ideas

Praying excessively or engaging in rituals triggered by


religious fear.

Fear of losing or not having things that might need

Accumulating junk such as old newspapers or empty food


containers.

Order and symmetry: the idea that everything must line up

Ordering or arranging things

DDx Panic Disorder:


Rule out other condition before Dx panic D

Medical conditions: CCF, MI, angina, thyrotoxicosis,


temporal lobe epilepsy, multiple sclerosis, COPD

Mental: depressive D, phobia D, OCD, PTSD

Drug: amphetamine, caffeine, nicotine, cocaine, alcohol or


opiate withdrawal

Mx for Panic Disorder:


Pharmacological:
Acute:

Benzodiazepine and SSRI


(benzo short course only then tapered dose when SSRI is
instituted because benzo can cause dependence)
Maintenance:

SSRI (paroxetine and sertraline)


Non pharmaco:

Relaxation training

Biofeedback

Cognitive therapy

Insight orientated psychotherapy

Family therapy
Mx for specific phobia:

Systemic desensitization

Psychotherapy
Mx for social phobia:

SSRI(paroxetine)

Cognitive and behavioural therapy

Management for OCD:


Pharmacological:

SSRI
Non pharmaco:

Behavioural therapy (exposure & response prevention)

58

Together in Delivering Excellence (T.I.D.E)

SURGERY
Notes

59

Together in Delivering Excellence (T.I.D.E)


Hernia & Scrotal Swelling [Amy]
HISTORY
Age : young : indirect ,old : direct
Gender
Male : inguinoscrotal
Female : femoral
RFx :

increased inta-abd pressure (chr cough, chr


constipation, BPH)

Weak abd wall (heavy-lifting, grandmultipara, hx of abd


surgery (eg, appendisectomy))
Onset, progression, painful/less, reducible/not,
Pain may felt aching/dragging sensation, groin to testis esp
while doing exercise
Complication

strangulated/obstructed

previously painless->painful

reducible ->non-reducible,
I/O symptom NBO, abd distention/pain

Diagnosis : side (R/L/bilateral), type (direct/indirect)


(Inguinal/Inguionoscrotal) hernia, (obstructed/non)
Differential Diagnosis:
1. Inguino/Inguinoscrotal hernia
2. Femoral hernia
3. Inguinal LN
4. Hydrocele (boys)/canal of Nuck(girls)
5. Saphenous varix
6. Undescended testis
7. Lipoma of the cord

PHYSICAL EXAMINATION
Standing
o Inspection : Lump site above or below inguinal lig,
uni/bilateral, position, enter scrotum, size, skin changes,
previous scar
o Cough impulse
o Palpate : consistency soft/fluctuant, size, shape, temperature,
tenderness, can get above lump, can feel testis,
o ask pt to cough + feel for palpable cough impulse
o ask pt to reduce lump reducible/incarcerated
Lie supine
o Deep ring occlusion test
1. Reduce the hernia
2. Locate deep inguinal ring
Find pubic tubercle : umbilicus down to pubic symphysis,
st
lateral, then 1 bony prominence
Find asis
Occlude at midpoint of inguinal lig (2cm above)
3. Keep pressure on ring and ask pt to stand up & cough
4. Remained reduced indirect hernia, bulge out direct
Percussion & Auscultation ascites, BS
**scrotal swelling translumination test
Offer:
1.
2.
3.

Abd exam : scars, masses, ascites, constipation, IO


DRE : BPH, impacted stool
Respi : COPD

**Scrotal swelling :
1. Hydrocele 1 or 2
2. Testicular Tumour
3. Epididymal cyst
4. Varicocele
INVESTIGATION

1.
2.

Abdominal Xray IO
U/S scrotum

Testicular Tumour
Age : young male (25-35 y/o)
RFx : undescended testis, trauma
Types : Seminoma, Teratoma, Combined, Instestitial
Spread :
Direct epididymis, spermatic cord
Lymphatic para-aortic node, Virchows node
Blood stream lungs, bones, liver, brain
Presentation : painless heaviness lump, dull ache
Examination : enlarged testis, mass inseperable from testis,
hard, irregular, nodular(teratoma)/
smooth(seminoma), non-tender, not
transilluminateable, can get above mass
**may present with 2 hydrocele
Staging : 1 : testis only, 2 : modes below diaphragm, 3 : nodes
above diaphragm, 4: lung or liver mets
Tumour marker : LDH, AFP, hCG, LDH
Management : orchidectomy + radiotherapy (seminoma) or
chemotherapy(teratoma cisplastin, methotrexate,
bleomycin, vincristine)

MANAGEMENT
Non-surgical
o Reduced intaabdominal pressure weight loss, change job, tx
medical condition chr cough, chr constipation
o Truss : for compression of reducible hernia at deep ring
o If obstructed/strangulated : NBM, IV drip, NG tube suction, IV
ABx
Surgical
o Herniotomy/Herniorapphy- indirect hernia
o Hernioplasty direct hernia eg. Lichtenstein
**Complication of surgery
A. Intra-op : injury to surrounding structure : blood vessels
femoral, inf epigastric artery, bowel or bladder, nerves
ilioinguinal, cord
B. Immediate post-op : AUR, haematoma, infection
C. Late cx : Recurrence, testicular atrophy, iatrogenic ascend
testis

60

Together in Delivering Excellence (T.I.D.E)


1.

Boundaries of inguinal canal 4cm oblique canal about 2cm above inguinal ligament

Roof : Internal oblique, tranversis abdominis ms


Anterior : Aponeurosis of external oblique
Floor : Inguinal lig, lacunar lig
Posterior : Transversalis fascia
Content : female ilioinguinal nerve, round ligament of uterus
male ilioiguinal nerve, spermatic cord
3 arteries : testicular a., artey to vas deferens, cresmater a. ;
3 nerve : nerve to cremaster, autonomic n. ;
3 structures : vas deferens, lymphatics, pampiform venous plexus

2.

Mid Inguinal Point mid point between ASIS and pubic symphysis
Midpoint of Inguinal Ligament mid point between ASIS and pubic tubercle

3.

Hesselbach Triangle

Course
Etiology
Neck
Reduction
Deep ring
occlusion
test

Indirect
Via deep inguinal ring and along the inguinal
canal, enter scrotum
Patent processus vaginalis
Lateral to inf epigastric vessels
Narrow, increase risk to strangulate
Manually
Do not bulge out

Direct
Via transversalis fascia (Hesselbachs triangle)
dont enter scrotum
Weak abdominal wall/muscle
Medial to inf epigastric vessels
Broad
Automatically
Bulge out

61

Together in Delivering Excellence (T.I.D.E)


Breast Lump [Nur Nadia]
HISTORY

*Hx of lump:
site, single/multiple, when/why was it first noticed,
painful/painless, overlying skin changes, any increase in size,
any nipple changes, nipple discharge, any lump elsewhere.
*Estrogen exposure hx:
i) Age of menarche
ii) Age of menopause
iii) Use of hrt> 1 year

PHYSICAL EXAMINATION

*Check list:
1. Breast
2. Nipple
3. Supraclavicular lymph nodes
4. Spinal tap
5. Lung effusion
6. Hepatomegaly

*Any family history of cancer; breast, ovarian, prostate, CRC


*Systemic review:
LOA, LOW, fever, bone pain, SOB, headache.
*Differential diagnosis:
A. Painless:
carcinoma(elderly, post-menopausal), cyst, fibroadenoma
B. Painful:
cyst, abscess, fat necrosis, periductal mastitis, galactocele,
carcinoma (rare; 10% & advanced)
INVESTIGATION
*Triple assessment:
i)
ii)

Clinical examination
Imaging:
a. Mammography = neodensity/ asymmetric density,
microcalcifixation < 0.5mm, stellate lesion with poor
outline/ comet sign, architectural distortion
b.

Ultrasound = hypoechoeic with/without thick


echogenic halo, irregular edges, high central
vascularity, microcalcification

c.

Breast MRI = useful for pre-menopausal women with


dense breast

iii) Histology:
FNAC
Core biopsy (Trucut/mammotome)
Incisional biopsy
Excisional biopsy

MANAGEMENT
*If triple assessments suggest a benign lump, follow-up with
physical examination for 1 year to ensure the lump is stable
and regress.
*If all 3 concordant of malignancy- further staging and
treatment.
*If 1 or 2 of 3 suggest of malignancy- further workup
excisional biopsy?

62

Together in Delivering Excellence (T.I.D.E)


Abdominal Mass & Pain [Julea]
HISTORY

Age gender
Mass (LORDSANFARO)
location, onset, timing, progression of the mass

Associated symptoms?
Pain (location,nature,radiation,duration, agg and
relieving fx)
Solid organs: dull & constant pain
Hollow viscera: colicky pain
Nausea/vomiting
Change of bowel or urinary habit
Flatus/gas
Features of jaundice
Constitutional symptoms e.g. weight loss, sweats,
fever, anorexia, pallor
Any blood in the faeces or urine?
Lower urinary tract symptoms

PHYSICAL EXAMINATION
General

Pallor
Jaundice
Cyanosis: cirrhosis liver with portal hpt
Clubbing: cirrhosis, uc, crohns
Lymphadenopathy

Specific
GIT examination:
Inspection: distension, mass ,umbilicus, scar,
pulsations
Palpation: guarding, rigidity, rebound tenderness,
mass, organomegaly
Percussion: shifting dullness, fluid thrill
Auscultation: bruit, bowel sound hyperactive,
sluggish, absent, normal

Others: PR, genitalia, hernia orifices

PMH any abdominal procedures-adhesion

INVESTIGATION
(Based on ddx)

MANAGEMENT
Depends on diagnosis.

Laboratory:
1. FBC: anemia, hyperleukocytosis
2. BUSE
3. LFT
Imaging:
1. USG abdomen + pelvis
2. CT abdomen (definitive)
3. Abdominal xray air fluid level, distended bowel,
stones

If abd distension + pain:


pain Mx: NSAIDS opioids, antispasmodic
symptomatic: drugs, ryles tube
antibiotics
surgery

63

Together in Delivering Excellence (T.I.D.E)


Abdominal mass:

64

Together in Delivering Excellence (T.I.D.E)


Upper GI Bleed [Amal]
HISTORY

Nature of bleed and amount


Hematemesis : fresh/coffee ground
Malena : fresh/stale/iron stool
(gloved finger in water and stirred
Malaena black
iron green
Causes
ULCERS : epigastric pain may radiate to back
(dyspepsia/heartburn) Drugs used? Coffe ground vomit?
Melena
VARICES : CLD symptoms? (easy bruising, N&V,LOW,
jaundice, dark urine, pale stools, pruritus, impotence) h/o
variceal bleedn OGDS?melena? Alcohol? Blood transfusion?
High risk behaviour? Hepatitis?
CANCER : early satiety? Dyspepsia? LOA LOW?
Bloating?Distension?

PHYSICAL EXAMINATION

General: ill looking, mental status, respi distress


Vitals : BP/HR/postural HPT, urine output
Peripheral : pallor, CLD stigmata, clammy palm (shock)
Abdomen : distension w ascites or organomegaly, dilated veins,
c.medusae, palpable liver/spleen, +fluid thrill/shift dull, audible
hum
DRE
Lymph nodes: gastric ca (Troisiers sign, Virchows : Lt
supraclvicular fossa )
Trousseau's sign : thrombophlebitis and DVT (nonmets effect of
malignancy esp gastric and pancreatic ca)

:
1.
2.
3.
4.
5.
6.

Complications
Anemia sx? perforated ulcers sx (board like rigidity, abdomen
xmove w respiration) shock?

Variceal bleed (portal HPT)


Bleeding PUD
Acute erosive gastritis
Stomach/esophagus ca
Malloryweiss tear
Bleeding d/o

Comorbidity : elderly, liver/renal dse, IHD, blood d/o


INVESTIGATION

Laboratory :
1. FBC Hb for anaemia. Pancytopenia in hypersplenism
2. BUSE + creat - urea & creat can be high in GI bleed
3. RFT/LFT tro hepatorenal syndrome(in CLD) and bilirubin and
albumin for Childs score
4. Coagulation profile coagulopathy in liver dse, also for Childs
score
5. Urease test H.pylori for PUD
6. GXM
7. Tumour marker alphafetoprotein (liver)
CA 19.9 (gastric)
Imaging :
1. OGDS
Diagnostic - to visualise source of bleeding, biopsy, CLO
test
Therapeutic- ligation, sclerotherapy, clip, etc
2. Barium meal : initial test for vague sx
Features gastric ca - Mucosal irregularity and ulceration,
apple-core, filling defect , Pseudoachalasia, Linitis Plastica
Gastric ulcers - filling defect (bulls eye), radiating folds,
ulcer collar (Hamptons line)
3. CT: for staging in ca, look for perforation in ulcer
4. CXR : air under diaphgram (perf)
5. CT TAP, EUS : mets and staging

MANAGEMENT
1)
2)
3)
4)
5)

Clear airway to pvt aspiration - ETT


Resuscitation (2 large bore, O2, fluid, CVP)
Blood for ix, catheter for i/o, gastric lavage bfore OGDS,
Sengstaken if suspect variceal
Early meds PPI (omeprazole), abx but not in ulcer, stop
offending drugs
OGDS- in 12H if high risk 24H low risk

Pharmacology

PUD
Gastroprotect (PPI, H2 antag,antimuscarinic)
Neutralization acid ( antacids )
Mucosal protect (Bismuth, sucralfate)
H.pylori eradication : MOA

Variceal

Vasopressin + GTN : to reduce portal pressure


Octreotide : to preventt re-bleed
Cefotaxime, ceftriaxone : prophlxs abx
PPI and beta blocker

Surgical

PUD partial gastrotectomy, vagotomy, highly selective


vagotomy

Variceal TIPPS, splenorenal, portocaval


Cancer
Sub/total gastrotomy
Chemo ( 5FU, cisplatin) and radiotx in bone mets

65

Together in Delivering Excellence (T.I.D.E)


Common/Possible Question in Exam + Answer

1. When we decide to transfuse blood?


Sbp<110mmhg, postural hypotension
Pulse>110/min
Hb < 8g/dl
Angina or cvs disease with hb < 10g/dl
2. PUD
1. 2 aggrasive factor ( pepsin and gastric xtvt)
2. 5 protective (pg, high mucosal blood flow to remove proton, mucus, bicarb, epitheliat regenration)
3. Forrest classification to asses risk of rebleeding
4. 3 major complications of pud are haemorrhage, perforation and gastric outlet obstruction
5. Complication of surgery in pud : recurrence, small stomach syndrome, early/late dumping, bile vomit,
malignant transformation, gallstone, nutrition def (b12, calcium and low)
6. Discharge patient with ppi, f/up to rescope in 6 weeks to look for heal or malignant transform, h.pylori
eradication
3. GASTRIC CA
1. Most common adenocarcinoma 90%. Other 10% : scc, gist, leiomyosarcoma
2. Stomach layers : serosa, musc propria ( longitudinal, circular, oblique), submucosa, mucosa
3. Staging
4. Complication gastrotomy :
early bleed, infection, anastomose leak, injury to surrounding esp pancreas
Late early satiety, intestinal hurry, dumping syndr, biliary reflus to stomach, afferent limb
syndrome, ntrient def
4. VARICEAL BLEED
1. Sites of portosystemic shunt : lower end esophagus, upper end anal canal, bare area of liver, paraumbilical
region, retroperitoneal area.
2. Sengstaken Blakemore : 3 tubes (gastric ballon, esophageal ballon, gastric aspiration). Keep in fridge, deflate
6 hourly to prevent pressure necrosis, use max 48H
3. Complications: aspiration pneumonia, resp obstruction, esophageal ulcer and rupture, rebleed, uncontrolled
varices
4. Contraindication : basal skull #, esophageal tear, severe facial injury
5. TIPPS : for Child B C. after fail medical and OGDS. Via IJV and SVC
6. Complication TIPPS : encephalopathy , liver capsule perforation, shunt stenosis
7. Surgery : for Child A, pt on long term beta blocker, chronic banding and sclerotherapy
8. CHILDS PUGH score

66

Together in Delivering Excellence (T.I.D.E)


Lower Git Bleed [Miza]
HISTORY
1.

Nature of bleeding
- Mixed (higher colon) or coating the stool (haemorrhoid)
- Torrential or drops or clot?
- Malaena (come from upper GI)
- Haematochezia, bright color (lower) vs darker color (higher)
- Any mucus?

2.

Aetiology clues (to rule out)

UGIB

Diverticulosis

Colorectal CA

Colitis

Hemorrhoid

Coagulopathy

3.

Any malena, hematemesis, coffee ground vomitus


Hx of PUD, gastritis, varices, cancer stomach, Mallory
Weiss tear
Usually torrential bleedingthat stop spontaneously,
altered clot
Hx of prev bleeding episodes
Constitutional symptoms: LOW, LOA, fatigue
Change in bowel habit tenesmus
Symptom of anaemia
Prev hx/ family hx of GIT or ovarian cancer
Infective: any fever, night sweat, nausea vomiting,
diarrhea, abd pain, recent travel/ contact hx, eating
seafood, prev TB exposure, BCG vaccine
Inflammatory: any hx of CD or UC, joint, liver, eye and skin
manisfestation
Iscahemic: artherosclerosis, risk factor, prev MI, stroke
Bleeding to passing motion
Any mass noticed at anus
Hx of constipation, hard stool, low fibre diet, chronic
straing, recent pregnant
Any hx of bleeding disorder
Easily bleeding, petechiae

PHYSICAL EXAMINATION

General
Pallor, confusion
Vital sign: HR. RR, BP, Temp
Sign of dehydration
Supraclavicular lymph node
Skin manifestation of IBD
Stigmata of CLD
Abdomen:
Scar
Any palpable mass to r/o Colorectal CA. if mass palpable
describe the mass- location, size, shape, consistency, margin,
mobility, tenderness, attached to underlying structure or skin,
surrounding skin
DRE
-

Proctoscopy: To look for internal haemorrhoid

MANAGEMENT
1.

Resuscitation (RAINBOVV)
a. Give O2, insert 2 large bore IV cannulae with fast
infusion of srytalloid (NS 500ml over hour)
b. Infuse colloids if ongoing blood loss while waiting for
the whole blood (GXM)
c. Catheterize and monitor urine output, insert NGT on
suction to detect UGIB
d. Continuous vital sign monitoring & I/O chart, CVP, stool
chart
e. ECG + cardiac enzymes to detect MI
f. Take blood for investigation (see next column )

2.

Identify source and stop bleeding. Arrange urgent colonoscopy.


Diagnostic: identify cancer, diverticulosis, angiodysplasia, areas of
inflammation or bleeding
Therapeutic: clip, diathermize, or inject adrenaline/sclerosant
into bleeding vessel.

3.

Treat according to cause:

Complications
- Anaemia symptoms: SOB, postural giddiness, palpitation,
chest pain, decrease effort tolerance, fatigue, syncope
- Symptom of dehydration & shock: extreme thirst, confusion,
pallor, decrease urine output

NVESTIGATION
Laboratory
1. FBC: to look for sign of anemia
2. BUSE: look for hydration status, any ARF from shock, electrolyte
imbalance- replace
3. Coagulation profile : PT/PTT to rule out coagulopathy and correct
it present
4. LFT to r/o bleeding varices
5. ABG may have metabolic acidosis in shock due to organ ischaemia
IV bicarb, dialysis if severe
6. GXM - 4 pint, to check for blood compatibility
7. ECG for anesthesia assessment
Imaging
1.
2.

Erect and supine AXR: to look for intestinal obstructionceacel


distention, near perforation
Erect CXR: in perforated tumor to detect air under diaphgram

Mass, blood, anal fissure, haemorrhoid

UGIB
Diverticulosis
Colorectal CA
Colitis
Haemorrhoid
Coagulopathy

67

Together in Delivering Excellence (T.I.D.E)


Common/Possible Question in Exam + Answer
1.

Define Upper GI and lower GI


Upper GI start from mouth until above ligament of Treits ( duodeno-jejunal junction)
Lower GI start from below ligament of Treits until anus.

2.

Indication of colonoscopy
Diagnostic
Hematemesis
Dyspepsia (<55 y.o)
Gastric biopsy
Duodenal biopsy
Persistent vomiting
Iron Deficiency Anaemia

Therapeutic
Tx of bleeding lesion
Variceals banding and sclerotherapy
Stricture dilatation
Palliation eg stent insertion, laser therapy
Argon plasma coagulation for suspected vascular abnormality

Indication of upper GI endoscopy


Diagnostic
Rectal bleeding
Iron deficiency anaemia
Persistent diarrhea
Biopsy of lesion seen on barium enema
Assessment or suspicion of IBD
Colon cancer survellance

Therapeutic
Polypectomy
Angiodysplasia
Decompression
Pseudo-obstruction
Volvolus

3.

4.

Staging of Colon cancer (American Joint Committee on Cancer)

5.

Differences between Cohn Disease and Ulcerative colitis

6.

Stage of haemorrhoid

68

Together in Delivering Excellence (T.I.D.E)


Appendicular Mass [Rozana]
HISTORY
Appendicular mass = A mass of inflamed tissue located in the
right lower quadrant, which surrounds an acutely inflammed
and/or ruptured appendix with appendicitis.

Common complication of acute appendicitis

Mass consists of greater omentum with oedematous


caecal wall & loops of distal small intestine with
inflamed appendix in centre natural phenomenon
to contain spread of infection

Sx : RIF , pain, anorexia, constipation

PHYSICAL EXAMINATION
General : tachycardia, feverish
Specific :
Tender irregular mass RIF beneath some rigidity of
the overlying musculature
Other sign : rovsing (pain > in RIF than LIF when LIF is
pressed), psoas ( pain on extending hip if retrocaecal
appendix), cope (pain on flexion and internal rotation
of right hip if appendix in close relation to obturator
internus)

INVESTIGATION
Laboratory
FBC : neutrophil leukocytosis
elevated CRP

MANAGEMENT
Appendix mass + pts condition satisfactory conservative =
Ochsner Sherren regime (nature has already localized the
lesion + surgery at this time is difficult)

Imaging
U/S helpful in assessing the nature & size of mass
Laparotomy/colonoscopy : exclude colonic tumor

A: Analgesic, Antibiotic (metronidaole + cefuroxime),


antipyretic
B: Bed rest
C: Charting (vital sign, size of the mass)
D: Diet (Keep Nil by Mouth)
E: Exploratory laparotomy KIV.(only in suspicion of
abscess)
F: Fluid maintenance (to correct dehydration)
Interval appendicectomy should be performed 3
months later to prevent recurrence

If pt p/w lower abdominal mass and demonstrates feature


of generalized peritonitis +/- bowel obstruction, operative
intervention indicated

However early surgery is now regards as a preferred


method. Laparoscopic appendicectomy is safe and should
be a preferred method compared to open surgery even
though in emergency setting.

69

Together in Delivering Excellence (T.I.D.E)


Obstructive Jaundice [Hidayah]
HISTORY

PHYSICAL EXAMINATION

1.

Obstructive jaundice : yellow sclera, tea colored urine, pale


stool

1.
2.

2.

Exclude pre&hepatic jaundice :


sx viral hepatitis (prodrome fever, malaise, arthralgia,
myalgia, N&V)
risk factor viral hepatits (travel hx, ingestion of seafood,
fmly hx of hepatitis (esp mother&siblings), blood
transfusion, drug abuse, needlestick injuries, sexual
contact)
Alcohol intake(pancreatitis,cirrhosis)
Drug hx (traditional med). OCP (intrahepatic cholestasis)
statin (hepatotoxicity)
Hx of chronic liver dis.

3.

3.

4.

5.
6.
7.
8.

INVESTIGATION
1.

Benign/Malignant

Vitals: fever,hemodynamically stable


General : Jaundice, pallor, any abdominal
distention,leg swelling
Peripheries : stigmata of CLD,scretch
marks,conjunctival pallor
Abdomen : Ascites, hepatosplenomegaly
(cirrhosis&portal HPT), mass per abdomen, palpable
gallbladder(courvoisiers law)
DRE : pale stool
Lymph node: cervical & supraclavicular LN
Bony tenderness
Respi examination

To confirm obstructive jaundice


LFT : bilirubin raised(+/- direct>indirect normal
3:7), raised ALP
II.
gamma-glutamyltransferase-if dx ambiguous.
I.

Benign
Recurrent spikes of similar
jaundice that resolves on
their own times (suggest
obstruction :
stones,stricture)
Young pt with painful
jaundice
Previous hx of
gallstone/biliary colic sx
Previous biliary surgery/ibd
(sclerosing cholangitis)

Malignancy
Old pt, new onset of
jaundice & progressively
worsening
No associated pain
Constitutional sx
Mets sx (bone pain, neck
lump, dyspnea)
Late pain, constant &
relentless (pancreatic ca)

4.Complication
Sx of cholangitis (fever,chills,rigor,RHC pain,jaundice)
Pruritus
Pancreatitis(gallstone as a cause)
Decompesation (heapatic encephalopathy,fetor
hepaticus, worsening ascites)
Fat malabsorption (steatorrhea,fat soluble vit deficiency
ADEK-coagulopathy.

2.

To confirm inflammation/infection
I.
FBC: Hb, WBC
II.
Renal profile : electrolyte(nutritional status), U:C
ratio
III.
Coagulation profile prolong PT (vit k
malabsorption, liver dysfx)
IV.
Tumor marker : CA 19-9, CEA
(cholangioca&pancreatic ca)
V.
Blood C&S : TRO HBS sepsis (if fever&febrile)
VI.
GXM : in case op

3.
I.

Imaging
HBS US
choledolithiasis - duct dilate>8cm
Gallstone dis/cholecystitis - GB
stone/sludge, thickned GB wall,
pericholecystic fluid, fat stranding
cx gall stone
liver consistency (fatty/cirrhotic)

II.
III.

CTAP : if suspect perforated,TRO ca


CXR : ARDS in cholangitis,p.effusion

MANAGEMENT
According to cause

70

Together in Delivering Excellence (T.I.D.E)


Common/Possible Question in Exam + Answer Notes [any classification,staging, mnemonic etc]
1.

Define jaundice:
Yellowish discoloration of kin, sclera and mucus membrane d/t bilirubin, a yellowish pigment
Usually clinically evident when serum bilirubin >3mg/dl (51.3umol/L)
Normal bilirubin: 3-17 umol/L

2.

Bilirubin transport

3.

Causes
Painless: CA,HOP,cholangioca,Periam ca
painful: Stricture,hepatic causes

Intraluminal
Benign
1.Gallstone
2.Parasitic infection
(schistosomiasis)

Mural
Benign
1. Stricture (post instrumentation &
post inflammation)
2. Primary sclerosing cholangitis
3. Choledochal cyst
Malignant
1. Cholangiocarcinoma

Extramural
Benign
1. Mirizzi syndrome
2. Pancreatitis
Malignant
1. Head of pancreas ca
2. Periampullary ca
3. Mets to porta hepatis

4.

What is Corvoisiers Law


If the gallbladder is enlarged& patient is jaundiced, the cause is unlikely to be gallstone

5.

Whait is Charcots triad


triad of fever, RHC pain &jaundice : suggest cholangitis.

6.

Urine & blood biochemistry in jaundice - Liver enzyme in ix of jaundice


ALP (alkaline phosphatase) & GGT : markers of cholestatis
ALT (alanine aminotransferase) & AST (aspartate aminotransferase) : markers of hepatocellular injury
AST: ALT ratio>1 alcoholic hepatitis (remember ST out)

71

Together in Delivering Excellence (T.I.D.E)


Multi-nodular Goitre (MNG) [Alliah]
HISTORY

Definition
Enlarged and diffusely heterogenous thyroid gland

Epidemiology
6x more common in woman

Check list hx for MNG :


Establish details of the swellingonset,duration,progression,recent growth,pain
Other neck symptoms(local) - discomfort during
swallowing, SOB,pain and hoarseness of voice
General sx - sx of endocrine dysfunction
Past medical hx - hx of radiation to the neck during
childhood
Family hx of autoimmune disease

DDx
Anaplastic carcinoma of thyroid
Autoimmune thyroiditis

PHYSICAL EXAMINATION
General
Moderate body built. Neither nervousness nor
lethargic
Skin - normal (no sweating;not dry)
Hair - normal
Hands no wasting or puffiness
Normal PR with no fine tremor
No eye signs
There may be general signs of thyrotoxicosis or in
elderly pt with very long-standing nodular goitres,
the signs of myxoedema.
Specific
A nodular swelling in front of the neck which moves
upwards on swallowing
Non tender
Smooth surface but assymetrical nodular
Firm in consistency
Neither skin or nerby muscle attachment
No signs of airway obstruction
Cervical LN not palpable
No bruits over the gland

INVESTIGATION
Laboratory
1)

2)

3)

TFT
To exclude mild hyperthyroidism
TSH : normal (0.5-5mu/L)
Free T4 : normal (9-24)
Free T3 : normal (2.2-5.4)
Thyroid antibodies
To exclude autoimmune thyroiditis
Thyroid peroxidase (TPO) antibody up to
25units/ml
Anti-thyroglobulin titres : significant when > 1:100
titres
FNAC
Will only be required for dominant swelling in a
generalised goitre

Imaging
1)
2)

Plain radiographs of chest and thoracic inlet


To exclude tracheal deviation or compression
U/S and CT scan

MANAGEMENT
Pharmaco

Thyroxine
Surgical

Total thyroidectomy and lifelong replacement of


thyroxine

Conventional subtotal thyroidectomy leave up to


8g of relatively normal tissue in each remnant

72

Together in Delivering Excellence (T.I.D.E)


Common/Possible Question in Exam + Answer
1.

Classification of thyroid swelling


Simple goitre (euthyroid)
Diffuse hyperplastic
(physiological,puberty,pregnancy)
MNG

Toxic
Diffuse (Graves Dz)
Multinodular
Toxic adenoma

Neoplastic (Benign & Malignant)


Inflammatory
Autoimmune (Hashimotos dz)
Granulomatous (De Quarvains
thyroiditis)
Fibrosing (Riedels thyroiditis)
Infective (acute and chronic)
Other - amyloid

2.

Complication
1) Tracheal obstruction
Due to gross lateral displacement or compression in lateral or AP plane by retrosternal extension of goitre
2) Secondary thyroitoxicosis (up to 30% pt)
Transient episodes of mild hyperthyroidism
3) Carcinoma (incidence 5-10%)

3.

Indication for surgery


Cannot be treated medically failed or unsuitable for medical tx
Cancer
Compression on neighbouring structures
Cosmetic
Compliance/cost problems with long term medical therapy

4.

Postoperative complication after thyroidectomy (mostly Hs, One Is, One Ts)

IMMEDIATE (< 24hrs)


Haemorrhage with
hematoma formation
Hoarness or airway
compression
Hyperthyroidism
Tracheomalacia

EARLY (< 1m)


Infection
Hypoparathyroidism leading to
hypocalcemia

LATE (> 1m)


Hypothyroidism
Hyperthyroidism
Permanent
hypoparathyroidism
Hypertrophic scarring or
keloid formation

73

Together in Delivering Excellence (T.I.D.E)


Hematuria [Ain]
HISTORY

PHYSICAL EXAMINATION

Definition : >3 rbc/hpf

DDX

1.

2.

Haemoglobinuria
Myoglobinuria
pseudohematuria(menses)
meds causing discolouration (rifampicin,phenytoin)

Which part of urine stream is blood stained?

Initial phase-bleeding from urethra distal to urogenital diaphragm

Throughout urination upper urinary tract or upper bladder

End of urine stream- bladder neck or prostatic urethra

Painful hematuria

Painless hematuria

UTI
Pyelonephritis
Hydronephrosis
Renal cysts
Ureteric stone
BPH,strictures
tumour

Malignancy- RCC, bladder,prostate


Drugs
Glomerulonephritis
ITP/HSP
Bleeding diathesis
Infection-malaria,schistosomiasis
Exercise-joggers hematuria

Severity any clots,s/s of anemia


Ass. symptoms frequency, dysuria, LUTS symptom, fever
DDX : nephrolithiasis, pyelonephritis, UTI, malignancy, bladder
outflow obstruction.
5. Others drug hx, malignancy symptom, recent travelling, sore throat,
medical illness eg: coagulopathy, autoimmune ds
INVESTIGATION

Is it painful or painless?

3.
4.

Urine dipstick, UFEME ,urine cytology for malignant cells,


urine phase contrast, urine C&S
Blood FBC,BUSE,PT/APTT,GXM
Imaging : Upper tract - imaging, lower tract - cystoscopy
Plain KUB : stone, size of kidney
Ultrasounds of kidney :renal size, stones, hydronephrosis,
hydroureters
Intravenous urogram (IVU) C/I in contrast allergy, renal
impairment (cr>200),on metformin cause lactic acidosis,
asthma given steroids 3 days b4 study, pregnancy
CT Urogram/IVP
Cystoscopy : detection of bladder tumour, biopsy can be
taken at the same time
Kidney biopsy suspect glomerular disease

Vital sign: hypotension and tachycardia are seen in


patients who are haemodynamically unstable from
acute blood loss,high temperature in infection
Sign of anemia pallor of conjunctiva, skin
Periorbital, scrotal, and peripheral oedema: may
indicate hypoalbuminaemia from glomerular or renal
disease.
Cachexia: may indicate malignancy.
Abdomen- renal/bladder mass, palpable bladder
(AUR),flank tenderness, suprapubic tenderness
Scrotum varicocele on the left (RCC of left kidney
with extension of tumour to renal vein, block
testicular vein from drained to left renal vein)
External genitalia - blood from urethra, trauma,
laceration
Digital rectal examination prostate enlargement (
BPH vs cancer)
The presence of a urethral catheter, suprapubic
catheter, ureteral stent, or nephrostomy tube may
signify an iatrogenic cause of bleeding that is
generally benign

MANAGEMENT
Depends on disease
Asymptomatic (isolated) hematuria generally
does not require treatment. In conditions
associated with abnormal clinical, laboratory,
or imaging studies, treatment may be
necessary, as appropriate, with the primary
diagnosis.
Surgical intervention may be necessary in
certain anatomical abnormalities, such as
ureteropelvic junction obstruction, tumor, or
significant urolithiasis.
Consultations are required in patients with
urinary tract anomalies and in some patients
with systemic diseases (eg, bleeding disorders,
collagen vascular diseases, sickle cell
nephropathy).
Referral to a urologist is required when clinical
evaluation and workup indicates a tumor, a
structural urogenital abnormality, or an
obstructing calculus.

74

Together in Delivering Excellence (T.I.D.E)


Common/Possible Question in Exam + Answer

Urine dipstick
Positive for hematuria

No blood
cell

urine
microanalysis

blood cell
present
If no proteinuria
(isolated hematuria)
-need to ix for systemic ds
Coagulation studies,complete
blood count,hb electrophoresis

Myoglobinuria or
hemoglobinuria

If proteinuria-suspect
glomerular ds and require
kidney biopsy

If pyuria and bacteriuria obtained urine culture (if


ve,consider interstitial nephritis)

Causes of hematuria :
Pre-renal

Renal

Post-renal

Drugs: analgesics(NSAIDS),
anticoagulants, CPM, OCP
Systemic : sickle cell diasease,
haemophilia, ITP
Metabolic: hypercalciurea, DM
Vascular: AV malformations, renal artery
ds,renal vein thrombosis
Glomerular: post-strep AGN, IgA
nephropathy, lupus nephritis
Interstitial: polycystic kidney disease,
stone, malignancy
Vascular: HSP, Wagener granulomatosis
Infections of ureter, bladder, prostate,
urethra
Cancer of ureter, bladder, prostate,
urethra
Stone

75

Together in Delivering Excellence (T.I.D.E)


Prostate Cancer [Fatin]
HISTORY
o

Epidemiology :
th

4 most common cancer among men

Peak age : 65-75 y.o


Risk factor :

Age

Obesity

FHx

A/w BP
S&S :

Asymptomatic incidental finding on DRE

Obst LUTS (hesitancy, staining on urinate, weak stream,


prolonged micturition, terminal dribbling, incomplete
voiding)

Cx LUTS hematuria, infection, stone, obst uropathy


@renal failure

IPSS (incomplete voiding, frequency, intermittency,


urgency, weak stream, straining, nocturia)
Complication :

Metastatic sx lower back pain (from Batson venous


plexus)

Other place: LN, rectum, bladder, lower ureter

PHYSICAL EXAMINATION

INVESTIGATION
o

Diagnosis :

Transrectal ultrasound (TRUS) with core biopsy (2


specimen)
-histology GLEASON score degree of
differentiation/microscopic appearance

PSA level
<4 ng/ml normal
4-10 ng/ml local ca
<30 ng/ml metastatic

Metastatis

Chest x-ray lung met

MRI prostate and CT scan TAP- staging

Bone scan bone met

TNM staging :
T1 accidental finding
T2 PR finding
2a -1/2 of 1 lobe
2b 1 lobe
2c 2 lobes
T3 extra prostate spread
3a prostate capsule
3b seminal vesicle extension
T4 adjacent structure other than seminal vesicle
4a
4b pelvic wall, abdominal wall
N0 no regional LN
N1 regional LN
MO no distant met
M1a non regional LN met
M1b bone
M1c other sites (liver, brain, lung)

Specific :
DRE hard, irregular, nodular, asymmetric area
BPH smooth, enlarge, symmetrical
Other :
Percuss spine for bone tenderness
How to stage :
Clinical examination palpable tumor T2
TRUS biopsy staging purpose
CT scan abdomen and pelvic tumor extension and
node status

Bone scan bone met

MANAGEMENT
o

Stage 1 &2 (T1,T2) locally invasive

Active surveillance wait and see for pt not for surgery (old
patient / early stage)
3-6 months
DRE & PSA level

Radical prostatectomy remove prostate, seminal vesicle,


prostatic urethra
Cx : incontinence, erectile dysfx

Radiotherapy brachytherapy
Cx : cystitis, urethral stricture, proctitis

Stage 3 (T3 N1) locally advance

Surgery

Radical radiotherapy

Hormonal manipulation
Orchidectomy (remove testes)
LHRH agonist/GnRH agonist e.g Zoladex, Goserelin
(flare response -> castration level)
s/e : HOT FLUSH, GYNAECOMASTIA, LIVER PROB,
SWEATING, LOSS LIBIDO
+ anti- androgen prevent testosterone flare

Stage 4 (T4 Met) advance (palliative treatment)

Hormone therapy
Adrenal suppression ketoconazole
Glucocorticoid dexamethasone, predinisolone

Chemotherapy docetaxel + prednisolone

76

Together in Delivering Excellence (T.I.D.E)


1.

Mc Neal classification (prostate zone)


Central
peripheral carcinoma
transitional BPH

2.

Pathology adenocarcinoma/ glandular ca

3.

Causes of PSA level


prostate ca
BPH >50 g
post ejaculation
urethral injury
UTI
iatrogenic (flexible scope)

4.

Source of testosterone
testes
adrenal gland
fat

5.

Family history of BRCA1, BRCA2, HPC1

6.

Pre-op assmnt of TRUS w biopsy


take consent
bowel prep ducalex/ravin supp 3 days b4 procedure
analgesic on table b4 procedure (lignocaine)
antibiotic 3 days b4, 2 days after
- quinolone penetrate prostate
-metronidazole
after TRUS no ridding
- no heavy lifting
- no SI

7.

Function of prostate
secretes an alkaline fluid at the time of ejaculation.
alkaline fluid helps neutralize the acidic environment of the female vaginal tract, prolonging the lifespan of
sperm and providing better motility

8.

What is PSA (prostate specific antigen)?


protein / tumor marker produced by epithelial cells of the prostate gland

77

Together in Delivering Excellence (T.I.D.E)


Renal Calculi [Dalilah]
HISTORY
1.

RENAL STONES
Asymptomatic unless stone gets lodge in the
pelviureteric junction
May cause hydronephrosis and subsequent infection
Vague flank pain

2.

URETERIC STONES (even small stones can cause severe


symptoms as the ureter is narrow)
Ureteric colic pain (severe intermittent loin to groin
pain )
Hematuria (gross or microscopic )
Irritative symptoms ( frequency, urgency)
Signs of pyonephrosis ( upper urinary tract infection
symptoms ) ; fever + chills + rigors, very ill due to
sepsis

3.

BLADDER STONES
May be asymptomatic
Irritative symptoms ( frequency, urgency)
Hematuria
If infection present ( dysuria, fever )
INVESTIGATION

For diagnosis
1. KUB radiograph
Radioopaque stone ( 90% of renal stones)
False negative; too small, false positive; phlebolith
(round with lucent centre),stools.
Kidney; kidney size , stone
Ureter ; trace the path of ureter, ureteric stones
Bladder; bladder stones
2. Intravenous radiogram
To visualize stone
Can show dilated urinary system secondary to stone
obstruction (hydroureter/hydronephrosis)
3. Ultrasound of kidney/ bladder
Features of stones ; echogenic rim, posterior acoustic
shadowing
For complications
4. Urine test ( dipstick, UFEME, Urine c n s)
-hematuria, pyuria
5.
6.

Intravenous urogram
Dilated urinary system ( hydronehprosis, hydroureter )
MAG-3- renogram
To give differential function of each kidney
Normal ; 50% on each side, out of 100% of both kidney
combined

PHYSICAL EXAMINATION
GENERAL

signs of anaemia ( pale hand, conjunctiva)

very ill, in pain, cachecic, febrile, tachycardia ( in case of


sepsis)
SPECIFIC

no guarding, no rebound ( symptoms are out of proportion


to sign)

tender suprapubic area ( in case of acute urinary retention )

positive renal punch (in case of pylonephritis)

kidney ballootable ( in case of hydronephrosis)

MANAGEMENT
Conservative
1. Stone smaller than 5 mm should be treated conservatively
(60% will passed out ); only treat if they do not pass after 4
to 6 weeks, or got symptoms
2. Treat UTI
3. Treat underlying disease eg; hypercalcemia
4. Diet
-high fluid intake
-low salt diet
-restriction of red meat, dairy product, refined sugar
-increase citrus fruit intake
Surgical
Indications :
S/S ; constant pain
complication

Obstruct urine flow

UTI

Significant bleeding

Kidney size increase

unlikely to resolve with conservative treatment :

Does not pass after one month

Too large to pass spontaneously


Types :
1. PCNL (percutaneous nephrolithotomy) more than 20 mm
(large stone )
2. ESWL (extracorporeal shock wave lithotripsy) 5 to 10 mm
( small stone)
3. Ureterorenoscopy with lithotripsy (URS + LL) Stones along
ureter
4. Cystolitholapaxy bladder stone (for stone crushing)
*renal stone 5 to 10 mm : Either ESWL OR PCNL
*can refer ANDRE SURGICAL NOTES for more info

78

Together in Delivering Excellence (T.I.D.E)


Common/Possible Question in Exam + Answer
1.
2.
3.
4.

What is the type of renal stones?


How renal stones are classified ?
Which stones are radioopaque, which stones are radiolucent ?
Treatment modalities based on size,types, location of stones.. ( when to use PCNL, ESWL)

79

Together in Delivering Excellence (T.I.D.E)

O&G
Notes

80

Together in Delivering Excellence (T.I.D.E)


Pregnancy Induced Hypertension [Hamizah]
HISTORY
Risk factors:
Primigravida, age>40, pregnancy interval>10 years, prior PE in a pregnancy by
same partner,multiple pregnancy, Fly history of PE , obese, renal dz, DM,
essential HPT, autoimmune dz (SLE,antiphospholipid syndrome)
Clinical features
-maternal manifestations:
Presenting complaints: Weight gain & edema, headache, blurred vision,
epigastric/ RUQ pain, SOB, oliguric, hemorrhagic manifestation, per vaginal
bleeding (abruption) fetal mv (assess fetal well being)
Complications:
A. Maternal
Heart: high output, pulmonary edema
Placental abruption (d/t necrosis of distal end spiral arteries)
Hematology: thrombocytopenia
Renal: renal failure, nephrotic syndrome
Liver: subcapsular hemorrhage, infarction, rupture, congestion
Lung: aspiration pneumonia (during fits)
CNS: eclampsia, cerebral hemorrhage, retinal detachment
B. Fetal
IUGR
Oligohydramnios

PHYSICAL EXAMINATION
General examination

Vitals(BP, HR, RR, spo2)

Weight

Pallor, petechiae, edema


Chest examination (crepitations in pulmonary edema)
Abdominal examination
Epigastric or right upper quadrant tenderness
Neurological examination
Hyperreflexia, clonus
Visual field testing
Obs examination
Symphysial fundal height (small for dates?)
Palpate for fetal lie & presentation
Palpate for uterine tenderness (woody hard indicate abruption)
Estimated liquor volume
DopTone for fetal heart sound

INVESTIGATION
Maternal
1. FBC - Anemia(d/t hemolysis in HELLP)
2. Thrombocytopenia
3. Renal profile + serum uric acid
4. Raised serum creatinine & urea
5. Raised serum uric acid level in PE
6. 24 hr urinary total protein
7. Liver function test
8. Raised AST, ALT
9. Indirect hyperbilirubinemia
10. Coagulation profile
11. PT & APTT +/- d-dimers, fibrinogen (for DIVC)
Fetal U/S

Estimated fetal weight, fetal lie & presentation, liquor volume, umbilical
artery Doppler flow
HELLP SYNDROME
-Hemolysis (anemia)
-Elevated liver enzyme (AST/ALT>70iu/L)
-Low platelet (<100)

MANAGEMENT
Antenatal
Identify risk factors & observe BP
PE, urinalysis, BP
Confirm diagnosis:

Mild PIH-outpatient

Severe PIH, PE- admission


Outpatient management
Antenatal clinic visit:
- Every 4 weeks if not on treatment, normal biophysical profile, good fetal
growth
- Every 2 weeks if on treatment
Tests:
Urinalysis (protein)
BP
SFH & liquor volume
BUSE,FBC,serum uric acid
Fetal surveillance: US monthly, FKC
Inpatient/ admission
1. BP 4 hourly
2. SFH & liquor volume
3. Daily OE chart, urine protein
4. FBC, BUSE, serum uric acid
5. LFT, coagulation profile (HELLP)
6. IO chart
7. Fetal surveillance:-FKC, CTG, US
8. AntiHPT agent only use of DBP>100mmHg (aim:90-100mmHg)
9. Dexamethasone if early delivery expected (<34 weeks)
Intrapartum management
i.
BP/ pulse half hourly
ii.
To cont oral antihypertensive treatment
iii.
Strict IO chart
iv.
Adequate analgesia (preferable epidural analgesia)
v.
CTG monitoring
vi.
Shortened 2nd stage-assisted delivery, episiotomy
vii.
NO syntometrine/ ergometrine
viii.
Use syntocinon 10 units
Portpartum management
1. Beware sx of IE & pulmonary edema
2. BP monitoring
3. hourly for at least 2-4 hours before sending to postnatal ward
4. 4 hourly in ward for 24-48 hours before d/c
5. Antihypertensive should be cont & stopped later on postnatal review
6. IO chart
7. Daily urine albumin, PE chart
Anti HPT med
Aim: to keep DBP btw 90-100mmHg
Drugs

MOA

Start dose
(mg/day)

Max dose
(mg/day)

Adverse effect

Methydopa

Centrally
acting

250 TDS

3000

Depression, drowlsy

Labetolol

a&B
blockers

100 TDS

2000

Nifedipine

CCB

15 TDS

60

Hydralazine

vasodilator

25

300

Heart block, IUGR,


hypoglycaemia,
bronchoconstriction
Headache, flushing
Tachycardia,
hypotension

81

Together in Delivering Excellence (T.I.D.E)


Questions:
Definition:

BP of 140/90 mmHg or more taken on 2 occasion at least 4 hour apart; OR


An increase in systolic BP of 30 mmHg or/and diastolic BP of 15 mmHg compared to pre-pregnancy level
Single reader of Diastolic BP more than 110mmHG.

1) GESTATIONAL HYPERTENSION

Is hypertension after 20th week of gestation in a previously normotensive woman


x proteinuria
Condition return to norm within 6 weeks after labour

2) PRE- ECLAMPSIA

Develop after 20 weeks gestation


New onset proteinuria (>0.3g/24 hr urine collection)

3) CHRONIC HYPERTENSION

Presence of hypertension of at least 140/90 mmHg before 20th week of pregnancy or beyond 6 weeks postpartum.
Includes essential & secondary hypertension.

4) CHRONIC HYPERTENSION WITH SUPERIMPOSED PRE-ECLAMPSIA

Development of pre-eclampsia in patient with pre-existing hypertension


Criteria used should include:
worsening of hypertension
proteinuria

5) ECLAMPSIA

criteria for PE met


presence of generalised tonic-clonic seizures which cnt be attributed to other causes

signs & symptoms of IE:

Headache, nausea & vomiting, visual disturbance, RUQ/ epigastric pain, frothy urine, pregressive edema at independent site

82

Together in Delivering Excellence (T.I.D.E)


MANAGEMENT OF ECLAMPSIA
1.
2.
3.
4.
5.
6.
7.
8.
9.
10.
11.

Call for help


Left lateral position
Secure airway, prevent pt injury
Wait for convulsion to abate, if not iv diazepam 10mg bolus
Max oxygenation-mask or intubation
Catheterization for io chart
Neurological examination
Bishop score
Prophylactic anticonvulsant therapy
BP control
Cervix favourable-vaginal delivery ; cervix unfavourable- c-sec

Anticonvulsive therapy
MgSO4 Maintainance dose:
*IV infusion of 1g/hour
* 5ml MgSO4 + 45ml 5%Dextrose sol.
Infuse at 20ml/hour( syringe pump) OR * 10ml MgSO4 in 500ml D5% at 33 dpm (drips)
Duration: continue for 24hours after last fit or after delivery
Monitoring for MgSO4 therapy:
1.Investigations
BUSE,FBC
Serum Ca2+,Mg
Renal function test (urea,uric acid,creatinine)
Coagulation profile
UFEME
ECG
GXM
2. STOP !!! If present signs of Mg toxicity:
(a) RR < 16/min
(b) Urine output < 25ml/hr
(c) patellar reflex absent
(d) Serum Mg > 3.5mmol/L (therapeutic range: 1.7-3.5)
(e) BP < 90/60 mmHg
3. Antidote: Ca gluconate 10%-10ml
Antihypertensive therapy
initiatiate parenterally if BP> 160/110mmHg

83

Together in Delivering Excellence (T.I.D.E)


Placenta Praevia [Alliah]
HISTORY
Definition
Placenta that has implanted into lower segment of uterus after 28wks
of gestation
Risk factor
Multiple pregnancy
Multiparity
Previous C-sec/ERPOC/uterine
surgery
Uterine structural abn.
S&S

Assisted conception
Advanced maternal age (>40)
Previous episode
Smoking

PHYSICAL EXAMINATION
General

Stable and less distress pt condition


May appear pale
BP,PR and pulse volume may fluctuate

Specific

Soft and non-tender abdomen


Fetal readily palpable
Fetal not engaged,abnormal presentation and lie
Usually normal fetal heart

Unprovoked painless PV bleeding


Nature : fresh blood and intermittent
Usually normal fetal movement
Less distressed pt condition

Complications
Maternal
Life-threatening
haemorrhage-shock
Placenta accreta
PPH
Need of c-sec
Infection
DDx

Fetal
Malpresentation
Fetal abn.
IUGR
Premature labor
Fetal hypoxia
Fetal death

Placenta Abruptio
Vasa previa
Local causes

INVESTIGATION
Laboratory
FBC

Hb- to check for anemia


TWBC & Platelet to establish
baseline
* both Hb and platelet can be
affected severely with massive
blood loss (anemia and DIC)

GXM
Coagulation profile (when
indicated)
Renal profile
Imaging
1) Transabdominal & Transvaginal U/S

To locate placenta

To confirm diagnosis

TVS : esp. when locating posterior PP that obscured by overlying


fetus & more accurate
2) Color Doppler U/S

To diagnose in pt high risk of placenta accreta (previous c-sec)

MANAGEMENT
Conservative
Pt with minor and major (no previous bleeding) PP
Consider :
- Distant house-hospital
- Transportation
- Educational level & know how to take care herself
Educate :
- Rest and no heavy work
- Avoid SI
- Avoid abdominal massage
- Immediate come to hosp. If presence of contraction pain
* Pt with major PP (had previous bleeding) : should admit from 34weeks
Acute
Resuscitation (RAINBOVV) +

V/S monitoring hourly

Pad chart monitoring

Assess baby : CTG and US

Give IM Dexamethasone if < 34wks

Check maternal rhesus status


Caesarean Section
Emergency

Gestational age > 36 wks of gestation

Irrespective of gestational age if : profuse bleeding &/or fetal


distress
Elective

At 38 or 39 wks

If :
a) placenta edge <2cm from internal os
b) posterior or thick (> 1cm)
* Minor PP SVD if placenta minimum 2cm away from cervical os

84

Together in Delivering Excellence (T.I.D.E)


1.

Classification
UK
Type
I. Lateral : < 5cm from os
II. Marginal : edge at int.os
Anterior
Posterior
III. Assymetrical cover os
IV. Symmetrical cover os
@ anterior & posterior

Minor

Major

US
Complete : covering os
Partial
: partially cover os
Marginal : edge within 2 cm
from int.os
Low-lying : edge within 25cm from int.os

2.

How can you diagnosed placenta accreta?


Color Doppler U/S (whether there is blood flow to the bladder)
MRI
Grey scale U/S

3.

When we opt for McAfee Regime?


Bleeding is not life-threatening & baby is still premature [expectant mx]
Components :
a) Admit pt to the ward
b) Close observation for any further bleeding
c) Availability of atleast 2 units of gxm
d) Availability of the ot

4.

If now pt is 38 weeks & pt admitted for elective LSCS. How do u prepare this patient?
Consent (operation, blood transfusion & explain future risk of Csec and the need for Csec the next pregnancy, risk
for hysterectomy)
FBC - check for Hb (aim at least 11g/dl)
GXM 4 pints
CBD should be done in OT

5.

Mode of anaesthesia in this patient? GA


Not spinal because spinal can cause hypotension
In PP surgery, there will be lots of blood loss & patient can go further hypotensive
GA - is easy to control the circulation in case the surgery went complicated

85

Together in Delivering Excellence (T.I.D.E)


PPROM (Preterm Pre-Labour Rupture Of Membrane) [Baisyatul]
HISTORY
1.

2.

3.

Presenting complain
Sudden gush of warm fluid vaginally, usually followed
by continuous dribble, colourless fluid
Volume ( soaked sarong/ pad)
Must distinguish from leaking urine ( ask about
frequency, urgency, dysuria) as incontinence or ITI
may present in similar way
Smell ( urine smell, foul smelly, odourless)
Vaginal discharge- UTI
Per-vaginal bleed-( a/w abruption)
Uterine irritability or contractions
Fetal movement may reduce, frequency or strength
also reduced
Risk factor assessment
Prev history of preterm labour/PPROM
Twin pregnancy
Polyhydramnions
Uterine/cervical abnormalities (fibroids)
Recurrent antepartum haemorrhage
Infection ( fever, vaginal discharge)
Smoking
Cocaine abuse
Low BMI mother
Asses complication
Chorioamnionitis ( maternal pyrexia, vaginal
discharge, uterine tenderness )

1.

PHYSICAL EXAMINATION
General examination:
Signs of infection: tachycardia, high temperature,
flushed appearance

2.

Abdominal examination:
Uterus smaller than date due to oligohydramnions
Malpresentation (beware cord prolapse)
Uterine tenderness if chorioamnionitis present

3.

Sterile speculum examination (definitive diagnostic tool)


Preferably after patient has been resting supine for
20-30 minutes
A pool of amniotic fluid in posterior fornix
Visualize the cervix; fluid may be seen trickling
through the external os and dilatation of cervix can
be assessed

Digital vaginal examination should be avoided:


Can stimulate prostaglandin production
Introduce organism into cervical canal

INVESTIGATION
Maternal
Check pool of liquor by:
a) Litmus paper (red to blue)
b) Nitrazine test (turn to black)
c) Ferning test (microscopic examination to look for
fetal epithelial cells)
Full blood count: check WBC, CRP for early marker for
infection
High vaginal swab: culture for gonorrhoea, Chlamydia,
trachomatis
Low vaginal swab: GBS
Urine FEME: to rule out UTI

1.
2.

Fetal
Serial CTG (beware of increasing baseline heart rate or
fetal tachycardia early sign for intrauterine infection
Ultrasound: look for AFI, fetal biophysical profile
Fetal fibronectin is a useful confirmatory test if doubt
about diagnosis

5.

3.
4.

6.
7.

8.

MANAGEMENT
Admit patient (inpatient management)
Monitor patient (Vigilance for chorioamnionitis)
Clinical (Maternal fever, tachycardia, uterine
pain/tenderness, purulent vaginal discharge, fetal
tachycardia)
Laboratory investigations e.g. total white count and
differential count, C-reactive protein, HVS C&S
Biophysical profile of fetus
IM dexamethasone
nd
12mg stat, repeat 2 dose after 12 hour
Prophylaxis antibiotic
Erythromycin 250mg orally 6hourly for 10 days
To reduce risk of chorioamnionitis
If chorioamnionitis: cephalosporin + metronidazole
(augmentin) immediately
Discharged after observe 48-72 hous if no more leaking
liquor, no infection, no labor symptoms
Delivery indicated if:
Chorioamnionitis is diagnosed
Fetal distress occurs
Post-natal:
Maintain vigilance and screening for infection.
Neonatal screen for sepsis

86

Together in Delivering Excellence (T.I.D.E)


COMMON QUESTIONS
1. Complications
Maternal complication:
1) Infection - Associated with 30% subclinical chorioamnionitis with PPROM.
2) Abruptio placenta is evident in 4-7% of women with PPROM.
3) Psychosocial sequelae, particularly related to PPROM, with the disruption created by maternal
hospitalisation and continued observation associated with uncertain fetal/neonatal prognosis.
4) Increased likehood of operative delivery associated with induction of labour will increased the chance of
maternal complications.
5) Increased incidence of marginal cord insertion and Battledore placenta which may account for increased
incidence of retained placenta. This in turn may be associated with the known increase in the incidence
of primary and secondary post-partum hemorrhage.
6) Associated with 10% incidence of endomyometritis
Fetal complication:
1) Prematurity and risks of prematurity like RDS, NEC, intraventricular hemorrhage, etc.
2) Neonatal sepsis - 2-4%
3) Oligohydroamnios tetrad characterised by facial anomalies, limb position defects, pulmonary hypoplasia
and impaired growth, all of which will lead to neonatal morbidity.
4) Fetal hypoxia due to greater risks of cord prolapse, cord compression and abruptio placenta
5) Neonatal morbidity will be increased due to mechanical difficulties encountered with delivery either by
vaginal or abdominal route as a result of increased incidence of malpresentations and reduced volume
of amniotic fluid
2. Advise upon discharge
- No sexual intercourse
- Come back if leak again, contraction pain, infection
- Frequently follow up antenatally
3. Role of tocolytic in PPROM
- No significant benefit in prolongation of pregnancy to term
- Allow course of steroid for fetal lung maturation to be completed
- To facilitate transfer of undelivered mother to a unit that able to provide appropriate neonatal care
Reference:
1. Obstetric 10 Teachers 19th edition (MOST)
2. Specialist notes
3. Student notes o&g

87

Together in Delivering Excellence (T.I.D.E)


Heart Disease (HD) in pregnancy [Hidayah]

HISTORY
Epidemiology : 1 % of pregnancy
Risk factors :
1. CHD
2.hypercholesterolemia
3. Family hx of HD , hypercholesterolemia
4.hx of RHD during childhood
History
1. Diagnosis -type of lesion, when/where/how?,
causes (CRHD,CHD,HPT), procedure( exercise
test,echo), surgical/corrective done?, any
improvement/residual? Medication& diet,
compliance/f.up, cx, admitted before?
2. Symptom fatigue at rest, exertional chest pain,
orthopnea, PND, palpitation, exertional syncope,
NYHA classification*
3. Any pre conceptional advice surgical advice?,
explain about mother& fetal risk, willing for
frequent f.up&long stay in ward, should have
early booking.
4. Booking- when, ix,echo&result, early u/s for
dating (risk of prematurity), u/s for CHD~20W
5. Symptom that may aggravated heart failureAnemia (pallor, lethargic), UTI (dysuria,
frequency), URTI (running nose, sorethroat)
6. Pregnancy- in labour ? fetal movement

PHYSICAL EXAMINATION
General
o Anemia
o Clubbing, infective endocarditis signs,
o xanthoma
o Pulses (arrhythmias)
o Blood pressure
o Jugular venous pressure
o Cyanosis
o Dental carries
o ankle edema
Abdomen examination (as usual)
Uterus smaller than date or any abnormalities
Chest examination
Shifted apex beat
Murmur
Basal crepitation

INVESTIGATION
Maternal
1.
2.
3.
4.
Fetal

FBC Hb , WBC ( can aggrevate HF)


Urine FEME (TRO UTI)
ECG
Echocardiogram

1. CTG : to look evidence of fetal hypoxia


2. U/S : detailed detection of fetal anomalies, AFI,
fetal growth&placentation.

88

Together in Delivering Excellence (T.I.D.E)


MANAGEMENT
Preconception
1. Fully accessed
2. Treat aggresively any medical illness (try to stabilize before
pregnancy)
3. If surgery needed, do b4 pregnancy (mitral valve replacement in
MS)
4. Counselling about:
Effect of haemodynamic changes&maternal risk
Effect of fetal growth & preterm
Genetic transmission
Effect of maternal drug & compliance
Need for frequent admission & long stay
5. Encourage complete family earlier&discourage from multiple
pregnancy.

Time and mode of delivery


1. Mild & moderate heart dis
Aim for spontaneous vaginal delivery
Avoid induction of labour if possible
2. Admit patient early, wait for birth if :
Severe heart disease
Develop acute heart failure
3. Prepare for preterm labour for severe heart dis pt (high chance
to deliver at 32-34W in severe MS)

Intrapartum
1. Aim delivery within 6 H
2. Stop heparin b4 pregnancy (clexane stop 6H b4 delivery)
3. Prop up to left lateral tilt
4. Continue ECG, CTG & pulse oximeter
5. Give oxygen (3L/min)
6. Give epidural anaestesia
7. AB prophylaxis (in severe cases)
IV ampicillin 2g stat & 8 H later 2x doses
IV gentamicin 800mg & 8H later 2x dose
8. Avoid fluid overload by close monitoring during fluid
therapy
nd
9. Shortened 2 stage by using instrumental delivery
rd
10. For 3 stage, give syntocinon(dont give ergometrine)

Postpartum
1. Keep pt in labour room for 6-24H after delivery
2. Give Lasix 40mg stat (to prevent fluid overload during
ntrapartum)
3. Admit ward at least 1 w
4. Close monitoring (i/o chart, BP, PR, RR, ECG)
5. Look for any complication
6. Continue prophylaxis AB for 5 days
7. Advise for contraception (POP,barriermethods, sterilizationBTL)

Management of Heart Failure


1. Inform cardiologist,obstetrics,anes-send to CCU/ICU
2. Semi-recumbent (propup)
3. Oxygen
4. Morphine (pain relief&dilate vein)
5. Frusemide
6. Digoxin-dysrhytmia
7. Endotracheal intubation
8. Assisted aspiration
9. Cardiac surgery
10. Leg exercise/stocking (prevent DVT)

Antenatal
Booking
1. All mother examined carefully for CVS abn.
2. If suspected, refer to cardiologist & do echo
3. Known case should book early
4. Cases should be managed in combined clinic

Antenatal
1. Regular antenatal checkup
Assess maternal : ( sx HF and NYHA), ECG, vital sign
Assess fetal : detailed scan for congenital anomalies,
serial growth scan to detect IUGR,
Fetal kick chart, CTG.
2. Avoid factor aggravate HF and treat
Anemia
Infection- UTI, URTI
Hyperthyroidism
Arrhytmia
Multiple gestation
3. Advise about : rest, no smoking, compliance to hematinic,
care about infection, dental checkup (prevent I.E)
4. Anticoagulant
indicated in pt congenital heart disease who have
pulmonary HPT or artificial valve replacement.
3 types of regime :

Continue warfarin throughout pregnancy, replace heparin


for delivery (1-2w prior to delivery
st

Replace warfarin with heparin in 1 trimester,other same


st
as 1 regime

Use heparin throughout pregnancy

89

Together in Delivering Excellence (T.I.D.E)


Common/Possible Question in Exam + Answer
1.

CVS Normal physiology in pregnancy


a.

b.
c.
d.
e.
f.
g.

2.

Risk stratification - maternal risk


Low risk
(mortality <10%)

1.
2.
3.

CO= SV x HR

st

CO in 1 trimester 30-50%, max at 20-24W d/t :

Stroke volume in early pregnancy

HR in late pregnancy
*bcoz of this pt prone to develop acute pulmonary edema at 20-24W
nd
Blood volume 40% , accelerated in 2 trimester, peak at 24W

Primi-1200ml, Multi-1500ml,Twins 2000ml


Falli in systemic vascular resistance & BP d/t AV shunt in placenta & vasodilator effex=ct of progesterone
in pulmonary blood flow but in pulmonary vascular resistance.
Term , uterus compression on IVC, 30% CO
During labour, anxiety, pain & uterine contraction CO bout 50%
Immediately after delivery
venous return d/t relief of caval compression & auto transfusion from contracting uterus.
HR & CO within hours post delivery

Acyanotic heart disease


Uncomplicated AR/MR
Uncomplicated septal defect
(ASD/VSD)

Moderate risk
(mortality 5-15%)
1.
2.

High risk
(mortality 25-50%)

Coarctation of aorta
Prosthetic valve on coagulation

1.
2.
3.
4.
5.

3.

Risk factor to develop heart failure

4.

5.

Systemic ventricular dysfunction (ejection


fraction<30%,NYHA class III-IV)
Pulmonary HPT
Cyanotic heart dis
Aortic pathology (dilated aortic root
>4cm,Marfan syndrome)
MS&AS (severe)

Respi / urinary infection


Anaemia
Obesity
Corticosteroid
Tocolytics

Multiple gestation
HPT
Arrhythmias
Pain related stress
Fluid overload

Stages of heart failure

Side effect of anticoagulant


Warfarin : maternal bleeding (at vitamin K & FFP during labour, fetal intracranial hemorrhage and teratogenicity.
Heparin : usage more than 6 month causing osteoporosis

6.

Complication of heart dis in pregnancy


:
Maternal
1. Heart failure
2. Arrythmia
3. Acute pulmonary edema

Fetal
1. Preterm labour
2. IUGR
3. Congenital heart disease
4. Maternal cyanosis (fetal hypoxia)
5. Effect of maternal drug
(teratogenesis, growth restriction, fetal
loss)

90

Together in Delivering Excellence (T.I.D.E)


Abnormal Lie/ Breech Presentation [Amy]
HISTORY
o

o
o
o
o
o

Age, Parity high parity increase risk, Gravida, LMP,


POA-prematurity, EDD
**unstable lie : lie of fetus change time to time beyond 37
weeks
U/L GDM or PIH macrosomia, fetal abn eg
hydrocephalus, anencephalus
Gynaecology problem/surgery done eg menorrhagia,
mass uterine fibroid , scar
History of antepartum bleeding placenta praevia
Congenital abn uterine malformation
Scan done
No of gestation multiple gestation
Liquor volume poly/oligohydramnios
Fetal growth - macrosomia
Fetal abnormalities
Fetal movement - IUD

PHYSICAL EXAMINATION
Inspection
Broad fundus transverse
Palpation
SFH smaller than date transverse/oblique
No presenting part felt over brim
Try to feel for uterine fibroid

Type of abnormal lie :


1. Transverse
2. Oblique
3. Unstable lie

Breech presentation
Palpation : longitudinal lie, firm lower pole, limbs to one side, hard
head at fundus
Auscultation : fetal heart heard above umbilicus
VE : no head in pelvis. Soft buttock felt and hard irregular sacrum.
Type of breech presentation :
1. Extended breech **
2. Flexed breech

3.

INVESTIGATION
1.
2.

3.

GSH before ECV


Ultrasound
o Confirm gestation no and age prematurity
o Normality congenital abn
o Biometry IUGR, macrosomia
o Viability IUD
o AFI oligo/polyhydramnios
o To confirm lie/presentation of fetus
o Uterus fibroid, placenta praeviae
CTG

Footling breech

MANAGEMENT
External Cephalic Version (ECV) breech
o Nulliparous : 36w, multi : 37w
o NBM 6h and request GSH
o Prepare for c-sec if ECV failed.
o Administer tocolytics eg. Nefidipine, Salbutamol
o Fetal heart rate monitoring before and after
o No more than 2 attempts at version
o Before depart, check FH, PV discharge/bleeding
o If failed, elective c-sec at 38-39 w or vaginal breech
delivery spontaneous breech delivery, assisted breech
delivery, total breech extraction
OR
Stabilizing Induction unstable lie
1. Vaginal delivery if cervix favourable
2. Ensure longitudinal lie & cephalic presentation
3. IV Syntocinon to start uterine contraction
4. Keep close watch to have continuing cephalic presentation
5. ARM when uterine contraction regular with :
o Stabilizing head into pelvis by assistant
o ARM when uterine contraction to prevent amniotic
fluid embolism
o Slow release of liquor
6. VE to exclude cord prolapsed.
If failed, elective c-sec at 38weeks.

91

Together in Delivering Excellence (T.I.D.E)


1.

Definitions
o Lie : relationship between longitudinal axis of fetus to longitudinal axis of maternal uterus
o Presentation : part of fetus that present in pelvic brim

2.

Complications
o Antenatal : risk PPROM and cord prolapsed
o Intrapartum
Fetal
o rapid compression and decompression cause intracranial injury
o Cord compression delay will cause birth asphyxia
o Abdominal viscera damage
Mother
o Uterine rupture/cervical lacerations/perineal laceration
o Uterine atony/haemorrhage

3.

ECV
**women should be counselled successful rate is 50%
Easier to perform if :
o Multiparous
o Adequate liquor volume
o Breech free above pelvic brim
o Flexed breech
Contraindications:
Absolute :
o Where c-sec is required
o Footling breech
o APH within 7days
o Abnormal CTG
o Major uterine anomaly
o Ruptured membrane
o Multiple pregnancy

Relative :
o SGA with abnormal Doppler parameter
o Proteinuric PE
o Oligohydramnios
o Major fetal abnormalities
o Scarred uterus
o Unstable lie

Risk of ECV :
o Placenta abruption
o Uterine rupture
o Rupture of membrane
o Cord accident
o Transplacental haemorrhage
o Fetal bradycardia
4.

Vaginal breech delivery- unfavourable :


o Other CI for vaginal delivery eg. Praeviae, compromised fetal condition
o CPD
o Footling or kneeling breech
o Large baby (>3.8kg)
o IUGR (<2.0kg)
o Previous c-sec

92

Together in Delivering Excellence (T.I.D.E)


Anemia in Pregnancy [Aiman]

HISTORY
Active symptom:
Weakness/lethargy
Headache
Palpitation
Shortness of breath
Dizziness
Further history:
1. Hx of bleeding (APH)
2. Current pregnancy APH, Multiple pregnancy, infection
- if diagnose with anemia - how?when?who?hb level at
booking, when diagnose, during subsequent follow up
and latest? On what treatment? compliance?
3. Hx of previous pregnancy PPC, spacing, infection,
anemia
4. Family hx of anemia require blood transfusion (her
thallasemia status and family)
5. Diet hx vege

Complication:
INVESTIGATION
Bedside:
Urine dipstic : proteinuria suggest UTI
Lab:
FBC (at booking and repeat back at 28weeks-RCOG)
Hb : anemia
MCV(80-95fl) and MCHC(20-35g/dL)
i. normal(normocytic normochromic) haemolysis, blood
loss
ii. lower(microcytic hypochromic) IDA, thallasemia,
anemia of chronic disease
iii. higher(macrocytic) folic acid or vit b12 deficiency,
aplastic anemia
Specific:
Serum ferritin: lower than normal (less than
10mg/dl)= IDA
Electrophoresis : normal or not for thallasemia

PHYSICAL EXAMINATION
General Examination
Pallor palm and conjunctiva
Increase respiratory rate
Angular stomatitis, glossitis, koilonychias suggest
anemia due to IDA
Others
CVS : can presented with CCF (ankle oedema, basal
crepitation) and murmur due to hyperdynamic blood flow

Differential Diagnosis :
IDA (most common in Malaysia, due to poor spacing)
Thallasemia
Sickle cell
Folic acid deficiency
MANAGEMENT
In Malaysia: all pregnant lady was prescribe with iron supplement as
prevention
And screening done during booking
# Management depends on cause and severity of anemia
For IDA
Antenatal:
Trial of iron therapy or iron supplement if IDA, patient HB will
increase in 2-3 weeks time (diagnostic and therapeutic)
Patient with severe anemia require referrel to specialist centre for
further management
Target is >10g/dL hb prior to delivery
Intra:
GSH
rd
PPH prevention active management during 3 stage
Start transfusion if severe anemia or excessive blood loss
Post:
Iron supplement
Counselling regarding contraception and spacing
Admission:
When need for transfusion (moderate, >37weeks with sign of
labour or severe or with complication)
When have active blood loss

93

Together in Delivering Excellence (T.I.D.E)


1. Hb level (expected to be lower than normal in pregnant lady):
Anemia @booking <11g/dL
2nd and 3rd trimester <10.5g/dL
Or (<10.5g/dL RCOG, <11g/dL WHO)
Mild (8-10g/dL), moderate (6-8g/dL), severe (<6g/dL)
2. Risk factor
a.
b.
c.
d.

IDA poor spacing, poor diet(vege), blood loss(worm,infection)


Thallasemia
Folic acid deficiency
Others myeloproliferaive disorder

3. Complication
a. Inability to withstand haemorrhage
b. Increase risk of :
i. Infection
ii. Cardiac failure
iii. PPH
c. Fetal : low birth weight, IUGR
d. Spontaneous abortion in Thallasemia
4. Diet consultation on how to take iron supplement
Take on empty stomach, 1 hour before meals, take with Vit. C (juice or supplement) to maximize absorption,
and avoid taking antacid, tea or coffee
Type

Name

Advantage

Oral iron

Ferrous fumarate(TD)
Ferrous sulphate(TD)

Cheap
Easy to take

Parenteral iron

Dextrin
Imferon

For severe anemia

Disadvantage
Dark stool
Nausea and vomiting
Not compliance
Anaphylactic reaction
Pain and
discolouration at
injected area
Skin necrosis

*Parenteral iron can be considered from the second trimester onwards and during the third trimester for women
with confirmed iron deficiency who fail to respond to or are intolerant of oral iron

94

Together in Delivering Excellence (T.I.D.E)


UTI In Pregnancy [Aiman]
HISTORY
Lower UTI
Dysuria, frequency, urgency
Occasional suprapubic pain and hematuria
Cloudy urine

PHYSICAL EXAMINATION
General Examination
- Depends on severity
Tenderness (urethra, suprapubic, loin area)

Acute pyelonephritis(rapid progress hr to day)


Fever + chilss and rigors, flank pain
Varying degree of dysuria, frequency, urgency
Mild: appear well
Severe: very ill, vomiting, dehydration, abdominal and loin
pain
Current pregnancy infection (vaginal discharge)
- if diagnose with UTI - how?when?who? when diagnose?
On what treatment? compliance?
Further history:
Previous history of UTI
U/L: DM, HIV, STD, obstruction(stone, tumour, stricture),
neurogenic bladder

Tachycardia +
Tachypnea +
Febrile +
+ in acute pyelonephritis

Differential Diagnosis :
Acute pyelonephritis
Cystitis
Urethritis
Acute appendicitis
Acute pelvic inflammatory disease
VE examination

Complication:

Miscarriage
Premature birth
PPROM
Low birth weight

INVESTIGATION
Bedside:
Urine dipstick (midstream urine must send within 4hr
sample taken)
proteinuria, pH, *leukocyte esterase, *nitrites
*if positive with symptom 90% is UTI, if negative less likely to
be UTI
Imaging:
USS screen for hydronephrosis, kidney stone, abscess

MANAGEMENT
1. Increase fluid intake
2. Increase bladder empty (adequate urine output to flush
out microorganism)
3. Antibiotic
Ampicillin/amoxicillin T. amoxicillin + clavulanate
(augmentin) 625mg BD for x7-14/7
-

Lab:
Urinalysis urine microscopy (white cell count
>10WBC/microL, bacteria, pyuria, rbc cast)
Urine C + S on cysteine-lactose-electrolyte-deficient(CLED)
medium using urostrip method (in complicated case or
when to change regime)
+ in pyelonephritis
ABG - Respiratory alkalosis PaCo2 <32mmHg
FBC - Leukocytosis <4000 or >12000 cells/mL
BUSE,Creatinine Renal function
LFT
Blood culture and sensitivity
Specific:
Urinalysis

Cephalosporin (not effective, not active against


proteus, and affecting normal flora)
Cephalexin 0.5-1g BD x3-7/7
Cefuroxime 250-500mg BD x3-7/7
[usually in pregnancy (complicated UTI) longer course is
needed for 1-2 weeks]
Follow up after 1-2 weeks complete antibiotic + urinalysis and
culture
4.

Antipyretic

95

Together in Delivering Excellence (T.I.D.E)


1. Why more in female:
a. Close to anus (short proximity to anus)
b. Its opening to labia
c. Short urethra length
d. Used of spermicide (diaphragm, cervical cap, condom), insertive rectal intercourse
2. Why in pregnancy is higher(detected in 2-85 of pregnancy):
a. Predispose factor for UTI:
i. Reduce urethral tone
ii. Reduce ureteral peristalsis
iii. Temporary incompetence of vesicoureteral valves.
iv. Bladder catheterization before or during delivery increase risk
b. Obstruction (pelvis or abdominal tumour, stricture, renal stone)
3. Etiology:
a. Uncomplicated (most common): E.coli
b. Complicated: wide range from gram +ve to ve
c. Community acquired: (Escherichia coli, Klebsiela pneumonia, Proteus mirabilis, Staphylococcus
saprophyticus, Enterococcus faecalis)
d. Hospital acquired: (E.coli, Pseudomonas aeruginosa, Proteus sp., Enterobacter)
4. Pathogenesis :
a. Ascending route (mostly)
Normal flora vs gram ve bacilli(abnormal flora)
b. Hematogenous
c. Lymphatic (rare)
5. Bacterial virulence factor (not all E.coli strain capable belongs to serogroup O, K, and H)
Bacterial adherence to uroepithelial celss (hairlike proteinaceous surface and p. fimbrae)
Produce cytotoxins, hemolysin and aerobactin (a siderophore to scavenging iron) make it resisaant to
bacteriacidal action of human serum

96

Together in Delivering Excellence (T.I.D.E)


Uterus smaller than date [Nur Diana]
HISTORY

PHYSICAL EXAMINATION

*Condition:

*Symphysial-fundal height (SFH) < 3cm of gestational age

*Causes:
1. Wrong date
2. Oligohydramnios
3. Missed miscarriage
4. Intrauterine death
5. Fetal growth restriction (FGR)
6. GDM
7. Anemia
8. PIH / pre eclampsia

*Examine fetal heart rate by using pinard

*Complication:
Fetal hypoxia, stillbirth
INVESTIGATION

*Serial ultrasound to detect any FGR, liquor amount

MANAGEMENT

* is tailored accordingly to treat the cause.

(oligohydramnious).
*A detail fetal anomaly scan
*Doppler study of umbilical artery flow
*investigations to find out the causes

*What are the causes of FGR?


*What do you understand by the term symmetrical & asymmetrical FGR?
A. Symmetrical FGR
is more common
usually begins late in pregnancy
manifested as restriction of weight followed by length
but the head continues to grow at normal or near-normal rates
B.

Asymmetrical FGR
is less common
begins early in pregnancy
manifested as generalized growth restriction

97

Together in Delivering Excellence (T.I.D.E)


Uterus Larger Than Date [Sofia]
HISTORY
st

PHYSICAL EXAMINATION

1. Ask LMP in detail, 1 scan n subsequent scan for REDD

PHYSICAL EXAMINATION (like usual flow of PE)(+ve findg)

2. Ask rgrdg causes of polyhydramnios (AFI >25 / deepest pool >8)


Maternal GDM- ask indication of MOGTT in the pt and result
Fetal: multiple pregnancy(d/t TTTS): when, where & how, risk
factor(assisted conception, family hx of multiple preg, high
parity, adv maternal age) , increase symptoms of pregnancy
(hyperemesis, abd discmfrt, GERD, SOB, frequency, leg swellg),
ask complication (TTTS, single fetal demise)
:fetal abN: esophageal atresia, tracheoesophageal fistula,
duodenal atresia, anecephaly,
placenta: chorioangioma, AV fistula
ask d AFI value
ask complication of polyhydramnios (malpresentation, abN lie,
preterm labour)

A.

Polyhydramnios:

Abd distention w tense shiny skin (not really)

Tense on palpation

SFH & fundal height larger than date

Difficult to palpate fetal part or poles

abN lie or presentation

+ve fluid thrill

B.

Multiple pregnancy:

SFH larger than date

2 fetal poles

2 fetal heart sound


MANAGEMENT

#Management for polyH, GDM refer dodis short notes


3. Ask regarding s/s of uterine fibroid / pelvic mass: menseswhether had heavy bleeding, pelvic pain, recurrent miscarriage,
pressure symptoms (frequency, urinary retention, constipation,
varisoce vein, leg edema) ask regrdg complication of fibroid to
pregnancy: fetal malpresentation, IUGR, preterm labour, placenta
previa, PPH, obstructed labour)

1.

4. Ask for macrosomic baby: whether pt DMT2 or GDM, MOGTT


indication & result, risk factor(famly hx, past hx of GDM), if GDM or
DM as regardg compliance to mdctn n diet in detail, HbA1c, s/s of
DM (polyuria, polydipsia, recurrent UTI, vaginal candidiasis),
previous baby weight

Establish the cause- if DM, need to optimize the glycemic


control, if due to TTTS, can do amniodrainage & laser ablation of
anastomose placental vessel, if d/t fetal aBN, monitor fetal
wellbeing
Amniocentesis: for severe poly, symptomatic poly, failed
indomethacine, cx: preterm labour, chorioamnionitis,
PPROM & abruption
Indomethacine (PG-E)- reduce liquor amount by antagonist
anti diuretic effect of ADH on collecting duct, shud
discontinued at 32-35w d/t risk of premature closure of
ductus arteriosus, cerebral vasoC, impaired renal fx

2.

Ultrasound- like describe earlier

5. Baby well being (fetal kick chart)

3.

Admit if- maternal discomfort, risk of preterm labour (give


dexamethasone), risk of membrane rupture (risk of cord
prolapsed & placenta abruption)

4.

Time of delivery: 38-39w

5.

Mode of delivery:
a) Spontaneous vaginal delivery- if cervix favourable &
longitudinal lie
b) Elective c-sec- if poly + transverse lie

6. Details of ultrasound (any fetal abN, AFI, any pelvic mass, fibroid,
position of placenta)
7. S/S of labour
INVESTIGATION

1.
2.
3.
4.
5.
6.

FBC to look at Hb level


BUSE- for baseline
GSH
Ultrasound- like I describe earlier
CTG- for fetal wellbeing
GDM ix- refer dodys notes

98

Together in Delivering Excellence (T.I.D.E)


Common/Possible Question in Exam + Answer
1.

UTERUS LARGER THAN DATE?

Wrong date (LMP)


Polyhydramnious (excess amount of Amniotic Fluid in Uterus)
Larger baby (macrosmia) ( d/t GDM: dodi akn cover)
Multiple pregnancy
Molar pregnancy
Uterine fibroids
Pelvic mass

2.

Uterus smaller than the dates?


Wrong date (LMP)
Oligo hydramnious
Smaller baby (microsmia)
Head engaged too deeply
Genetic (maternal paternal small size)
Intrauterine Growth Restriciton (IUGR)
Intrauterine death (IUD)
Transverse lie

3.

Amniotic fluid index


Summation of all max vertical pool (4 quadrants)
rd
3 trimester value: (N: 10-25)
<5cm : oligo
<10cm : reduce
25cm : polyhydramnios

99

Together in Delivering Excellence (T.I.D.E)


Dysmenorrhoea [Farhan]
HISTORY
Definition: painful menstruation
Primary (absent of pelvic pathology) vs secondary (presence
of pelvic pathology)
Risk factor for severe dysmenorrhea:
i. Early menarche
ii. Nulliparity
iii. Long menstruation period
iv. Heavy menstruation
v. Smoking
vi. Positive family history
Primary dysmenorrhoea:
i. occurs 6 to 12 months following menarche
ii. pain: lower abdominal and cramping in nature; may
radiate to the back and to the inner thigh, lasts from 8
to 72 hours and accompanies menstrual flow or
precedes it by only a few hours

iii. a/w other systemic symptoms such as vomiting,


nausea, diarrhoea, fatigue, and headache
Secondary dysmenorrhea:
i. occurs years after menarche (30s 40s)
ii. pain: not consistently related to menstruation alone,
may worsens as menstruation progress
iii. a/w irregular bleeding pattern, heavy periods,
dyspareunia, vaginal discharge
iv. common causes: endometriosis, chronic pelvic
inflammatory disease, adenomyosis, intrauterine
polyps, submucosal fibroids
INVESTIGATION
Urine pregnancy test
FBC: to exclude anaemia
WBC, ESR, and CRP: raised WBC with a high ESR and CRP
acute/chronic inflammatory disease, but not specific
Swabs: for chlamydia and gonorrhoea from the endocervix and
posterior fornix of the vagina
Tumour markers such as Ca-125: should be checked in the
presence of an ovarian mass, but non-specific and levels can be
elevated in endometriosis
Ultrasonography: fibroids, adnexal pathology, endometriomas,
and intrauterine contraceptive devices are best assessed with
ultrasonography
Laparoscopy: aids in the diagnosis of endometriosis, adhesions,
and pelvic inflammatory disease and at the same time can offer
treatment
MRI and CT pelvis and/or abdomen

Hysteroscopy

PHYSICAL EXAMINATION

i.

ii.

iii.
iv.

Do a complete pelvic examination, external gentalia,


speculum and bimanual examination
Primary dysmenorrhea: usually unremarkable
Secondary dysmenorrhea
Endometriosis: positive in ~40% of the time with
adnexal, rectovaginal, and uterine tenderness,
together with cervical excitation.
Acute or chronic inflammatory disease: tender pelvic
examination and abnormalities such as the presence of
adnexal masses (usually bilateral) may occur,
particularly in chronic disease. Speculum examination,
in acute cases, may reveal a purulent vaginal discharge
and an erythematous vagina and cervix.
Fibroid uterus: suggested by an enlarged uterus with
an irregular contour.
Adenomyosis: a tender and globular shaped enlarged
uterus is frequently palpable during bimanual
examination

MANAGEMENT
Primary dysmenorrhea: to provide relief from cramping pain and
associated symptoms
Secondary dysmenorrhea: pain control and treat the underlying
cause
NSAIDs: decrease menstrual pain by decreasing intrauterine
pressure and lowering prostaglandin F2 (PGF2) levels in
menstrual fluid. e.g. Ibuporfen, Diclofenac, Mefenamic acid,
Naproxen
Oral contraceptives: COCPs, the levonorgestrel intrauterine
device, and depot medroxyprogesterone acetate provide effective
pain relief and are associated with reduced menstrual flow. It may
be necessary to add an NSAID to the OC.
Others: direct application of heat, diet e.g. low fat vegetarian, fish
oil, vitamin E

Prevention: exercise, smoking cessation

100

Together in Delivering Excellence (T.I.D.E)


1. How does Prostaglandin cause pain?

prostaglandin production cause :


uterine cramping uterine ischaemia and pain
cause hypersensitization of pain terminal to physical and chemical stimuli
studies shown PG production during 1st 48-72 hours of menses
PG can decrease the pain

2. How do birth control pills reduce cramping?

thickness of the endometrium and change the hormone status to the same levels as those found in the early
proliferative stage
lowest level of PG production
cramping, uterine ischaemia and pain

101

Together in Delivering Excellence (T.I.D.E)


Ovarian CA [Rafidah]
HISTORY
Epidemiology

PHYSICAL EXAMINATION
GENERAL

Lifetime risk in general population 1:70 (1.4%)


Mean age 64 yrs old
Women with hereditary cancer present early with mean age of 54 years
Epithelial ovarian ca >50 yrs
Malignant Germ cell tumour younger than 20 yrs old

SPECIFIC

Aetiology and Risk factor


Decreased risk of ovarian Ca
Multiparity
OCP
Tubal ligation
Hysterectomy

Increased risk of ovarian ca


Nulliparity
Intrauterine device
Endometriosis
Cigarette smoking
Obesity
Early menarche
Late menopause
Clomiphene citrate > 1 year
Hereditary ovarian ca
BRCA1,BRCA 2, HPNCC

S&S

Pallor
Loss of weight
Cachexia(late)
Palpable left supraclavicular LN

Asymptomatic
Abnormal uterine bleeding d/t hormone producing ovarian cancer
such as granulosa cell tumour
Gastrointestinal sx: bloating, abdominal distension, abdominal
discomfort
Abdominal mass
Respi sx due to abdominal distension, plueral effusion, or
metastases
Constituitional sx: LOW, LOA, lethargy
Persistent pelvic and abdominal pain
Acute sx are rare, this result from pain due to torsion, rupture,
infection, or intracystic hemorrhage
Bowel and urinary sx if being compressed

Abdominal examination

Bimanual examination

1)
2)
3)

CA 125 >35iu/ml (epithelial ovarian ca more in serous type)


CEA in mucinous
AFP (in germ cell tumour such as endodermal sinus tumour, immature
teratoma, dysgerminoma
4) Hcg in choriocarcinoma
5) Inhibin and estrogen in granulosa cell tumour
6) Androgen in sertoli leydig cell tumour
7) FBC anemia
8) BUSE electrolyte imbalance
9) LFT - metastasise
10) Chest xray - metastasise

Imaging
a.
Ultrasound :
Features of malignancy bilaterality, presence of solid areas, papillary
projection, ascites, hydronephrosis, or liver secondaries
b. CT scan:
To identify lymphadenopathy and detect peritoneal tumour deposit,
assessment of operability

Endoscopy

Done in those with occult blood or significant intestinal sx

To exclude primary colonic cancer with ovarian metastases

Adenexal mass fixed/ immobile if spread to surrounding


structure

Chest examination

Palpation: Reduced chest expansion


Reduced vocal fremitus
Percussion: stony dull
Auscultation: reduced breath sound and vocal resonance

Breast examination

INVESTIGATION
Laboratory

Abdominal distension
Fluid thrill +ve in gross ascites, shifting dullness in mild ascites

Breast lump (breast ca may metastasise to ovarian

MANAGEMENT (EOC)
STAGE 1a, 1b, 1c
TAHBSO the best therapeutic management
Omentectomy is also done - as it harbour microscopic disease
All clinically stage 1c will be given post operative adjuvant chemotherapy

In stage 1a or 1b additional features such as grade of tumour ,


histological type need to be considered => 1a or 1b with grade 2 or
grade 3 tumour, risk of relapse is high it is recommended to give
adjuvant chemo

If patient wish to keep fertility, consider conservative fertility sparing


surgery (justified after careful exploration to exclude metastatic disease)

Unilateral salpingo-oophorectomy

Omentectomy

Peritoneal biopsy

Pelvic/paraortic disecction
STAGE 2a, 2b 2c

TAHBSO with omentectomy is the treatment of choice in many


centres

Adjuvant therapy include systemic chemotherapy


ADVANCED STAGE : STAGE 3 & 4

Patients who have had cytoreduction (debulking) should receive


adjuvant chemotherapy

Patients who could not undergo primary cytoreduction prior to


chemotherapy due to physical unfitness:
interval debulking may be considered after three cycles of
systemic chemotherapy (neoadjuvant chemo)

102

Together in Delivering Excellence (T.I.D.E)


Chemotherapy
Can be given as:

Primary treatment
Adjunct following surgery
relapsedisease

First line treatment:

Combination of platinum compound with paclitaxel (given as outpatient basis, 3 weeks apart for 6 cycles)
Platinum
Most effective chemotherapeutic agent in ovarian cancer
MOA: heavy metal cause cross linkage of DNA strand, thus arresting cell replication
Eg: carboplatin (less renal toxic, less nausea), cisplatin (more S/E compared to carboplatin)
Paclitaxol
MOA: work by causing microtubular damage to cell, thus prevent replication and cell division
Given together with steroid to prevent high sensitive reaction, side effect of peripheral neuropathy, neutropenia,
myalgia and loss of total body hair.

FIGO staging
Stage
I
IA
IB
IC
II
IIA
IIB
IIC
III
IIIA
IIIB
IIIC
IV

FIGO definition
Growth limited to one ovaries
Limited to one ovaries: no external tumour. Capsule intact, no ascites
Limited to both ovaries: no external tumour, capsule intyact, no ascites
Either IA or IB, but tumour on surface of ovary or with capsule rupture or with ascites positive for tumour
cells
Growth limited to pelvis
Extension and/or metastasise to uterus or tubes
Extension to other pelvic organ
As IIA, or IIB, but tumour on surface of ovary or with capsule ruptured or with ascites positive for tumour
cells
Growth limited to abdominal peritoneum or positive retroperitoneal or inguinal lymph nodes
Tumour grossly limited to pelvis with negative nodes but histologically confirmed microscopic peritoneal
implant
Abdominal implants <2cm in diameter
Abdominal implants >2cm diameter or positive retroperitoneal or inguinal lymph nodes
Growth involving one or both ovaries with distant metastases
Must have positive cytology on pleural effusion, liver parenchyma

Prognostic factor in ovarian cancer


-

Stage of disease
Volume of residual disease post surgery
Histological type and grade of tumour
Age at presentation

103

Together in Delivering Excellence (T.I.D.E)


Endometrial Mass (Zuraidah)
PHYSICAL EXAMINATION

HISTORY
Endometriosis : presence of endometrial tissue outside both the uterine
cavity & myometrium
Adenomyosis : endometrial tissue found within the myometrium
Fibroid : benign tumors of uterine smooth muscle (leiomyoma)

Endometriosis

Adenomyosis

Uterine fibroid

History
Peak in 30-45years of age
20 dysmenorrhea- pain begin prior to onset of the
menses, increase intensity during flow, gradually
improves as bleeding settles
Acute/chronic
Pain in lower abdomen, back or perineum
Abnormal bleeding, irregular, menorrhagia
Deep dyspareunia
Infertility
Occur in older multiparous women
0
2 dysmenorrhea
Cyclic, cramping uterine pain
Severe menorrhagia
Distended abdomen (symmetrical)
Occurs in reproductive age
Risk factor: nulliparity, obesity, +ve family hx
Usually asymptomatic
Symptoms
Menorrhagia (submucous)
Dysmenorrhea (slight discomfort to colicky pain at
suprapubic, low backache
Acute abdomen/pelvic pain: degenerated/ torsion of
fibroid
Urinary sx (pressure)
Sbfertility :mechanical distortion/occlusion

INVESTIGATION
Laboratory
1. FBC - hb, mild leukocytosis in endometriosis
2. ESR elevated in endometriosis
3. CA 125 elevated in endometriosis & adenomyosis
Imaging
Endometriosis

Adenomyosis

Fibroid

Ultrasound
May found
Thickened
endometrioma (
myometrium
chocolate cyst) - cyst Ill-defined
containing echogenic
heterogenous
material (inside of
echotexture
cyst looks full of
within
white shadows with
myometrium
layering rather than
uniformly
dark:presence of
blood)
MRI
Definitive ix as it
detect lesion > 5mm
provides excellent
in size, particularly in
images of
deep tissue, eg:
myometrium,
rectovaginal septum
endometrium &
areas of
adenomyosis
Laparoscopy
Dx only confirmed
- gold standard
by histology after
- typical endometrial
hysterectomy
appearance powder
burn small brown/
black puckered
lesion look like
remaining cigarette
burn

hypoechoic, but
can be sooechoic
or even hyper
compared to N
myometrium
calcification is seen
as echogenic foci
with shadowing
cystic areas of
necrosis or
degeneration

Hysterosalpingograp
hy/ hysteroscopy
- detect submucous
fibroid

General

Anemic pallor, conjunctiva pale

Lethargy
Specific
Endometriosis

Palpable mass

Tender if rebound tenderness + rigid abdomen indicate of large


endometrioma ruptures, spilling blood into peritoneal cavity (acute
abdominal pain)

Pelvic exam areas of tenderness / palpabe mass d/t adhesions or


endometrioma.

Rectovaginal exam tender nodular indurations along the uterosacral


ligaments
Adenomyosis

Pelvic exam:
- Symmetrical enlarged uterus & tender all over
- Soft and boggy uterus
Fibroid

Uterus palpable if > 14w


Usually non tender, if present only occur over one localized fibroid (d/t
degeneration)
Firm in consistency & well defined margin
Pelvic exam :
- Enlarged uterus
- regular/ irregular
- uterus not felt separate from the swelling
- cervix move with movement of swelling
- move side to side

MANAGEMENT
Adenomyosis:

definitive tx is hysterectomy with or without ovarian conservation


Endometriosis
Medical
1. Analgesic : NSAIDs
2. Combined oral contraceptive
(COC) taken continuously for
6m induce amenorrhea
3. Progestogens if COC is at risk
4. Gonadotropin releasing
hormone (GnRH) agonist
relieve severity of sx by induce
state of hypogonadotropic
hypogonadism/ pseudomenopause with low circulating
estrogen
Surgical
1. Laparoscopic with diathermy,
laser vaporization or excision
drained & inner cyst lining
should be excised
2. Hysterectomy & bil
salpingooophorectomy
(definitive)
for severe/ progressive dz
women whose completed
families
followed by HRT

Fibroid
Conservative Asymptomatic &
detected incidentally
repeat clinical exam/ us after 612month interval
Medical
1. Gonadotropin releasing hormone
(GnRH) agonist shrinking
fibroids by downregulate the
pituitary thus reduce estrogen
level. Limited to use in
preparation for surgery
(myomectomy or hysterectomy)
Surgical
1. Hysteroscopic removal
menorrhagia a/w submucous
fibroid or fibroid polyp
2. Myomectomy
bulky fibroid that causes
pressure sx
preservation of fertility
3. Hysterectomy (definitive)
4. Uterine artery embolization
(UAE) embolization of both
uterine arteries under radiological
guidance. Lead to shrinkage of
fibroid & reduction in menstrual
blood loss

104

Together in Delivering Excellence (T.I.D.E)

Types of fibroid
Submucous fibroid whorled tumour located adjacent to & bulging into the endometrial cavity
Intramural fibroid centrally within myometrium
Subserosal fibroid at the outer border of the myometrium
Pedunculated fibroid attached to the uterus by a narrow pedicle containing blood vessels

Different types of degeneration in fibroid?

Red
Hyaline
Cystic
Calcification
Malignant/sarcomatous

105

Together in Delivering Excellence (T.I.D.E)


Miscarriage [IQbaL]
HISTORY
Definition : Spontaneous loss of pregnancy at or before 24 weeks gestation
10-20% incidence in confirmed pregnancy
Majority at 1st trimester
Risk factors (RF)
a) Embryonic

Chromosomal abnormalities
b) Maternal

Advanced maternal age (>35yo, dec good no. of good Oocytes)

Genital tract infection (bacterial vaginosis)

Medical/endocrine disorder

Uterine abnormalities

Drugs/chemicals

INVESTIGATION
1.
2.
3.
4.
5.

UPT
FBC infection (anemia, TRO septic miscarriage)
Rhesus blood group
Serum B-hCG titre to confirmed pregnancy
Tran-vaginal US

Key Dx of miscarriage provided serum B-hCG


>1500UI/L
i. empty gestational sac
ii. no visible yolk sac
iii. crown-rump length > 7mm but no fetal heart
activity

Help to differentiate btw complete/incomplete

Key diagnostic factors

i.
ii.

Presence of RF

Vaginal bleeding with or without clots


Other diagnostic factors

Suprapubic pain, low back pain

Recent post-coital bleed

Uterine structural abnormality

Chronological History
Confirmed pregnant? UPT done?
Hx post-coital?
Any atypical blood clot expelled?
Severity of vaginal bleeding? flooding, clots, inc in duration & no.
of pads used per day, anemic sx
Important TRO ectopic pregnancy (pelvic/suprapubic pain a/w
fainting attack or unexplained anaemia)
Recent genital infection? Fever?
Hx of recent trauma

PHYSICAL EXAMINATION
GENERAL

Clinically well

If ill-looking significant blood loss


SPECIFIC
A. CVS

Any pale, tachycardia, hypotension (anemic)

Dyspnea (ectopic pregnancy)


B.

Abdominal-pelvic

Size of uterus

C.

Perineum

Any local lesion that lead to bleeding

Speculum examination

Early pregnancy tissue?

Cervical Os : open/closed?

Laceration, cervical ectropion?

MANAGEMENT
Initial Management

Monitoring : BP, pulse rate, temperature

Lab Ix : Hb level, GXM ( if pt is severely compromised)

Pt with miscarriage can have expectant, medical or surgical


management
Expectant
No intervention
May have unplanned surgery if bleed heavily
Surgical
ERCP (evacuation of product of conception)
Successful rate : 95-100%
Risk of cervical trauma, subsequent cervical incompetence,
uterine perforation, intrauterine adhesion & post-operative
pelvic infection
Medication
Prostaglandin : Orally (misoprostol) or vaginally
(Gemeprost)
Increase rate of successful : prostaglandin + Mifepristone
(progesterone agonist)
Counselling
To ensure pt understand most miscarriage are nonrecurrent

106

Together in Delivering Excellence (T.I.D.E)


WHO classification
Type

THREATENED
[cervical os CLOSE]

Clinical Presentation

U/S finding

Management
i.
ii.
iii.
iv.

Unprovoked vaginal bleed


With/out lower abd pain
Occur <22 weeks gestation

v.
Intra-uterine pregnancy

INEVITABLE
[cervical os OPEN]

Painful vaginal bleed


POC at cervical
If excessive bleeding hypovolemic
shock Ferguson reflex (oxytoxin
release)
# Pregnancy will not continue
proceed to incomplete/ complete
miscarriage

vi.
i.
ii.
iii.
iv.
v.

i.
INCOMPLETE
[cervical os OPEN]

COMPLETE
[cervical os CLOSED]

Missed miscarriage
[cervical os CLOSED]

Painful vaginal bleed


POC partially expelled

Minimal pain or bleeding

With/out pain and bleeding

Retained POC

No retained POC

Fetal pole present but no fetal


heartbeat
Gestational sac present (diameter
>20mm) but no fetal pole
identified

REASSURE pt
Admit if excessive bleeding
TCA scan in 1-2 weeks
Advice : proper rest at home, no strenuous
or heavy work, no SI until 12th week
gestation & no further bleed
Give Duphaston : to restore luteal f(x) &
relax smooth ms of uterus
Anti-D immunoglobulin if indicated
Stabilized pt
Manual evacuation : digitally or with sterile
ovum or sponge-holding forceps
Analgesic
Counselling
Anti-D immunoglobulin if indicated

ii.
iii.
iv.
v.

Controlled bleeding
(IV/IM Ergometrine or IM syntometrine)
Surgical evacuation (D&C)
Analgesic
Counselling
Anti-D immunoglobulin if indicated

i.
ii.
iii.

Analgesic
Counseling
Anti-D immunoglobulin if indicated

i.

ii.

Cervical softening (vaginal gemeprost 1mg,


oral/vaginal misoprostol) 2-3 hour before
evacuation
Surgical evacuation under GA/reginal
anaesthesia

Complication
Septic miscarriage : ascending bacterialinfection secondary to incomplete miscarriage or termination of pregnancy

Common organism : E.coli, bacteroids, streptococci

Symptoms : unwell, fever, lower abd pain. Foul smelling vaginal discharge

Signs
: spiking temperature, lower/pelvic pain, guarding

U/S
: presence of POC

Management
o
Admit pt
o
Ix : FBC, Blood C&S, swab from endocervix fot C&S, BUSE and creat & LFT
o
Fluid resuscitation
o
Anti-biotic (cover gram positive, negative & anaerobes
DIFFERENTIAL DIAGNOSIS
CONDITION
Differentiating signs/symptoms
Differentiating test
TAS is diagnostic : no intrauterine gestational sac,
complex or cystic adnexal mass with/out free fluid in
pouch of Douglas
Atypical symptoms that can be missed ( iliac fossa/suprapubic
pain, unexplained pallor, tachycardia or syncope
Serial serum B-hCG + single measure of
Ectopic pregnancy
progesterone : to distinguish between early viable,
O/E : adnexal tenderness or suggestion of haemoperitoneum
poor prognosis or ectopic gestation
If in doubt, laparoscopy to confirm Dx : distended,
ruptured or haemorrhagic fallopian tube or other
extra-uterine preg site.
Uterine size more larger than expected gestational age
TAS : classic Snow-storm appearance
Hydatidiform mole ( Molar )
Pregnancy Sx are marked
Partial hydatidiform : reveal fetus but unusuallooking placenta
Very rare : passed some molar, grape-like tissue
Suprapubicpain + DYSURIA & FEVER
Urine microscopy and culture
Cystitis
May have haematuria
Pregnancy co-excisting with a
May be suspected from appearances of the ecto-cervix on
Confirm after U/S or by spontaneous avulsion of
bleeding cervical polyp/ large
speculum examination
polyp
ectropion

107

Together in Delivering Excellence (T.I.D.E)

ORTHOPAEDIC

Notes

108

Together in Delivering Excellence (T.I.D.E)


Osteomyelitis [Fatin]
HISTORY
*Inflammation of bone caused by infection
o Epidemiology :
cause by Staph A. (H.influenza in children)
site : children (metaphysis and epiphysis of long
bone) & adult (spine, pelvic & foot if has DFU)
o Causes :
Hemato spread UTI, pneumonia
Local infection infected puncture wound
Direct contamination open #, iatrogenic (surgry)
o Risk factor :
DM, Sickle cell dz, peripheral vascular dz, chronic dz,
alcohol, steroid Rx recent injury, ortho surgery
o S&S :
Pain, malaise, fever
Swelling, redness, tender, warmth, ROM
o Complication :
Spread-septic arthritis (joint), metastatic OM (other
bone)
Shortening / deformity (physis is damage)
Chronic OM pain, fever, redness, swelling, discharge
sinus
No- union fracture

PHYSICAL EXAMINATION
General
Ill looking, in pain
Vital sign : fever, tachycardia
Sign of septicemia
Specific
Swelling, redness, tender, warmth, ROM
Chronic discharge, sinus, non-heal ulcer

1.
2.
3.
4.

INVESTIGATION
o

Laboratory
FBC- leukocytosis
ESR -
Blood culture
Needle aspiration

MANAGEMENT
o

o
o

Imaging
X-ray
- Acute : First 10 days normal finding
Later periosteal rxn, lytic area, soft tissue swelling
- Subacute (Brodies abcess) : lytic area surrounded by
sclerotic bone
- chronic : sequestrum, involucrum, cloaca, sinus tract
Bone scan positive b4 x-ray changes appear

Abscess
Malignant bone tumour (Osteosarcoma, Ewing
sarcoma
Lymphoma
4) Metastatic bone tumour

Non-surgical
Antibiotic (3-6/52)
Adult : cloxacillin & fusidic acid
rd
Children : 3 gen cephalosporin (gram ve)
Analgesic
Surgical
Drainage of abcess
Debridement of infected, dead bone
Bone graft/ packing material
Removal of implant

109

Together in Delivering Excellence (T.I.D.E)


Common/Possible Question in Exam + Answer
1.

List the factors that predispose to post-traumatic infection.


a) Inadequate debridement.
b) Early closure of the wound.
c) Unfixed / unstable fracture.
d) Wound tension.
e) Tight dressing.
f) Haematoma formation.
g) Use of foreign material implant eg. Internal fixation.

2. What are the complication of osteomyelitis?


I. General :
Septicaemia
Metastatic abcsess
ii. Local :
Septic arthritis
Spontaneous fracture
Shortening/ Deformity
Chronic osteomyelitis.
3.

Why metaphysis is affected in children


Highly vascular
Thin cortex
Hairpin like arrangement of capillary blood stasis
Lack of pmn

4.

Pathogenesis

110

Together in Delivering Excellence (T.I.D.E)


Fracture Malunion & Non Union [Rozana]
HISTORY

Non union = bone/fracture fails to unite within expected


time, with the evidence of cessation of fracture healing.

Malunion = The partial and incorrectly angled joining of


two or more large fracture fragments

Risk factor/causes :
Malunion : Failure to reduce a fracture adequately,
Failure to hold reduction while healing proceeds,
Gradual collapse of comminuted or osteoporotic
bone.
Non union : Inadequate blood supply, Infection,
Inadequate fracture immobilisation , Intact fellow
bone, interposition, Smoking, NSAID

Specific :
Malunion : tenderness, deformity & shortening of the
limb, surgical scars , swelling , Limitation of
movements,

Non union : painless, deformity

Sx :
Malunion : pain, loss of function, deformity, swelling,
crepitus, abnormal movement, or positioning of a
limb, soft tissues
Non union : painless, deformity
INVESTIGATION

I.

PHYSICAL EXAMINATION

Imaging :
Non union :
a) Atrophic non-union : - Bone looks inactive /
cessation of fracture healing (Bone ends are often
tapered / rounded), Relatively avascular
b) Hypertrophic non-union : - bone gap, Excessive
bone formation on the side of the gap, (callus)

Malunion : Present of fracture line , altered


anatomical position

MANAGEMENT
i. Non union mx
Depends on the factors : Infection, Inadequate blood
supply, Inadequate immobilisation
Mx aimed at optimizing 1)biology (infection, blood
supply, bone graft), 2) mechanics (skeletal
stabilization)
Atrophy Fixation & bone grafting
Hypertrophy Rigid fixation
ii. Malunion
Angulation in a long bone (> 15 degrees) & Marked
rotational deformity
Osteotomy & internal fixation +/- bone graft

Shortening (> 3cm) in 1 of the lower limbs


A raised boot
OR
Bone operation

111

Together in Delivering Excellence (T.I.D.E)


Common/Possible Question in Exam + Answer Notes [any classification,staging, mnemonic etc]

Indication for ORIF


a. Failed conservative rx
b. 2# in 1 limb
c. Bilateral identical #
d. Intraarticular #
e. Open #
Emergency mx for open # 7 As
o ATLS

Correct shock , give blood >1.5 L lost or continued bleeding, control of bleeding may require surgery
o Assesment

Neurovascular status, soft tissues and photograph wound (reduces number of wound inspection)
o Antisepsis

Take swab form wound

Use copious irrigarion with sterile 0.9% saline

Cover with a large antiseptic soaked dressing


o Alignment

Align # and splint (+ pain relief)


o Anti tetanus

o
o

Cx

Check status and immunize appropriately


Antibiotics
rd

3 gen cephalosporin eg. Ceftriaxone 1g/24h +/- IV mtronidazole if grossly contaminated


Analgesia

Intravenous opiate analgesia titrated to effect

Immediate
Internal bleeding
External bleeding
Organ injury
Nerve/ skin injury
Vessel injury (limb
ischaemia)

Later local
Skin necrosis/ gangrene
Pressure sores
Infection
Non/ delayed union

Later general
Venous/ fat embolism
PE
Pneumonia
Renal stones

Gustillo classification of open #


o Type I - low energy wounds <1cm long eg coz by bone piercing skin
o Type II low energy wound >1cm, causing moderate tissue damage
o Type III all high energy injuries irrespective of wound size

IIIA adequate local soft tissue coverage

IIIB - inadequate local soft tissue coverage

IIIC implies arterial injury needing repair


Methods of traction (hold the reduction)
o Skin traction uses adehesive strapping to attach the load, cons : load cannot be great & sensitivity to adhesive
o Fixed - Thomas splint, weight can be added over a pulley to relieve pressure on ischial tuberosity
o Skeletal traction pin thru bone, bigger forces can be employed
o Balanced weight of limb against the load, enable pt to easily lift leg off bed
o Gallows suitable for children up to 2 years, buttock rise above the head
FES
o Clinical condition in which circulating fat emboli/macroglobules lead to multisystem dysfunction
o Common in long bone fracture and pelvic fracture, more frequent in closed fracture than open fracture.
o Usually asymptomatic, few patient develop S&S of multiorgan dysfunction ( triad of brain, skin, lungs)
o Source of fat emboli bone marrow
o Management is supportive, with an estimated mortality rate of 5 -15 %
site of bone graft taken?
o

ilium and the fibula are the most common sites for bone-graft harvesting

Radiographic Determinants of Healing:


- restoration of cortical continuity (look for healing on 4 cortices - AP and lateral views);
- loss of distinct fracture line;
- presence of callus (w/ IM nail or external fixator)

112

Together in Delivering Excellence (T.I.D.E)


Gouty Arthritis [Farhan ]
HISTORY
Monoarticular pain with edema and inflammation
>50% occurs at MTP of big toe (podagra)
Other joints are ankle, foot, small joints of hands, wrist,
elbow and knee
Can be polyarticular (10%)
Male predominance (4:1)
Caused by deposition of monosodiumurate crystals in/near
joints
Precipitated by e.g. trauma, surgery, starvation, infection or
diuretics (thiazide)
Acute vs chronic attack
Causes (primary vs secondary):
i. Hereditary
ii. High purine diet
iii. Alcohol excess
iv. Diuretics
v. Leukaemia
vi. Cytotoxics (tumour lysis)
Comorbidities:
i. Cardiovascular disease
ii. Hypertension
iii. Diabetes mellitus
iv. Chronic renal failure
Complications:
i. Severe degenerative arthritis
ii. Secondary infections
iii. Urate nephropathy
iv. Renal stones
v. Fractures (in joints with tophi)
INVESTIGATION

PHYSICAL EXAMINATION
Acute:
Pyrexia often present
Red, warm, tender and painful joint(s)
Usually monoarticular
Most common MTP joints
Chronic:
Tophi (chalky coloured nodules on pinna, tendons,
joints)
Chronic arthritis (secondary OA, restriction of joint
movements)
MANAGEMENT
Asymptomatic hyperuricaemia should not be treated
1. Treat acute attacks
Use high-dose NSAIDs or coxib
If contraindicated; use colchicine
Steroids can be used
Rest and elevate joints
Ice packs may help

2. Prophylaxis for acute flares


Allopurinol 100mg/24hr, increasing every 2 weeks
(max 900mg/24hr)
o SE: rash, fever, WCC low
o Can trigger acute attack, so wait until 3 weeks
after acute attack and cover with NSAIDs or
colchicine
1. Serum urate high level not diagnostic, low level does not
o Excreted through kidney (beware in renal
exclude diagnosis
impairment)
2.
3.
4.
5.

WBC often raised


ESR often raised
Synovial fluid analysis Negative birefringent (under
polarized light), needle shaped crystals
X-rays soft tissue swelling, punched-out erosions
(difference from RA: maintenance of joint space, does not
involve joint capsule, absence of periarticular osteopenia)

Febuxostat (80mg/24hr)
o Alternative if allopurinol is contraindicated or
not tolerated
o SE: increase LFT
o Metabolised and excreted by liver
3. Diet (low purine, hydration) and exercise

113

Together in Delivering Excellence (T.I.D.E)


Differential diagnosis

Differentiating
signs

Pseudogout
Presentation may be
identical to that of gout
Is less common in young
age (<50)
Is more likely to affect
wrist and knee joints
(proximal)

Differentiating
tests

X-ray: Chondrocalcinosis

(radiographic calcification
of cartilage in certain

joints) is usually present.


Ultrasound: Calcium
pyrophosphate deposits
are found deeper in the
cartilage and are less

homogenous (lumpybumpy) than the


superficial double contour
sign seen in gout.
Synovial fluid: Calcium
pyrophosphate crystals;
are rhomboid-shaped,
weakly positively
birefringent crystals.

Rheumatoid Arthritis
A chronic systemic
inflammatory disease.
Check extraarticular
involvement;
lympadenopathy,
vasculitis, pleural &
pericardial effusion
Chronic tophaceous and
polyarticular gout may
present like RA, and tophi
can be misdiagnosed as
rheumatoid nodules.
Symmetrical swollen,
painful, and stiff small
joints of hands and feet.
Worse in morning.

X-ray: Juxta-articular
osteopenia
Synovial fluid is
inflammatory (WCC 1
50000/mm^3), but no
monosodium urate
crystals are found.
Anticyclic citrullinated
peptide (anti-CCP) has a
high specificity (98%), but
a low sensitivity and it
may be useful in the early
detection of patients who
will have severe RA.

Septic Arthritis
Presentation may be identical
to that of gout.
Occurs in both sexes and at
any age.
Risk factors for infection,
such as intravenous drug use
and immunocompromise,
may be present.

Synovial fluid microscopy and


culture may be Gram positive
and show growth.
Blood cultures may grow the
causal bacteria.
X-ray & CRP may be normal.
Co-existence of crystals and
infection in the joint is not
uncommon.

Purine content food


High (avoid) liver, anchovies, sardines, mackerel (kembung), crab, shrimps, scallops and other shellfish
Moderate (eat in moderation) red meat, chicken, soya beans, peas, spinach, cauliflower, asparagus, mushrooms
Low dairy products, eggs, pasta/noodles, bread, fruits, nuts, cereal products

114

Together in Delivering Excellence (T.I.D.E)


Bone Tumor [Rafidah]
HISTORY

PHYSICAL EXAMINATION

Epidemiology

Malignant common in adult (except osteosarcoma and ewing


sarcoma in young)
In pt over 50 years bone mets seen more frequently than all primary
malignant bone tumour

Ill or in pain
Cachexia
Pale

Specific

Risk factor

General

Smoking
Previous irradiation
Bone mets previous history of malignancy(breast, prostate, kidney,
lung, thyroid, bladder and GIT)

Lump
Inspection (location, size, number, shape, skin changes, surrounding
skin, scar, dilated vein, discharge)
Palpation (tenderness, temperature, surface, margin ,arise from the
bone (non mobile side horizontally and vertically) or muscle (mobile
horizontally but not vertically) or above the muscle (mass disappear
when the muscle contracted), consistency, pulsatile, expansile,
slipping sign, fluctuation test, transillumination test)
Auscultation(bruit)
If the mass near to the joint, assess for effusion and ROM
Assess the peripheral nerve and peripheral pulses

S&S

Mass palpable, rapidly progressive increase in size


Pain at rest/ night pain
LOA,LOW
Loss of function
Asymptomatic but later presented with pathological # - any break in
midshaft should be regarded a pathological until proven otherwise
Neurological symptom paraesthesia or numbness cause by pressure
or stretching of peripheral nerve
Back pain mets
Systemic review ask about symptom in respective system that
suggestive of malignancy (To look for primary tumour causing
metastasise to the bone)

Complication

Metastasize lung

Pathological fracture

Other examination:
Metastasise:

Spine deformity or tenderness

Lungs

liver ascites, hepatosplenomegaly

lymph node

Primary cancer (based on history)


Thyroid
Breast
Lungs
Prostate (DRE)
Kidney

INVESTIGATION

MANAGEMENT
I.

Laboratory

FBC
Serum ALP
ESR
Serum protein electrophoresis abnormal globulin fraction in
myeloma
Bence jones protein in urine - myeloma

Imaging

Plain radiograph
1) Description
Type of xray
Age (skeletally immature vs mature)
Solitary or multiple
Lucency(lytic) vs density(blastic)
Site(bone and area metaphysis,diaphysis, epiphysis)
Feature:
a) zone of transition(wide or narrow)
b) periosteal reaction
c) cortical destruction or cortical thickening
- stipled calcification inside vacant area is characteristic of cartilage tumour

CT&MRI

-to assess true extent opf tumour and its relationship to surrounding
structures
- deciding how much tissue to remove in local extent of the tumour

Bone scan

-to reveal site of small tumour or presence of skip lesion/silent secondary


deposits

Biopsy for diagnosis and planning treatment


Others: chest xray, ct chest

ii.

Surgical
Tumour excision :
Intracapsular excision
Marginal excision
Wide local resection
Radical resection
Limb salvage
Done if certain that there are no skip lesions and functional limb
can be preserved
Amputation
High grade lesion
Doubt whether lesion intracompartmental
Local control for tumour resistant to chemo and radio rx

Non surgical

Multi agent chemotherapy


methotrexate, doxorubicin, cyclophosphamide, vincristine and cisplatinum
rx 8-12 weeks preop,
maintenance another 6-12month postop
Radiotherapy
adjunctive rx for high grade tumour
inaccessible sites
inoperable because of their size, proximity to major blood vessel
for marrow cell tumour such as multiple myeloma
for metastatic deposits
for palliative local tumour control where no surgery is planned

115

Together in Delivering Excellence (T.I.D.E)


1.

Enneking classification

GRADE (SURGICAL)

G0 BENIGN
G1 LOW GRADE MALIGNANT
G2 HIGH GRADE MALIGNANT

SITE

T0 BENIGN INTRACAPSULAR &


INTRACOMPARTMENTAL
T1(A) INTRACOMPARTMENTAL
T2(B) EXTRACOMPARTMENTAL

METASTASIS

M0 NO REGIONAL/DISTANT METS
M1 REGIONAL / DISTANT METS

2.

What is intracompartmental and extracompartmental?


Intracompartmental: lesion confined to an enclosed tissue space (e.g. a bone, a joint cavity, muscle group within its
fascial envelope)
Extracompartmental: extend into interfascial or extrafascial plane with no natural barrier to proximal or distal spread

3.

Malignancy vs benign in radiograph

Malignant
ill defined
Irregular Break of cortex
Periosteal reaction codmans triangle, lamellated, sun burst
appearance
Wide zone of transition

Benign
Well defined
Regular cortical destruction
Thick, wavy, uniform callus formation due to chronic
irritation
Narrow zone of transition

4. How is it benign periosteal reaction can be differ from the malignant type?
In the case of benign, slowly growing lesion, the periosteum has time to lay down thick new bone and remodel it into a more
normal-appearing bone while aggressive periostitis dows not have time to consolidate
5. Complication of biopsy
Hemorrhage, wound breakdown, infection, pathological fracture
6. Commonest site for bone mets?
Vertebrae,pelvis, proximal half of the femur and humerus

116

Together in Delivering Excellence (T.I.D.E)


Diabetic Foot Ulcer [Zuraidah]
HISTORY
Definition:
A diabetic foot is a spectrum of condition as a cx of DM to the foot,
ranging from nerve, skin, fascia, muscle, tendon, bone and blood
vessels
General and Medical History

History of presenting foot complaints and duration

painful/not, rest pain/ claudication

Duration of diabetes, management and control

Cardiovascular, renal, ophthalmic evaluation & other co morbidities

Social history alcohol / tobacco / occupation / dietary habits

Current medication and antibiotic use

Allergies

Past Medical & Surgical history

Cultural habits walks barefoot / wets feet at work / wear socks /


walks alot

Patients perception of Diabetes Mellitus, necessity of weight and diet


control

Able to afford diabetic drugs


History of Foot Problems

Daily activity and current diabetic foot status

Footwear shoes / slippers / sandals / use different footwear / Fit

Foot-care aware of foot problem / inspect foot / wash feet / proper


nailclipping / attend podiatry

Callus formation

Deformities and previous foot surgery

Neuropathy and ischemic symptoms

Skin & nail problems sweaty feet / fungal infections / skin disease /
blisters/ Ingrown toenails
History of Foot Ulcer

Site, size, duration, odour and type of drainage

Precipitating event or trauma

Recurrences number of times

Associated infections

Frequency of hospitalizations and treatment given

Wound care / measures to reduce plantar pressure

Patient compliance

Previous foot trauma or surgery

Features of Charcots joint


INVESTIGATION
Laboratory
1. Full Blood Count
2. Erythrocyte Sedimentation Rate (Esr)
3. Blood Glucose
4. Renal Function Test
5. Urinalysis
6. Glycosylated Haemoglobin Level (Hba1c)
7. Ulcer Swab Culture
8. Tissue Biopsy
9. X-ray- AP and lateral view to exclude osteomyelitis and gas
gangrene
10. Ankle-brachial systolic index( by Doppler u/s)
Normal->0.9
>0.5-Ulcer healing
<0.5-Impendent gangrene
<0.4-Amputation needed

PHYSICAL EXAMINATION
General

alert conscious, any septic looking

Vital signs
Specific

System CVS, RESPI, CNS look for cx


Local examination
1) Features of neuropathic vs ischemic ulcer (see table in notes)
2) Ulcer assmnt
Site
Size
Wound
bed

Infection
signs

Exudate
Wound
edge

Depend on types
Length, width, depth and location, preferably with clinical
photograph
Appearance:
Black (necrosis)
Yellow, red, pink
Undermined
Odour
Be aware that some signs (fever, pain, increased white blood
count/ ESR) may be absent. Evaluate the ulcer for
signs of infection, inflammation and oedema.
Copious, moderate, mild, none
Callus and scale, maceration, erythema, oedema

Evaluation of vascular status


pulses (dorsalis pedis, posterial tibial, popliteal, femoral) / capillary
refill
edema/ temperature gradient/ color changes
changes of ischemia (trophic changes)
Evaluation of neurological status

vibration perception : tuning fork 128hz

pressure & touch :cotton wool(light), monofilament 10g (Semmes


Weinstein)

-pinprick (pain) /2 point discrimination / temperature

deep tendon reflex (ankle,knee)/ clonus /Babinski /Rombergs test

MANAGEMENT
The aim is to obtain wound closure as soon as possible and to prevent
recurrence.
Principles of Treatment

Debridement of necrotic tissue

Wound care

Reduction of plantar pressure (off-loading)

Treatment of infection

Vascular management of ischaemia

Medical management of co morbidities

Surgical management to reduce or remove bony prominences and /


orimprove soft tissue cover

Reduce risk of recurrence

117

Together in Delivering Excellence (T.I.D.E)


Common/Possible Question in Exam + Answer
1) Pathophysiology of the ischemic foot? Occlusive vascular dz involving both microangiopathy & atherosclerosis of large & medium sized artery.
2) pathophysiology of diabetic neuropathy and types of neuropathy?
accumulation sorbitol + fructose in Schwann cells -- Increase IC osmolality --influx of water caused osmotic cell injury -- damage schwann cell (segmental
demyelination ) -- axon degeneration irreversibly -- disrupt neural function -- Diabetic neuropathy

Characteristics

Clinical
presentations

Sensory
neuropathy
Loss of protective sensation
No perception of shoes
rubbing or temp changes

Autonomic
neuropathy
Reduced sweating - dry cracked skin
Increased blood flow - warm foot

Motor
neuropathy
Dysfx of the motor nerves that control the movement of the
foot. Limited joint mobility may increase plantar pressure
Foot deformities develop

Unaware of a foot ulcer/ lack


of discomfort when a wound is
being probed

Dry skin with cracks and fissures


Bounding pulses
Dilated dorsal veins
Warm feet

Hammer toes
High medial Longitudinal arch, leading to prominent
Metatarsal heads and pressure points over the plantar forefoot
Clawed toes
Altered gait

3) Wagners classification?

4) How to monitor diabetic at annual review?

Eyes : visual acuity, fundoscopy

Sensory system : touch, pinprick & vibration

Deep tendon reflex

CVS: BP, peripheral pulses

Biochemistry : urine & blood sugar, albumin, Hba1c, creatinine

Notes
Neuropathic vs ischemic ulcer
Clinical signs
Foot
deformities
Foot temperature/
footpulse

Neuropathic ulcer

Clawed toes, possible high arch, possible Charcot deformities

Warm, palpable pulse

Ischaemic ulcer
No specific deformities. Possible absent toes/forefoot from
previous amputations
Cold or decreased temperature, pulse may be absent or
reduced
Pale/bluish. Pronounced redness when lowered (dependent

Skin colour

Normal or red

Skin condition

Dry skin due to decreased sweating

Thin, fragile and dry.

Ulcer location

On the plantar aspects (forefoot 80%) of the foot/toes,

Distal/tips of the toes, heel, or margins of the foot

Callus present

Commonly seen on the weight bearing areas and is generally thick

Not usually. If present, distal eschar or necrosis

Ulcer characteristics

Usually painless, with a punched out appearance (granulation or


deeper base) surrounded by callus

rubor), blanching on elevation

Painful, especially with necrosis or slough


Sensation may be present but decreased if there is associated

Sensation

Reduced or absent sensation to touch, vibration, pain, and pressure

Ankle reflexes

Usually not present

Usually present

Foot pulses

Present and often bounding. Dilated, prominent veins

Absent or markedly reduced

neuropathy

118

Together in Delivering Excellence (T.I.D.E)


Osteoarthritis/Rheumatoid Arthritis [Ain]
HISTORY
Osteoarthritis :
Chronic joint disorder in which there is progressive
softening and disintregation of articular cartilage
accompanied by new growth of cartilage and bone at
the joint margins (osteophytes) and capsular fibrosis.
Rheumatoid arthritis :
Chronic systemic inflammatory disorder involving mainly
the joints with a peripheral symmetrical nonsuppurative disorder.

PHYSICAL EXAMINATION
OA

The information needed to help diagnose includes:

Description of the symptoms :


Pain,swelling,stiffness,deformity,reduced range of
motion
Details about when and how the pain or other
symptoms began : acute vs chronic
Do you have any general symptoms that seem to
affect your whole body, such
as fatigue, weight loss, or fever? : RA involved
systemic, OA doesnt cause whole body symptom
Does exercises help your pain or make it worse.
Details about other medical problems that exist :
in RA most common involves lung, heart n eyes
Location of the pain, stiffness or other symptoms :
which joint, unilateral/bilateral, duration of
stiffness
How the symptoms affect daily activities driving,
working,
List of current medications and complication of
drugs : prolong NSAID used, methotrexate,
cytotoxic drug in RA
- Assess detailed about risk factor in hx
- Assess complication
RA involves systemic response- osteoporosis,
carpal tunner syndrome, heart problems, lung
disease
( pleuritic/effusion/interstitial pulmonary
fibrosis),anemia, cervical myelopathy,
mononeuritis multiplex
OA more localized foot pain, reduce mobility,
septic arthritis after arthroplasty, gout

Antalgic gait

Joint swelling, warmth, and tender.(the small joints of


the hands and feet are affected in a relatively symmetric
distribution)

Presence of fluid on the joint-(effusion)

The presence of bumps (rheumatoid nodules) over


pressure points in the body.

Extra-articular involvement of organs such as the skin,


heart, lungs, and eyes.

Varus deformity
Quadriceps wasting
Bony bumps on the finger joint
closest to the fingernail
(Heberden's nodes),bony
bumps on the middle joint of
the finger (Bouchard's nodes),
or bony bumps at the base of
the thumb.

Tenderness and/or swelling


in weight-bearing joints such
as the hips and knees.

Pain, limited movement,


and/or a creaking noise or
feeling (crepitus) that occurs
when the joints are moved.
Eg; knee examination

RA

Swelling and tender joint


Crepitus
Limited rom
Contracture
Varus stress test positive
Joint line tendernes
Osteophytes

Decreased range of motion


Joint and tendon destruction: deformities such as ulnar
deviation, boutonniere and swan-neck deformities,
hammer toes, genu varus/valgus

Important findings in the physical exam include the:

Pattern of symptoms in the affected joints.


Presence of swelling or tenderness in the joints.

119

Together in Delivering Excellence (T.I.D.E)


INVESTIGATION
Laboratory :
FBC
Inflammatory marker : ESR,CRP
Rheumatoid factor : RA
Synovial fluid analysis : high WBC indicate
inflammatory or infection response/other
cause of arthritis eg. Gout, RA, septic
arthritis
Anti-citrullinated protein antibody
(anti-CCP ) : in RA
Anti-nuclear antibody (ANA)- TRO other
form of arthritis eg. SLE

MANAGEMENT
Main principle in OA treatment :
1) Relieve pain analgesic, NSAID
2) Increase movement physiotheraphy ( increase in range n power)
3) Reduce load walking stick,soft-soled shoes, weight reduction, avoid
prolong activity
Principle in RA :
To control joint inflammation
To prevent joint damage and disability

Pharmacological
(medical)

# there is no specific lab ix for OA compared to


RA,mainly its by clinical dx as well as x-ray.

OA
-analgesic eg.PCM,tramal
-steroid oral/injection
- NSAID eg. Ibuprofen,
naproxen, celecoxib

Imaging :
X-ray
OA

RA

-Loss/narrowing of joint
space.
-Osteophytes.
-Subchondral cyst.
-Subchondral sclerosis.
-deformity

-Juxta-articular
osteopenia.
-Soft tissue swelling.
-Loss of joint space.
Advanced :
-Bony erosion
-subluxation

MRI

To detect arthritis. MRI can be helpful in


evaluating joint damage, particularly
damage to the spine, knee, or shoulder.

Non medical/
conservative

-Rest, reduce weight,


use of public transport
-physiotheraphy
-occupational theraphy

Surgery

-osteotomy
-arthroplasty (eg.total
knee replacement)
-arthrodesis

RA
-oral steroid eg. Prednisone
- NSAID
- DMARD (disease modifying
anti-rhuematic drugs)
Within 3 months after onset
of symptom eg.
Methotrexate, penicillamine,
tumor necrosis factor
antagonist
-occupational theraphy
-physiotheraphy

-arthroscopy/synovectomy
(to remove debris or inflamed
tissue from a joint)

-arthroplasty (joint
replacement)
-carpal tunnel release
-cervical spinal fusion
-finger & hand surgery

To track the progress of disease. In repeat


scans, MRI can determine how fast the
arthritis is progressing

Common/Possible Question in Exam + Answer


1.

Classification of osteoarthritis

Common OA question.
1. What is radiological features
changes? - as mentioned above
2. Whats common joint affected ?
knee joint
3. Treatments?
as mentioned above
4. Indication for surgery ?
severe intolerable pain,pain
at rest,affect normal
function

120

Together in Delivering Excellence (T.I.D.E)


2.

Difference of OA/RA

3.

American rheumatic association criteria for RA

RF RISES - R : rheumatoid factor


F : finger/hand joint involved for > 6 weeks
R : rheumatoid nodules
I : involvement of 3 or more joint area
S : stiffness (morning) >1 hour for > 6 weeks
E : erosion on xray
S : symmetrical arthritis > 6 weeks
4.

Detailed history about risk factor


OA (mainly affect cartilage)
Age
Older age >45 y.o
Family hx

Sex

Equal
Men < 55 y.o
Women >later in life
Non-immune
-age,genetic,previous injury to
joint,obesity,hormone

Trigger

RA (mainly affect synovium)


At any age even in children
Average : 30- 50 y.o
More in RA
Women > men 3:1

Autoimmune
-genetic,infection,hormone,smoking

bilaterally,symmetrical,destructive
and performing polyarthropathy.

121

Together in Delivering Excellence (T.I.D.E)


PERSONAL NOTES

122

Das könnte Ihnen auch gefallen